You are on page 1of 234

Website: tailieumontoan.

com

LỜI NÓI ĐẦU


Nhằm đ{p ứng nhu cầu của các bạn học sinh yêu toán Page tài liệu toán học đã
sưu tầm và tổng hợp 47 đề thi học sinh giỏi toán lớp 10 và lời giải chi tiết. Toán lớp 10
nâng cao kế thừa rất nhiều từ to{n THCS như phương trình, bất đẳng thức, hệ phương
trình, các bài toán nghiệm nguyên, to{n Logic.... tuy nhiên được nâng cao lên một nấc
mới, cùng với đó l| sự bổ sung thêm kiến thức về nhiều phần quan trọn, hay và khó
như hình tọa độ trong mặt phẳng, hình vecto, dấu của tam thức bậc 2.
Từ lớp 9 lên lớp 10 ít nhiều các bạn sẽ có nhiều bỡ ngỡ, một mặt là do kiến thức
lớp 10 tuy kế thừa nhiều phần lớp 9 nhưng khó hơn rất nhiều, phần khác là do các bạn
đang t}m lý của người mới thi lên một cấp mới có phần xả hơi sau th|nh công của kì
thi.Để chuẩn bị tốt cho các kì thi học sinh giỏi lớp 11, lớp 12 thì việc rèn luyện chắc kiến
thức lớp 10 l| điều không thể thiếu. Khi lên cấp 3 kiến thức các bạn học sẽ chia làm
nhiều chủ đề, do đó c{c bạn phải rèn luyện nhiều phần, cùng với đó c{c mộn lý, hóa,
sinh..... đều rất khó và mới, có thể nói lên cấp 3 là một cấp học mới hoàn toàn so với cấp
2.
Cũng như nhiều tập đề khác, tập đề này có 2 phần rõ r|ng đó l| đề thi v| đ{p {n
chi tiết, có những bài toán khó sẽ được trình bài nhiều cách và nhận xét. Các bạn chú ý
thường các bài toán sẽ có nhiều cách giải khác nhiêu, vì thế ngoài các giải được đề cập
trong đ{p {n c{c bạn nên tư duy tìm thêm lời giải mới, không nhất thiết phải là quá
nhiều đề mà chúng ta cần l|m kĩ v| nghiên cứu sâu.
Cuối cùng chúc các bạn có những phút giây hứng thú thi làm toán và có kết quả
tốt nhất trong các kì thi HSG!

1
Page: Tài Liệu Môn Toán
Website: tailieumontoan.com

MỤC LỤC

Phần 1. Đề luyện thi

Phần 2. Đ{p {n

 Đề 1:______________________________________________________Trang <.49
 Đề 2:______________________________________________________Trang <.53
 Đề 3:______________________________________________________Trang <.56
 Đề 4:______________________________________________________Trang <.59
 Đề 5:______________________________________________________Trang <.63
 Đề 6:______________________________________________________Trang <.67
 Đề 7:______________________________________________________Trang <.70
 Đề 8:______________________________________________________Trang <.74
 Đề 9:______________________________________________________Trang <.78
 Đề 10:______________________________________________________Trang <.82
 Đề 11:______________________________________________________Trang <.86
 Đề 12:______________________________________________________Trang <.91
 Đề 13:______________________________________________________Trang <.97
 Đề 14:______________________________________________________Trang <..102
 Đề 15:______________________________________________________Trang <..108
 Đề 16:______________________________________________________Trang <..110
 Đề 17:______________________________________________________Trang <..115
 Đề 18:______________________________________________________Trang <..119
 Đề 19:______________________________________________________Trang <..121
 Đề 20:______________________________________________________Trang <..125
 Đề 21:______________________________________________________Trang <..129
 Đề 22:______________________________________________________Trang <..133
 Đề 23:______________________________________________________Trang <..139
 Đề 24:______________________________________________________Trang <..143
 Đề 25:______________________________________________________Trang <..151
 Đề 26:______________________________________________________Trang <..153
 Đề 27:______________________________________________________Trang <..157
 Đề 28:______________________________________________________Trang <..161
 Đề 29:______________________________________________________Trang <..163
 Đề 30:______________________________________________________Trang <..165
 Đề 31:______________________________________________________Trang <..167
 Đề 32:______________________________________________________Trang <..170
 Đề 33:______________________________________________________Trang <..171
 Đề 34:______________________________________________________Trang <..176
 Đề 35:______________________________________________________Trang <..178
 Đề 36:______________________________________________________Trang <..181
 Đề 37:______________________________________________________Trang <..184
 Đề 38:______________________________________________________Trang <..186
 Đề 39:______________________________________________________Trang <..187
2
Page: Tài Liệu Môn Toán
Website: tailieumontoan.com

 Đề 40:______________________________________________________Trang <..193
 Đề 41:______________________________________________________Trang <..196
 Đề 42:______________________________________________________Trang <..200
 Đề 43:______________________________________________________Trang <..203
 Đề 44:______________________________________________________Trang <..207
 Đề 45:______________________________________________________Trang <..212
 Đề 46:______________________________________________________Trang <..214
 Đề 47:______________________________________________________Trang <..216

3
Page: Tài Liệu Môn Toán
Website: tailieumontoan.com

ĐỀ 1
PAGE TÀI LIỆU TOÁN HỌC ĐỀ CHỌN HỌC SINH GIỎI TOÁN LỚP 10

MÔN TOÁN
(Thời gian: 180 phút, không tính thời gian giao đề)
Câu 1 (3,0 điểm)
a) Cho phương trình bậc hai x2  2mx  3m  2  0 , trong đó x là ẩn, m là tham số. Tìm
tất cả các giá trịcủa m để phương trình đã cho có hai nghiệm x1 , x2 và x12  x22 đạt giá trị
nhỏ nhất.
b) Cho tam thức bậc hai f  x   ax 2  bx  c, a  0 . Chứng minh rằng nếu f  x   0 với
mọi x  thì 4a  c  4b .
Câu 2 (2,0 điểm)
a) Giải phương trình x  2  3x  1  2 x  3  x  
 x  y   x  xy  y  3  3  x  y   2
 2 2 2 2

b) Giải hệ phương trình   x, y  


 x  6  y  3   x  2x  8

2

Câu 3 (2,0 điểm)


a) Cho a, b, c là các số thực dương thỏa mãn abc  1 . Chứng minh rằng
a b c 3
  
 a  1 b  1  b  1 c  1  c  1 a  1 4
b) Giải bất phương trình 3
3  x  1 x  2  x  
Câu 4 (3,0 điểm)
a) Cho tam giác ABC  AB  AC  nhọn, không cân, nội tiếp đường tròn (O), trọng
tâm G và a  BC, b  CA, c  AB . Gọi M l| trung điểm của cạnh AC. Chứng
minh rằng nếu bốn điểm A, O, M, G cùng nằm trên một đường tròn thì
b2  c2  2a 2 .
b) Cho tam giác ABC không vuông và a  BC, b  CA, c  AB . Chứng minh rằng
nếu a 2  b2  2c2 và tan A  tan B  2 tan C thì ABC là một tam giác cân.
c) Trong mặt phẳng với hệ trục tọa độ vuông góc Oxy ; cho tam giác ABC có tọa độ
 11 1 
t}m đường tròn ngoại tiếp, trong tâm lần lượt có tọa độ là I  4;0  , G  ;  . Tìm
 3 3
tọa độ c{c đỉnh A, B, C của tam giác ABC biết rằng đỉnh B nằm trên đường

4
Page: Tài Liệu Môn Toán
Website: tailieumontoan.com

thẳng  d  : 2 x  y  1  0 v| điểm M  4;2  nằm trên đường cao kẻ từ đỉnh B của


tam giác ABC.
ĐỀ 2
PAGE TÀI LIỆU TOÁN HỌC ĐỀ CHỌN HỌC SINH GIỎI TOÁN LỚP 10

MÔN TOÁN
(Thời gian: 180 phút, không tính thời gian giao đề)

Câu 1 (3,0 điểm)



(2 x  3) 4 x  1  (2 y  3) 4 y  1  2 (2 x  3)(2 y  3)
1. Giải hệ phương trình: 
 y  x  4 xy

2. Tìm tất cả các hàm số f :  thoả mãn:

 1  f ( x)
f ( x  y)  f ( x)  y x, y  và f    2 x  0 .
 x x
Câu 2 (2,0 điểm)
Tìm tất cả các số nguyên tố p , q sao cho  7 p  4 p  7q  4q  chia hết cho pq .

Câu 3 (2,0 điểm).


Cho tứ giác ABCD ngoại tiếp được một đường tròn. Một đường thẳng  đi qua A cắt
đoạn thẳng BC, tia đối của tia CD tương ứng tại E, F (E, F không trùng với B, C). Gọi
I1 , I 2 và I 3 lần lượt l| t}m đường tròn nội tiếp của các tam giác ABE, ECF và FAD.
Tiếp tuyến của đường tròn ( I1 ) song song với CD (ở vị trí gần CD hơn) cắt  tại H.
Chứng minh rằng H là trực tâm của tam giác I1I 2 I 3 .

Câu 4 (2,0 điểm).


Xét các số thực dương a, b, c thỏa mãn a  2b  3c  20. Tìm giá trị nhỏ nhất của biểu
thức
3 9 4
L  abc   
a 2b c
Câu 5 (1,0 điểm).

5
Page: Tài Liệu Môn Toán
Website: tailieumontoan.com

Tìm tất cả các tập hợp X là tập con của tập hợp số nguyên dương thoả mãn các tính
chất: X chứa ít nhất hai phần tử và với mọi m, n  X , m  n thì tồn tại k  X sao cho
n  mk 2 .

ĐỀ 3
PAGE TÀI LIỆU TOÁN HỌC ĐỀ CHỌN HỌC SINH GIỎI TOÁN LỚP 10

MÔN TOÁN
(Thời gian: 180 phút, không tính thời gian giao đề)

Câu 1 (3,0 điểm).


 xy  3 y 2  8 y  7  3xy 3  6 y 2

a) Giải hệ phương trình: 


xy  y  7  y 5  3 y  1  
b) Cho đa thức với hệ số thực P  x   x 4  ax3  bx 2  cx  d thoả mãn
P 1  3, P  3  11, P  5  27 . Tính P  2   7 P  6  .

Câu 2 (1,5 điểm). Tìm tất cả các cặp số nguyên dương  x; y  thoả mãn phương trình:

x  4 y 2  28  17  x 4  y 4  14 y 2  49 
2 2

Câu 3 (3,0 điểm). Cho tam giác nhọn ABC không cân nội tiếp đường tròn (O), có đường
cao AH v| t}m đường tròn nội tiếp là I. Đường thẳng AI cắt lại đường tròn (O) tại điểm
thứ hai M. Gọi A' l| điểm đối xứng với A qua O. Đường thẳng MA' cắt c{c đường thẳng
AH, BC theo thứ tự tại N và K.
1) Chứng minh rằng tứ giác NHIK nội tiếp đường tròn.
2) Đường thẳng A'I cắt lại đường tròn (O) tại điểm thứ hai D, hai đường thẳng AD và
BC cắt nhau tại điểm S. Chứng minh rằng nếu AB  AC  2BC thì I là trọng tâm của
tam giác AKS.
Câu 4 (1,5 điểm). Cho các số thực a, b, c, d thoả mãn 4a 2  b2  2 và c  d  4 . Tìm giá trị
lớn nhất của biểu thức P  2ac  bd  cd .
Câu 5 (1,0 điểm). Cho tập hợp M gồm 2014 số dương a1 , a2 ,..., a2014 . Xét tất cả các tập con
khác rỗng Ti của M, gọi si là tổng các số thuộc tập con Ti . Chứng minh có thể chia tập
hợp tất cả các số si được thành lập như vậy thành 2014 tập con khác rỗng không giao
6
Page: Tài Liệu Môn Toán
Website: tailieumontoan.com

nhau, sao cho tỉ số của hai số bất kì thuộc cùng một tập tập con vừa được phân chia
không vượt quá 2.
ĐỀ 4
PAGE TÀI LIỆU TOÁN HỌC ĐỀ CHỌN HỌC SINH GIỎI TOÁN LỚP 10

MÔN TOÁN
(Thời gian: 180 phút, không tính thời gian giao đề)
1 1
Câu 1. a) Giải phương trình   2 x 
x 2  x2
b) Cho phương trình bậc hai x2  2mx  m2  2m  4  0 ( x là ẩn và m là tham số).
Tìm tất cả các giá trị thực của m sao cho phương trình đã cho có hai nghiệm
không âm x1 , x2 . Tính theo m giá trị của biểu thức P  x1  x2 và tìm giá trị
nhỏ nhất của P .
 x 2  xy  y 2  x  2 y  0
Câu 2. Giải hệ phương trình:   x, y  
 2 x  xy  y  2

Câu 3. Cho a, b, c l| độ dài ba cạnh của một tam giác không nhọn. Chứng minh rằng

 1 1 1
a 2
 b2  c 2   2  2  2   10
a b c 
Câu 4.
a) Cho tam giác ABC, nhọn, không cân và nội tiếp đường tròn  O; R  . Gọi G và M
lần lượt là trọng tâm tam giác ABC v| trung điểm cạnh BC. Chứng minh rằng
nếu đường thẳng OG vuông góc với đường thẳng OM thì
AC 2  AB2  2BC 2  12R2 .
b) Cho tam giác ABC có độ d|i c{c đường cao kẻ từ đỉnh A, B, C lần lượt là m, n, p .
Tính độ dài các cạnh AB, BC, CA theo m, n, p .
c) Trong mặt phẳng với hệ trục tọa độ Oxy , cho tam giác ABC có phương trình
đường thẳng chứa đường cao kẻ từ c{c đỉnh A, B, C lần lượt có phương trình l|
x  2 y  0, x  2  0, x  y  3  0 .
Tìm tọa độ c{c đỉnh A, B, C, biết rằng b{n kính đường tròn ngoại tiếp tam giác
ABC bằng 10 v| đỉnh A có ho|nh độ âm.

7
Page: Tài Liệu Môn Toán
Website: tailieumontoan.com

Câu 5. Cho tứ giác lồi ABCD và một điểm M nằm bên trong tứ gi{c đó (M không nằm
trên các cạnh của tứ giác ABCD). Chứng minh rằng tồn tại ít nhất một trong các góc
MAB, MBC, MCD, MDA có số đo không lớn hơn 450 .
ĐỀ 5
PAGE TÀI LIỆU TOÁN HỌC ĐỀ CHỌN HỌC SINH GIỎI TOÁN LỚP 10

MÔN TOÁN
(Thời gian: 180 phút, không tính thời gian giao đề)

Câu 1 (3,0 điểm).


 1 1
 x  2 y   x  3 y  3x  y 

1. Giải hệ phương trình   x, y  
 1  1  2  y 2  x2 
 x 2 y
2. Tìm tất cả các giá trị của a, b sao cho phương trình x3  ax2  bx  3a  0 có các
nghiệm đều là các số nguyên dương.
Câu 2 (2,0 điểm). Giả sử a, b, c, d là các số nguyên sao cho a  b  c  d là số nguyên
lẻ và chia hết a 2  b2  c2  d 2 . Chứng minh rằng với mỗi số nguyên dương n
đều có a  b  c  d chia hết a n  bn  cn  d n .
Câu 3 (3,0 điểm). Trong mặt phẳng cho tam giác ABC không cân ngoại tiếp đường
tròn tâm I. Lấy E và F lần lượt trên c{c đường thẳng AC và AB sao cho
CB  CE  BF , đồng thời chúng nằm về cùng một phía với A đối với đường
thẳng BC. C{c đường thẳng BE và CF cắt nhau tại G.
1. Chứng minh rằng bốn điểm C, E, I và G cùng nằm trên một đường tròn.
2. Trên đường thẳng qua G và song song với AC lấy điểm H sao cho HG  AF
đồng thời H khác phía với C đối với đường thẳng BG.
1
Chứng minh rằng EHG  ·CAB.
2
Câu 4 (1,0 điểm). Ký hiệu để chỉ tập hợp các số thực khác 0. Tìm tất cả các hàm
số f x{c định trên , nhận giá trị thực và thỏa mãn
 1 y  1 x
xf  x    yf ( y )   yf  y    xf ( x)  x, y  0
 y x  x y

8
Page: Tài Liệu Môn Toán
Website: tailieumontoan.com

Câu 5 (1,0 điểm). Một số nguyên dương được gọi là dễ thương nếu trong biểu diễn
thập phân của nó không có chứa chữ số 0 và tổng bình phương c{c chữ số của
nó là một số chính phương.
1. Tìm số dễ thương lớn nhất có hai chữ số.
2. Hỏi có hay không số dễ thương có 2013 chữ số?
ĐỀ 6
PAGE TÀI LIỆU TOÁN HỌC ĐỀ CHỌN HỌC SINH GIỎI TOÁN LỚP 10

MÔN TOÁN
(Thời gian: 180 phút, không tính thời gian giao đề)

Câu 1 (4,0 điểm).

1. Giải phương trình: x2  x  1  x2  x  1  2 x .


Giả sử phương trình bậc hai ẩn x ( m là tham số): x 2  2  m  1 x  m3   m  1  0
2
2.
có hai nghiệm x1 , x2 thỏa mãn điều kiện x1  x2  4 . Tìm giá trị lớn nhất và giá trị nhỏ
nhất của biểu thức sau: P  x13  x23  x1 x2  3x1  3x2  8 .


 x  x y  xy  xy  y  1
2 3 2

Câu 2 (1,5 điểm). Giải hệ phương trình:  4 ( x, y  ) .


 x  y  xy (2 x  1)  1

2

Câu 3 (1,5 điểm). Cho x, y là hai số thực dương thoả mãn điều kiện

x  1  x2  y  
1  y 2  2012 . Tìm giá trị nhỏ nhất của P  x  y .

Câu 4 (3,0 điểm).


1. Cho tam giác ABC nội tiếp trong đường tròn tâm O. Gọi M, N, P lần lượt l| điểm
đối xứng của O qua c{c đường thẳng BC, CA, AB; H là trực tâm của tam giác ABC và
L là trọng tâm tam giác MNP. Chứng minh rằng OA  OB  OC  OH v| ba điểm O, H,
L thẳng hàng.
2. Cho tứ giác lồi ABCD. Giả sử tồn tại một điểm M nằm bên trong tứ giác sao cho
MAB  MBC  MCD  MDA   . Chứng minh đẳng thức sau:

AB 2  BC 2  CD 2  DA2
cot   ,
2 AC.BD.sin 
trong đó  là số đo góc giữa hai đường thẳng AC và BD.
9
Page: Tài Liệu Môn Toán
Website: tailieumontoan.com

3. Trong mặt phẳng với hệ trục tọa độ vuông góc Oxy, cho tam giác ABC ngoại tiếp
đường tròn tâm I . C{c đường thẳng AI, BI, CI lần lượt cắt đường tròn ngoại tiếp tam
 7 5   13 5 
giác ABC tại c{c điểm M 1; 5 , N  ;  , P   ;  (M, N, P không trùng với các
 2 2  2 2
đỉnh của tam giác ABC). Tìm tọa độ c{c đỉnh A, B, C biết rằng đường thẳng AB đi qua
điểm Q  1; 1 v| điểm A có ho|nh độ dương.

ĐỀ 7
PAGE TÀI LIỆU TOÁN HỌC ĐỀ CHỌN HỌC SINH GIỎI TOÁN LỚP 10

MÔN TOÁN
(Thời gian: 180 phút, không tính thời gian giao đề)

Câu I (4 điểm)
x  y  m  2
1. Cho hệ phương trình  2 (trong đó m là tham số; x và y là
 x  y  2 x  2 y  m  4
2 2

ẩn)
a) Tìm m để hệ phương trình trên có nghiệm.
b) Tìm giá trị lớn nhất, nhỏ nhất của biểu thức A  xy  2  x  y   2011.

2. Tìm tất cả các giá trị m để phương trình sau có bốn nghiệm phân biệt đều lớn hơn 3
x4   3m  1 x 2  6m  2  0

Câu II (1,5 điểm)



 x  y  xy  1
Giải hệ phương trình 
 x 3  y 3  4

2 2

Câu III (1 điểm)


1 1 1
Chứng minh rằng nếu x, y là các số thực dương thì  
1  x  1  y  1  xy
2 2

Câu IV (3,5 điểm)


1. Trong mặt phẳng với hệ trục tọa độ Oxy, cho hai điểm A 1; 2  và B  4;3 . Tìm tọa độ
điểm M trên trục hoành sao cho góc AMB bằng 450 .

10
Page: Tài Liệu Môn Toán
Website: tailieumontoan.com

2. Trong mặt phẳng với hệ trục tọa độ Oxy, cho tam giác ABC nhọn với trực tâm H. Các
đường thẳng AH, BH, CH lần lượt cắt đường tròn ngoại tiếp tam giác ABC tại D, E, F (D
khác A, E khác B, F khác C). Hãy viết phương trình cạnh AC của tam giác ABC; biết rằng
 6 17 
D  2;1 , E  3; 4  , F  ;  .
5 5 
3. Cho tam giác ABC, có a  BC, b  CA, c  AB . Gọi I, p lần lượt l| t}m đường tròn nội
IA2 IB 2 IC 2
tiếp, nửa chu vi của tam giác ABC. Chứng minh rằng   2
c  p  a a  p  b b  p  c

ĐỀ 8
PAGE TÀI LIỆU TOÁN HỌC ĐỀ CHỌN HỌC SINH GIỎI TOÁN LỚP 10

MÔN TOÁN
(Thời gian: 180 phút, không tính thời gian giao đề)


 xy  y  3x  6 y  0
2 3

Câu I (4,0 điểm) 1. Giải hệ phương trình  2


 x  xy  3  0

2. Giải phương trình 18x  16  4 2 x2  5x  3  7 4 x2  2 x  2  7 2 x2  8x  6
Câu II (1,0 điểm) Tìm tất cả các bộ ba số hữu tỷ dương  m; n; p  sao cho mỗi một trong
các số
1 1 1
m ; n ; p
np pm mn
là một số nguyên.
Câu III (2,0 điểm) 1. Giả sử a, b, c là các số thực dương thỏa mãn
a 2012 b2012 c 2012
   2011. Chứng minh rằng luôn tồn tại số tự nhiên n sao cho
b2010 c 2010 a 2010
a n3 bn3 c n3 2011 a n 2 b n 2 c n 2
     n  n
bn1 c n1 a n1 2010 b n c a
2. Cho a, b, c là các số thực dương. Chứng minh rằng với mọi số tự nhiên m ta có bất
a m 3 b m 3 c m 3 a m  2 b m  2 c m  2
đẳng thức    m  m  m
bm1 c m1 a m1 b c a
Câu IV (2,0 điểm) Cho tam giác ABC nhọn với ba đường cao AD, BE, CF cắt nhau tại
điểm H. Tiếp tuyến tại B, C của đường tròn (O) ngoại tiếp tam giác ABC cắt nhau tại
điểm T, c{c đường thẳng TD và EF cắt nhau tại điểm S. Gọi X, Y lần lượt l| giao điểm
của đường thẳng EF với c{c đường thẳng TB, TC; M l| trung điểm của cạnh BC.

11
Page: Tài Liệu Môn Toán
Website: tailieumontoan.com

1. Chứng minh rằng H, M lần lượt là tâm đường tròn nội tiếp các tam giác DEF và XTY.
2. Chứng minh rằng đường thẳng SH đi qua trung điểm của đoạn thẳng BC.
Câu V (1,0 điểm) Kí hiệu chỉ tập hợp các số tự nhiên. Giả sử f :  là hàm số
 
thỏa mãn các điều kiện f 1  0 và f m2  2n2   f  m    2  f  n   với mọi m, n 
2 2
.
Tính các giá trị của f  2  và f  2011 .
ĐỀ 9
PAGE TÀI LIỆU TOÁN HỌC ĐỀ CHỌN HỌC SINH GIỎI TOÁN LỚP 10
MÔN TOÁN
(Thời gian: 180 phút, không tính thời gian giao đề)

Câu 1 (2 điểm) a) Cho parabol (P): y   x  4 x  5 v| điểm I (1;4) . Tìm trên (P) hai
2

điểm M, N đối xứng nhau qua điểm I.

b) Tìm các giá trị của m để phương trình x  2  m  m có 4 nghiệm phân biệt.
2 4 2

Câu 2 (3 điểm) a) Giải bất phương trình: ( x  1) x  2  ( x  6) x  7  x  7 x  12


2


(x  1)(y  6)  y(x  1)
2 2

a) Giải hệ phương trình: 


(y  1)(x  6)  x(y  1)

2 2

b) Tìm m để phương trình 3 x  1  m x  1  2 4 x 2  1 có nghiệm.


Câu 3 (3 điểm) a) Cho tam giác ABC có trọng tâm là G. Hai điểm D và E được x{c định
2
bởi các hệ thức: AD  2 AB; AE  AC . Chứng minh rằng: D, E, G thẳng hàng
5
b) Gọi H là trực tâm  ABC, M l| trung điểm của BC. Chứng minh rằng
1
MH .MA  BC 2
4
c) Trong mặt phẳng tọa độ Oxy, cho hình bình hành ABCD, điểm M (2;0) là
7 
trung điểm của cạnh AB, điểm H (1; 1) là hình chiếu của B trên AD v| điểm G  ;3 
3 
là trọng tâm tam giác BCD. Đường thẳng HM cắt BC tại E, đường thẳng HG cắt BC tại F.
Tìm tọa độ c{c điểm E, F và B

Câu 4 (1 điểm) Cho x, y là các số thực thỏa mãn x  y  1 . Tìm giá trị lớn nhất và giá
2 2

( x  y)2  3 y 2
trị nhỏ nhất của biểu thức S  .
xy  1

12
Page: Tài Liệu Môn Toán
Website: tailieumontoan.com

Câu 5 (1 điểm) Cho x, y là các số thực thay đổi. Tìm giá trị nhỏ nhất của biểu thức
A  ( x  1)2  y 2  ( x  1)2  y 2  y  2
ĐỀ 10
PAGE TÀI LIỆU TOÁN HỌC ĐỀ CHỌN HỌC SINH GIỎI TOÁN LỚP 10
MÔN TOÁN
(Thời gian: 180 phút, không tính thời gian giao đề)
5 10
Câu 1 (2 điểm). Giải bất phương trình  .
x2 x 1

Câu 2 (2 điểm). Giải phương trình  x  4  x  1  3 x  5 x  2  6 .


2


2 x  4 xy  2 y  3 x  3y  2  0
2 2

Câu 3 (2 điểm). Giải hệ phương trình  2 .


 x  y  2 y(2 x  1)  0

2

Câu 4 (2 điểm). Tìm tất cả các giá trị của m để phương trình sau có nghiệm thực x:
x 2  2 x  m2  2 x  4 .
Câu 5 (2 điểm). Tìm tất cả các giá trị của m để tập nghiệm của bất phương trình
x 2  mx  m  1  0 có biểu diễn trên trục số là một đoạn có độ dài bằng 1.
Câu 6 (2 điểm). Giả sử tam giác ABC có diện tích là S; a, b, c lần lượt l| độ dài các cạnh
BC, CA, AB. Chứng minh rằng 4S(cot A  cot B  cot C)  a2  b2  c2 .

Câu 7 (6 điểm). Trong mặt phẳng tọa độ Oxy cho đường thẳng d1: x+y-6=0 v| đường
thẳng d2: x+2y-5=0.
1-Gọi  là góc giữa đường thẳng d1 v| đường thẳng d2.

3cos   sin   10
Tính giá trị của biểu thức m  .
2 cos   sin 
2-Viết phương trình của đường tròn (C) có t}m l| điểm I thuộc đường thẳng d1, I
có ho|nh độ bằng 2 v| đường tròn (C) cắt đường thẳng d2 tạo thành một dây cung có
độ dài bằng 2.
3-Biết tam giác ABC cân tại A, cạnh AB và cạnh BC lần lượt nằm trên c{c đường
thẳng d1 và d2. Viết phương trình của đường thẳng chứa đường cao kẻ từ B của tam
giác ABC.

Câu 8 (2 điểm). Cho a, b, c là ba số thực không âm thỏa mãn a  b  c  3 . Tìm giá trị
2 2 2

lớn nhất của biểu thức S  a2  7b  3 b2  7c  3 c 2  7a .


3

13
Page: Tài Liệu Môn Toán
Website: tailieumontoan.com

ĐỀ 11
PAGE TÀI LIỆU TOÁN HỌC ĐỀ CHỌN HỌC SINH GIỎI TOÁN LỚP 10
MÔN TOÁN
(Thời gian: 180 phút, không tính thời gian giao đề)
Câu 1 (2 điểm). Tìm các giá trị của tham số m để phương trình sau có nghiệm:

x2  m
0
 x2  4 x  5
Câu 2 (4 điểm). Giải phương trình, bất phương trình sau:

a, 8 ( x  2)( x  32)  x( x  30)  73 b, x. 3  2 x 1  0

Câu 3 (4 điểm). Giải các hệ phương trình sau:


x  x y  2 y
3 2

2 x  5 x  2 y  3 y  7 x  y
a,  b, 
x y  y  y
  xy  6

2 3

Câu 4 (2điểm). Tìm các giá trị nguyên của tham số m để hệ phương trình sau có nghiệm
mx  y  3m
duy nhất ( x; y ) sao cho x, y là các số nguyên 
2mx  y  m  3
1 1 1
Câu 5 (2điểm). Cho x, y, z > 1 và thoả mãn điều kiện   2
x y z
Tìm giá trị lớn nhất của biểu thức A = (x - 1)(y - 1)(z - 1).
Câu 6 (2điểm). Trong hệ tọa độ xOy , cho hình thang vuông ABCD vuông tại A và B,
đ{y lớn AD. Biết chu vi hình thang là 16  4 2 , diện tích hình thang là 24. Biết
A(1;2), B(1;6) . Tìm tọa độ c{c đỉnh C và D biết ho|nh độ điểm D lớn hơn 2.
C}u 7 (4điểm). Trong hệ toạ độ xOy cho đường tròn (C) có phương
trình x 2  y 2  2 x  4 y  3  0 , v| đường thẳng d có phương trình : x + y = 0

a, Viết phương trình đường thẳng  song song với đường thẳng d và cắt (C) theo dây
cung có độ dài bằng 1
b, Viết phương trình tiếp tuyến của (C) biết khoảng cách từ A(0; 2) đến tiếp tuyến là
lớn nhất
c, Viết phương trình đường thẳng qua gốc tọa độ O tạo với đường thẳng d góc 600

14
Page: Tài Liệu Môn Toán
Website: tailieumontoan.com

ĐỀ 12
PAGE TÀI LIỆU TOÁN HỌC ĐỀ CHỌN HỌC SINH GIỎI TOÁN LỚP 10

MÔN TOÁN
(Thời gian: 180 phút, không tính thời gian giao đề)

Câu I (2,0 điểm). Cho parabol (P): y  x  2 x  1. Tìm tất cá các giá trị của m để đường
2

thẳng d: y   x  m cắt parabol (P) tại hai điểm phân biệt A, B sao cho tam giác OAB
vuông tại O (với O là gốc tọa độ)..
Câu II (4,0 điểm). 1. Tìm tất cả các giá trị của tham số m để phương trình sau có bốn
nghiệm phân biệt :
 m  2 x4  2  m  1 x2  2m  1  0

2. Cho 3sin
4
  8cos 2   5, 0    . Tính P  sin
4
  cos3  .
2
Câu III ( 6,0 điểm). 1. Giải phương trình: x  2 x  x  1  x  3  x4  x2  1
2 2


 x  5x  y  9
2

2. Giải hệ phương trình:  3


3x  6 x  x y  2 xy  18

2 2

3. Giải bất phương trình: 3  3x 2  2 x  2 3x 2  2 x .


Câu IV ( 2,0 điểm). Cho tam giác ABC có BAC  60 , AB  5, AC  10 , trung tuyến
0

AD ( D  BC ) và M là một điểm thỏa mãn 3MA  2MC  0 . Tính độ d|i đoạn BM và


chứng minh AD  BM .

x2 y 2
Câu V( 4,0 điểm) 1. Trong mặt phẳng Oxy, cho elip (E):   1 có hai tiêu điểm
25 9
F1 , F2 . Tìm tọa độ điểm M thuộc elip (E) sao cho b{n kính đường tròn nội tiếp tam giác
4
MF1F2 bằng
3
2. Trong mặt phẳng Oxy, cho đường tròn  C  :  x  2    y  1  8 v| đường
2 2

thẳng d : x  2 y  3  0 .

a) Tìm điểm M nằm trên d sao cho từ M kẻ được hai tiếp tuyến tới đường tròn
(C) và hai tiếp tuyến đó vuông góc với nhau.

15
Page: Tài Liệu Môn Toán
Website: tailieumontoan.com

b) Cho hình thoi ABCD có tất cả các cạnh đều tiếp xúc với đường tròn (C), biết A
thuộc đường thẳng d v| ho|nh độ của A không nhỏ hơn 1, BD = 2 AC. Tìm tọa độ A.

Câu VI( 2,0 điểm). Cho x, y, z là ba số thực dương thỏa mãn x  y  z  3 . Tìm giá
2 2 2

1 1 1
trị nhỏ nhất của biểu thức A =   .
xy  2 yz  2 zx  2
ĐỀ 13
PAGE TÀI LIỆU TOÁN HỌC ĐỀ CHỌN HỌC SINH GIỎI TOÁN LỚP 10

MÔN TOÁN
(Thời gian: 180 phút, không tính thời gian giao đề)
Câu I (2,0 điểm).
Cho parabol (P): y  x 2  2 x  m . Tìm tất cá các giá trị của m để đường thẳng
d: y  2 x  1 cắt parabol (P) tại hai điểm phân biệt A, B sao cho AB=2.

Câu II (4,0 điểm).


1. Tìm tất cả các giá trị của tham số m để phương trình sau có ba nghiệm phân
biệt:

 m  2 x4  2mx2  m2  4  0
2sin  a  b 
2. Chứng minh rằng  tan b  t ana .
cos  a  b   cos  a  b 

1  1  4 x2
Câu III ( 6,0 điểm). 1. Giải bất phương trình:  3.
x
2. Giải phương trình: 2 x 2  2 x  5   4 x  1 x 2  3 .


 x  3x  6 x  4  y  3 y
3 2 3

3. Giải hệ phương trình:   x, y  .



 2 4  x 2
 3 3  2 y  y 2
 3 x  2

Câu IV ( 1,0 điểm). Chứng minh rằng trong mọi tam giác ABC ta có
AB 2  AC 2  BC 2
AB. AC  .
2
Câu V( 5,0 điểm)

16
Page: Tài Liệu Môn Toán
Website: tailieumontoan.com

1. Trong mặt phẳng toạ độ Oxy, cho hình thoi ABCD có 4 đỉnh trùng với c{c đỉnh
của một elip , b{n kính đường tròn nội tiếp hình thoi bằng 2 . Viết phương trình chính
1
tắc của elip biết tỉ số giữa tiêu cự v| độ dài trục lớn của elip bằng .
2
2. Trong mặt phẳng với hệ tọa độ Oxy cho hai đường thẳng
d1 : 2 x  y  1  0, d2 : 2 x  y  3  0 cắt nhau tai I; điểm A thuộc d1 , A có ho|nh độ
dương kh{c 1. Lập phương trình đường thẳng    đi qua A, cắt d 2 tại B sao cho diện
tích IAB bằng 6 và IB  3IA.

 1 13 
3. Trong mặt phẳng với hệ tọa độ Oxy cho điểm J  ;  , đường thẳng
2 2 
d : x  y  1  0 v| đường tròn C  : x2  y 2  4x  2 y  4  0 . Gọi M l| điểm thuộc
đường thẳng d và nằm ngo|i đường tròn (C). Từ M kẻ các tiếp tuyến MA, MB đến
đường tròn (C) (A, B là các tiếp điểm). Gọi (J) l| đường tròn tâm J và tiếp xúc với đường
thẳng AB. Tìm tọa độ điểm M sao cho đường tròn (J) có chu vi lớn nhất.
Câu VI( 2,0 điểm). Cho ba số thực dương a, b, c thỏa mãn ab  bc  ca  3 . Tìm giá trị
nhỏ nhất của biểu thức P  a6  b6  1  c6  b6  1  a6  c6  1 .
ĐỀ 14
PAGE TÀI LIỆU TOÁN HỌC ĐỀ CHỌN HỌC SINH GIỎI TOÁN LỚP 10
MÔN TOÁN
(Thời gian: 180 phút, không tính thời gian giao đề)
 x 3  12 x  7 1
Câu 1(2 điểm). Giải bất phương trình 
x2  x  2 2
Câu 2(2 điểm). Tìm các giá trị của tham số m để phương trình:
m  1x  2x  m  1  0
2

có 2 nghiệm phân biệt x1 , x2 thỏa mãn: 2 x1  3x1 x2  2 x2  3x1 x2  2


3 2 3 2

( x  1)( y  1)( x  y  2)  6
Câu 3(2 điểm). Giải hệ phương trình:  2
x  y  2x  2 y  3  0
2

Câu 4 (6 điểm). Trong hệ tọa độ Oxy cho điểm A(1; -2), điểm B(3; -1) v| đường thẳng d
có phương trình: 2x - y + 4 = 0.
a, Viết phương trình đường tròn tâm A tiếp xúc với đường thẳng d.
b, Tính chu vi và diện tích tam giác ABO.
c, Viết phương trình đường thẳng  đi qua B v| tạo với đường thẳng d một góc 450.

17
Page: Tài Liệu Môn Toán
Website: tailieumontoan.com

d, Tìm tọa độ điểm M thuộc đường tròn (C): ( x  2)  ( y  1)  1 sao cho


2 2

MA2  MB2 đạt giá trị nhỏ nhất.


1
Câu 5(2 điểm). Cho sin   cos  . Tính giá trị của biểu thức:
3
F  tan   cot  tan 2   cot2 
Câu 6 (2 điểm). Giải bất phương trình: x  5 x  4 1 
2
 x3  2x 2  4x 
 2 x 2  4 y 2 4. (2 x  3 y )( x  y )
  1
Câu 7 (2 điểm). Giải hệ phương trình sau:  xy xy

 2( x  y  3)  x  y  3

Câu 8 (2 điểm). Cho ba số dương a, b, c thỏa mãn a + b + c  1 . Tìm giá trị nhỏ nhất của
1 1 1 1
biểu thức: P    
a b c
2 2 2
ab(a  b) bc(b  c) ca (c  a)

ĐỀ 15
PAGE TÀI LIỆU TOÁN HỌC ĐỀ CHỌN HỌC SINH GIỎI TOÁN LỚP 10
MÔN TOÁN
(Thời gian: 180 phút, không tính thời gian giao đề)
x x
Câu I (1,5 điểm) 1) X{c định tính chẵn - lẻ của hàm số y   
10  x 10  x
2) Cho các nửa khoảng A  (a; a  1], B  [b; b  2). Đặt C  A  B. Với điều
kiện nào của các số thực a và b thì C là một đoạn? Tính độ dài của đoạn C khi đó.

Câu II (2,0 điểm) 1) Tìm m để phương trình x  1  m  m  1 có bốn nghiệm phân


2 4 2

biệt.

2) Giải và biện luận (theo tham số m) bất phương trình:


 m  1 x  2  m  1 .
x2
Câu III (2,5 điểm) 1) Giải phương trình x2  7 x  8  2 x.
 7 x  y  2 x  y  5
2) Giải hệ phương trình 
 x  y  2 x  y  1.

18
Page: Tài Liệu Môn Toán
Website: tailieumontoan.com

Câu IV (3,0 điểm) 1) Cho tam giác ABC có AB = c, AC = b và BAC  60 . C{c điểm M, N
0

được x{c định bởi MC  2MB và NB  2 NA . Tìm hệ thức liên hệ giữa b và c để


AM và CN vuông góc với nhau.
2) Cho tam giác ABC. Trên các cạnh BC, CA và AB của tam gi{c đó, lần lượt lấy
c{c điểm A ', B ' và C '. Gọi S a , Sb , Sc và S tương ứng là diện tích của các tam giác
AB ' C ', BC ' A ', CA ' B ' và ABC. Chứng minh bất đẳng thức
3
Sa  Sb  Sc  S . Dấu đẳng thức xảy ra khi và chỉ khi nào?
2
Câu V (1,0 điểm) Trong mặt phẳng tọa độ Oxy, cho đường tròn tâm O bán kính R (R > 0,
R không đổi). Gọi A và B lần lượt l| c{c điểm di động trên trục hoành và trục tung sao
cho đường thẳng AB luôn tiếp xúc với đường tròn đó. Hãy x{c định tọa độ của c{c điểm
A, B để tam giác OAB có diện tích nhỏ nhất.
ĐỀ 16
PAGE TÀI LIỆU TOÁN HỌC ĐỀ CHỌN HỌC SINH GIỎI TOÁN LỚP 10
MÔN TOÁN
(Thời gian: 180 phút, không tính thời gian giao đề)
Câu I ( 4 điểm) Cho parabol ( P) : y  ax  bx  1
2

 3 11 
1) Tìm các giá trị của a; b để parabol có đỉnh S  ; .
 2 2 
2) Với giá trị của a; b tìm được ở câu 1, tìm giá trị của k để đường thẳng
 : y  x(k  6)  1 cắt parabol tại hai điểm phân biệt M ; N sao cho trung điểm
của đoạn thẳng MN nằm trên đường thẳng d : 4 x  2 y  3  0 .

Câu II ( 2 điểm) Cho tam gi{c đều ABC v| c{c điểm M , N , P thỏa mãn BM  k BC ,
2 4
CN  CA , AP  AB . Tìm k để AM vuông góc với PN .
3 15
Câu III(9 điểm)
3m +1
1) Tìm m để phương trình x + 6 x - 9 + m x + 2 x - 9 -8 = x +
2
có hai nghiệm x1 , x 2 sao cho x1  10  x2

2) Giải phương trình x  3  x . 4  x  4  x. 5  x  5  x. 3  x

19
Page: Tài Liệu Môn Toán
Website: tailieumontoan.com

 x2  y 2  2 y  6  2 2 y  3  0
3) Giải hệ phương trình 
 .
( x  y )( x  xy  y  3)  3( x  y )  2

2 2 2 2

Câu IV( 3 điểm) Cho hình vuông ABCD cạnh có độ dài là a. Gọi E; F l| c{c điểm
1 1
x{c định bởi BE  BC , CF   CD, đường thẳng BF cắt đường thẳng AE tại
3 2
điểm I .

1) Tính giá trị của EA.CE theo a.


2) Chứng minh rằng AIC  90 .
0

Câu V ( 2 điểm) Cho các số dương a, b, c có a+b+c=3. Tìm giá trị nhỏ nhất của biểu thức
a a b b c c
P   .
2c  a  b 2a  b  c 2b  c  a
ĐỀ 17
PAGE TÀI LIỆU TOÁN HỌC ĐỀ CHỌN HỌC SINH GIỎI TOÁN LỚP 10
MÔN TOÁN
(Thời gian: 180 phút, không tính thời gian giao đề)
Bài I ( 5,0 điểm)
1. Giải phương trình x2  2 x  3  x  3 .
 x y  x y  2

2. Giải hệ phương trình 

 x y x y 4
2 2

Bài II ( 5,0 điểm)


x 1
1. Tìm tham số m để bất phương trình  1 có tập nghiêm là .
mx  4 x  m  3
2


 x  y  2 x  2 y  2m
2 2

2. Tìm tham số m để hệ phương trình  có đúng hai nghiệm


 
2

 x  y  2  4
phân biệt.
1
Bài III ( 2,0 điểm) Tam thức f ( x)  x 2  bx  c thỏa mãn f ( x)  với x  1;1 . Hãy
2
tìm các hệ số b và c .

20
Page: Tài Liệu Môn Toán
Website: tailieumontoan.com

Bài IV (2,0 điểm) Cho x, y, z là ba số thực dương thỏa mãn xy  yz  zx  1 . Chứng minh
x y z 3
rằng ta luôn có:    .
x 1
2
y 1
2
z 1 2
2

Bài V ( 6,0 điểm)


1
1. Cho tam giác ABC trọng tâm G. C{c điểm M, N được x{c định bởi CN  BC ;
2
3MA  4MB  0
a/ Chứng minh rằng ba điểm G, M, N thẳng hàng .
b/ Đường thẳng MN chia tam giác CAN thành hai tam giác. Tính tí số diện tích của
hai tam gi{c đó.

2. Tam giác ABC có c{c đường phân giác trong AE, BF và CP. Chứng minh rằng ta
luôn có:
SEFP 2abc
 ( với BC  a; AC  b; AB  c ) .
SABC (a  b)(b  c)(c  a)

ĐỀ 18
PAGE TÀI LIỆU TOÁN HỌC ĐỀ CHỌN HỌC SINH GIỎI TOÁN LỚP 10
MÔN TOÁN
(Thời gian: 180 phút, không tính thời gian giao đề)
Bài 1 (4 điểm) :

1. Giải phương trình: 3 3 4x - 3 - 4 6 - 2x + 5 = 0


2. Cho hai số x, y thoả mãn 4x2 + y2 = 4
Tìm giá trị lớn nhất, giá trị nhỏ nhất của M = x2 -3xy +2y2

Bài 2 (4 điểm) :


x + y + x + y = 8
2 2

1.Giải hệ phương trình:  2



2
(x + x)(y + y) = 12.
2
2. Giải phương trình : 2010x - 4x + 3 = 2009x 4x - 3

Bài 3 (4,00 điểm) :


21
Page: Tài Liệu Môn Toán
Website: tailieumontoan.com

Trong mặt phẳng với hệ toạ độ Oxy. Viết phương trình đường thẳng (d) qua
1 1
M( 5; -2) cắt Ox, Oy lần lượt tại A, B sao cho: + đạt giá trị nhỏ nhất.
OA 2 2OB2
Bài 4 (4,00 điểm) :
Trong mặt phẳng với hệ toạ độ Oxy. Cho hai điểm A( 1; 1), B(4 ; -3). Tìm điểm C
thuộc đường thẳng (d) : x – 2y – 1 = 0 sao cho khoảng cách từ C đến đường thẳng AB
bằng 6.
Bài 5 (4,00 điểm) :
Cho hai số dương x, y thoả mãn x + y = 2010. Hãy tìm giá trị nhỏ nhất của biểu thức:
x y
P= +
2010 - x 2010 - y
ĐỀ 19
PAGE TÀI LIỆU TOÁN HỌC ĐỀ CHỌN HỌC SINH GIỎI TOÁN LỚP 10

MÔN TOÁN
(Thời gian: 180 phút, không tính thời gian giao đề)
Câu 1.
a) Giải bất phương trình: x2  6 x  2  2(2  x) 2 x  1.


 x  xy  y  y
5 4 10 6

b) Giải hệ phương trình: 


 4x  5  y  8  6

2

Câu 2. Tìm tất cả các giá trị của tham số m để hệ phương trình sau có nghiệm

 x  m  y ( x  my )
2

 2
 x  y  xy

Câu 3. Trong mặt phẳng với hệ tọa độ Oxy, cho điểm I (2; 4) v| c{c đường thẳng
d1 : 2 x  y  2  0, d2 : 2 x  y  2  0 . Viết phương trình đường tròn (C ) có tâm I sao cho
(C ) cắt d1 tại A, B và cắt d 2 tại C , D thỏa mãn AB2  CD2  16  5 AB.CD.

Câu 4.
1. Cho tam giác ABC có AB= c ,BC=a ,CA=b .Trung tuyến CM vuông góc với phân
CM 3
giác trong AL và  52 5 .
AL 2
22
Page: Tài Liệu Môn Toán
Website: tailieumontoan.com

b
Tính và cos A .
c
9
2. Cho a,b  thỏa mãn: (2  a)(1  b) 
2

Tìm giá trị nhỏ nhất của biểu thức: P  16  a 4  4 1  b4


Câu 5. Cho f  x   x 2  ax  b với a,b  thỏa mãn điều kiện: Tồn tại các số nguyên
m, n, p đôi một phân biệt và 1  m, n, p  9 sao cho: f  m  f  n   f  p   7 .

Tìm tất cả các bộ số (a;b).


ĐỀ 20
PAGE TÀI LIỆU TOÁN HỌC ĐỀ CHỌN HỌC SINH GIỎI TOÁN LỚP 10

MÔN TOÁN
(Thời gian: 180 phút, không tính thời gian giao đề)
Câu 1 (2 điểm)

i. Cho hàm số y  x  2mx  3m và hàm số y  2 x  3 . Tìm m để đồ thị các


2

hàm số đó cắt nhau tại hai điểm phân biệt v| ho|nh độ của chúng đều dương.
ii. Giải bất phương trình:  x 2  8x  12  10  2 x
Câu 2 (2 điểm)
3
c) Giải phương trình: (4 x  x  3)  x 
3 3 3

2
d) Giải phương trình: 2 x  11x  23  4 x  1
2

Câu 3 (2 điểm)
a) Trong mặt phẳng tọa độ Oxy cho điểm M (1;4) . Đường thẳng d qua M, d cắt
trục hoành tại A(ho|nh độ của A dương), d cắt trục tung tại B(tung độ của B
dương). Tìm gi{ trị nhỏ nhất của diện tích tam giác OAB.
b) Trong mặt phẳng tọa độ Oxy cho đường tròn (C): ( x  2)  ( y  3)  9 và
2 2

điểm A(1; 2) . Đường thẳng  qua A,  cắt (C) tại M và N. Tìm giá trị nhỏ
nhất của độ d|i đoạn thẳng MN.
Câu 4 (3 điểm)

23
Page: Tài Liệu Môn Toán
Website: tailieumontoan.com

a) Chứng minh rằng tứ giác lồi ABCD là hình bình hành khi và chỉ khi
AB2  BC 2  CD2  DA2  AC 2  BD2 .
1 1 1
b) Tìm tất cả các tam giác ABC thỏa mãn: 2
 2  2 (trong đó AB=c; AC=b;
ha b c
đường cao qua A là ha ).

Câu 5 (1 điểm)
Cho a, b, c là các số thực dương . Chứng minh rằng:

 a  b  b  c    c  a 
2 2 2
2a 2b 2c
   3
bc ca ab a  b  c
2

ĐỀ 21
PAGE TÀI LIỆU TOÁN HỌC ĐỀ CHỌN HỌC SINH GIỎI TOÁN LỚP 10

MÔN TOÁN
(Thời gian: 180 phút, không tính thời gian giao đề)
Câu 1 (2,5 điểm)
2
a) Cho hàm số y  x  3x  2 và hàm số y   x  m . Tìm m để đồ thị các hàm
số đó cắt nhau tại hai điểm phân biệt A, B đồng thời khoảng cách từ trung điểm I của
đoạn thẳng AB đến các trục tọa độ bằng nhau.
1 1
b) Giải bất phương trình:  0
2
x  4x  3 2x  4

Câu 2 (2,5 điểm)


a) Trong mặt phẳng tọa độ Oxy cho tam giác ABC có B(1;2) . Đường thẳng  là
đường phân giác trong của góc A có phương trình 2x  y  1  0 ; Khoảng cách từ C đến
 gấp 3 lần khoảng cách từ B đến  . Tìm tọa độ của A và C biết C nằm trên trục tung.
b) Cho tam giác ABC vuông ở A, gọi  là góc giữa hai đường trung tuyến BM và
3
CN của tam giác. Chứng minh rằng sin  
5
Câu 3 (2,5 điểm)

24
Page: Tài Liệu Môn Toán
Website: tailieumontoan.com

2
a) Cho tam giác ABC. Gọi D, E lần lượt l| c{c điểm thỏa mãn: BD  BC;
3
1
AE  AC . Tìm vị trí của điểm K trên AD sao cho 3 điểm B, K, E thẳng hàng.
4
b) Cho tam giác ABC vuông ở A; BC = a; CA = b; AB = c. X{c định điểm I thỏa
mãn hệ b2 IB  c2 IC  2a 2 IA  0 ; Tìm điểm M sao cho biểu thức
thức:
( b MB  c MC  2a MA ) đạt giá trị lớn nhất.
2 2 2 2 2 2

Câu 4 (2,5 điểm)

a) Giải phương trình: 1   6 x  2  2 x  1  2 5 x  4 x


2
 2

b) Cho x, y, z là các số thực dương thỏa mãn x  y  z  xyz . Chứng minh rằng:

1  1  x2 1  1  y 2 1  1  z 2
   xyz .
x y z
ĐỀ 22
PAGE TÀI LIỆU TOÁN HỌC ĐỀ CHỌN HỌC SINH GIỎI TOÁN LỚP 10

MÔN TOÁN
(Thời gian: 180 phút, không tính thời gian giao đề)
Câu I (2,0 điểm):

Cho parabol (P): y  x – 2 x  4 v| c{c đường thẳng (dm): y  3x  2m  1


2

(m là tham số)
1) Biện luận số giao điểm của (P) và (dm) theo tham số m.
2) Khi (dm) cắt (P) tại hai điểm A, B (A và B có thể trùng nhau), tìm tập hợp trung
điểm I của AB khi m thay đổi.
Câu II (3,0 điểm):

1) Giải bất phương trình:  2x  5  x2  x  25  x2  5x  6  0

2. 3 x  3 y  3 5(8 x  y )
2) Giải hệ phương trình:  2
 x  y  2 x  4 y  31  0
2

Câu III (3,0 điểm):

25
Page: Tài Liệu Môn Toán
Website: tailieumontoan.com

1) Trong mặt phẳng tọa độ Oxy, cho tam giác ABC cân tại A có phương trình
đường thẳng chứa cạnh AB là: x  2 y  2  0 , phương trình đường thẳng chứa cạnh
AC là: 2 x  y  1  0 . Điểm M(1;2) thuộc đoạn thẳng BC. Tìm tọa độ điểm D sao cho
DB.DC có giá trị nhỏ nhất.
2) Cho tứ giác ABCD; hai điểm M, N thay đổi sao cho AM  k AB; DN  k DC
(0  k  1) . Gọi I l| điểm thỏa mãn 3IM   2IN . Tìm tập hợp c{c điểm I khi M, N
thay đổi.
Câu IV (2,0 điểm):

1) Tam giác ABC có S  b  (a  c) với S là diện tích tam giác; a = BC; b = AC;
2 2

c = AB. Tính tan B .

2) Cho a, b, c là các số thực dương thay đổi thỏa mãn: a  b  c  3


2 2 2

Tìm giá trị lớn nhất của biểu thức:


ab bc ca
M  
5a 2  10ab  10b2 5b2  10bc  10c 2 5c 2  10ca  10a 2
ĐỀ 23
PAGE TÀI LIỆU TOÁN HỌC ĐỀ CHỌN HỌC SINH GIỎI TOÁN LỚP 10

MÔN TOÁN
(Thời gian: 180 phút, không tính thời gian giao đề)
Câu I(2,0 điểm)
Cho parabol (P): y   x v| đường thẳng (d) đi qua điểm I (0; 1) và có hệ số
2

góc là k . Gọi A và B l| c{c giao điểm của (P) và (d). Giả sử A, B lần lượt có ho|nh độ là
x1; x2 .
1) Tìm k để trung điểm của đoạn thẳng AB nằm trên trục tung.
2) Chứng minh rằng x1  x2  2  k  R 
3 3

Câu II(3,0 điểm)


1) Giải phương trình: 3x  1  5 x  4  3x 2  x  3

 x  x y  xy  xy  y  1
2 3 2

2) Giải hệ phương trình: 


 x  y  xy (2 x  1)  1

4 2

26
Page: Tài Liệu Môn Toán
Website: tailieumontoan.com

Câu III(4 điểm)


1) Trong mặt phẳng tọa độ Oxy cho tam giác ABC có đỉnh A(2;6) , ch}n đường
 3
phân giác trong kẻ từ đỉnh A l| điểm D  2;   , t}m đường tròn ngoại tiếp tam giác
 2
 1 
ABC l| điểm I   ;1 . Viết phương trình của đường thẳng BC.
 2 
2) Cho tam giác ABC có BC  a; CA  b; BA  c (b ≠ c) và diện tích là S . Kí hiệu
ma ; mb ; mc lần lượt l| độ dài của c{c đường trung tuyến kẻ từ c{c đỉnh A, B, C. Biết
rằng 2ma  mb  mc .
2 2 2

2
a) Chứng minh rằng a 4S.cotA
b) Gọi O và G lần lượt l| t}m đường tròn ngoại tiếp và trọng tâm tam giác ABC;
M l| trung điểm của BC. Chứng minh rằng góc MGO không nhọn.
Câu IV (1 điểm). Cho a; b; c là các số thực dương thay đổi và thỏa mãn
3 3
abc . Tìm giá trị lớn nhất của biểu thức:
2
1 1 1
M 2  2  2 .
a  b  3 b  c  3 c  a2  3
2 2

ĐỀ 24
PAGE TÀI LIỆU TOÁN HỌC ĐỀ CHỌN HỌC SINH GIỎI TOÁN LỚP 10
MÔN TOÁN
(Thời gian: 180 phút, không tính thời gian giao đề)
Bài 1. ( 4,0 điểm) Giải phương trình x2  2 2 x  7  2 3  2 x  5( x  ) .

Bài 2. ( 4,0 điểm) Cho tam giác ABC ( AB  AC ) nhọn, không cân nội tiếp đường tròn
 O  . C{c đường cao AD, BE và CF cắt nhau tại H . Gọi M l| trung điểm cạnh BC.
Đường tròn  J  ngoại tiếp tam giác AEF cắt đường tròn  O  tại điểm thứ hai là K
( K  A). Đường thẳng AM cắt đường tròn  J  tại điểm thứ hai là Q (Q  A). EF cắt
AD tại P. Đoạn PM cắt đường tròn  J  tại N .

a) Chứng minh c{c đường thẳng KF , EQ , BC đồng quy hoặc song song v| ba điểm K,
P, Q thẳng hàng.
b) Chứng minh rằng đường tròn ngoại tiếp tam giác DMN v| đường tròn ngoại tiếp
tam giác BNC tiếp xúc nhau.

27
Page: Tài Liệu Môn Toán
Website: tailieumontoan.com

Bài 3. ( 4,0 điểm) Tìm tất cả các bộ ba số nguyên (a, b, c) sao cho số
(a  b)(b  c)(c  a)
 2 là một lũy thừa của 20162017 (Một lũy thừa của 20162017 là một số
2
có dạng 20162017 n với n là một số nguyên không âm).
Bài 4. ( 4,0 điểm) Cho a, b, c là ba số thực dương thỏa mãn
bc ca ab  1 1 1 
   2     . Chứng minh rằng: a 2  b2  c2  3  2(ab  bc  ca) .
a b c  ab bc ca 
Bài 5. ( 4,0 điểm) Cho một bảng ô vuông kích thước 10 10 , trên đó đã điền các số
nguyên dương từ 1 đến 100 vào các ô vuông con theo trình tự như hình a. Ở mỗi bước
biến đổi, người ta chọn tùy ý ba ô vuông con liên tiếp theo hàng hoặc theo cột hoặc theo
một đường chéo của hình vuông kích thước 3  3 (xem hình b) rồi thực hiện: Hoặc là
giảm số ở ô nằm giữa đi 2 đơn vị đồng thời tăng số ở hai ô liền kề lên 1 đơn vị, hoặc là
tăng số ở ô nằm giữa lên 2 đơn vị đồng thời giảm số ở hai ô liền kề đi 1 đơn vị. Giả sử
rằng sau hữu hạn bước biến đổi, tập hợp tất cả các số ghi trên bảng ô vuông vẫn là tập
{1; 2; 3; <; 100}. Chứng minh rằng khi đó c{c số ghi trên bảng theo đúng vị trí như
trước khi biến đổi.

1 2 3 < 9 10

11 12 13 < 19 20

21 22 23 < 29 30

< < < < < <

91 92 93 < 99 100
Hình b- Ba ô vuông con liên tiếp
Hình a – Bảng ô vuông ban đầu

ĐỀ 25
PAGE TÀI LIỆU TOÁN HỌC ĐỀ CHỌN HỌC SINH GIỎI TOÁN LỚP 10

MÔN TOÁN
(Thời gian: 180 phút, không tính thời gian giao đề)
Câu 1 (4 điểm)

28
Page: Tài Liệu Môn Toán
Website: tailieumontoan.com

7 x  y  3xy  x  y   12 x  6 x  1
 3 3 2

Giải hệ phương trình 


2 x  3  9  y  1

2 2

Câu 2 (4 điểm)
Cho đường tròn (O) v| d}y AB. C{c đường tròn (O1) và (O2) nằm về một phía đối
với đường thẳng AB, tiếp xúc với nhau ại T đồng thời tiếp xúc với AB v| tiếp xúc trong
với đường tròn (O). Tiếp tuyến chung tại T của c{c đường trong (O1) và (O2) cắt đường
tròn (O) tại C (với C thuộc nửa mặt phẳng với bờ l| đường thẳng AB có chứa hai đường
tròn (O1) và (O2)). Chứng minh rằng T l| t}m đường tròn nội tiếp tam gi{c ABC.
Câu 3 (4 điểm)
Cho m v| n l| c{c số nguyên dương thỏa mãn 2016m +1 l| ước của 2016n +1.
Chứng minh rằng m l| ước của n
Câu 4. (4 điểm)
Cho ba số dương a, b, c thay đổi và thỏa mãn a + b + c = abc.
2
b c a 1 1 1
Chứng minh rằng: 3  2  2  2       3
a b c a b c
Câu 5. (4 điểm)
Cho tập hợp X có 2016 phần tử. Chọn ra 64 tập con X1, X2,....X64 của tập X (mỗi tập
con đều chứa nhiều hơn 1008 phần tử). Chứng minh tồn tại tập con A của X có số
phần tử không vượt qu{ 6 m| A ∩ X ≠ ∅, với i = 1, 64

ĐỀ 26
PAGE TÀI LIỆU TOÁN HỌC ĐỀ CHỌN HỌC SINH GIỎI TOÁN LỚP 10

MÔN TOÁN
(Thời gian: 180 phút, không tính thời gian giao đề)

Câu 1 (6,0 điểm)


1. Giải phương trình sau trên : 4 x 2  12 x x  1  27  x  1
9
2. Giải bất phương trình sau:  x2
x 5 3

Câu 2 (3,0 điểm)


29
Page: Tài Liệu Môn Toán
Website: tailieumontoan.com

Tìm tất cả các số nguyên dương n sao cho hai số n +26 và n – 11 đều là lập phương của
hai số nguyên dương n|o n|o đó.
Câu 3 (3,0 điểm)
Cho tam gi{c ABC v| điểm k thuộc cạnh BC sao cho KB = 2KC, L là hình chiếu cảu B
trên AK, F l| trung điểm cua rBC, biết rằng KAB  2KAC . Chứng minh rằng FL vuông
góc với AC.
Câu 4. (4,0 điểm)
Cho A là tập hợp gồm 8 phần tử, tìm số lớn nhất của các tập con gồm 3 phần tử của A
sao cho giao của 2 tập bất kỳ trong các tập con này không phải là một tập hợp gồm 2
phần tử.
Câu 5. (4,0 điểm)
Cho các số dương x, y, z. Chứng minh bất đẳng thức:

 x  1 y  1  y  1 z  1  z  1 x  1


2 2 2

   x y z 3
3 3 z 2 x2  1 3 3 x2 y 2  1 33 y2 z2 1

ĐỀ 27
PAGE TÀI LIỆU TOÁN HỌC ĐỀ CHỌN HỌC SINH GIỎI TOÁN LỚP 10

MÔN TOÁN
(Thời gian: 180 phút, không tính thời gian giao đề)

Câu 1 (4 điểm)
Cho hàm số y  x 2  x  2

a) Lập bảng biến thiên và vẽ đồ thị của hàm số đã cho.


b) Tìm m để đường thẳng  : y  x  m cắt đồ thị của hàm số đã cho tại hai điểm
phân biệt A, B thỏa mãn độ d|i đoạn thẳng AB bằng khoảng cách từ O đến ∆.
Câu 2 (6 điểm)

 xy  x  10  y  10   81

a) Giải hệ phương trình: 
 x  y  10 x  10 y  18  0

2 2

b) Giải phương trình: 2 x2  5x  7  3  x  1 x  4   8

30
Page: Tài Liệu Môn Toán
Website: tailieumontoan.com

c) Tìm m để phương trình: 4  x  4  x  2 16  x 2  m có nghiệm duy nhất.


Câu 3 (4 điểm)
a) Cho các số dương a, b, c. Chứng minh rằng:
1 1 1 4a  b  c
  
ab  ac bc  ba ca  cb  a  b  b  c  c  a 

b) Cho các số thực x, y, z thỏa mãn x + y + z = 0, x2 + y2 + z2 = 8. Tìm giá trị nhỏ nhất
của biểu thức S  x  y  z

Câu 4. (3 điểm)
a) Cho tam giác ABC có diện tích S và các cạnh BC = a, CA = b thỏa mãn điều kiện
a 2  b2
cotA + cotB= . Chứng minh rằng tam giác ABC vuông.
2S
b) Cho tam gi{c ABC, O l| trọng t}m của tam gi{c. M l| một điểm nằm trong tam
gi{c M kh{c O.Gọi D E F lần lượt l| hình chiếu vuông góc của m lên c{c cạnh BC,
CA, AB. Chứng minh rằng đường thẳng OM đi qua trọng t}m của tam gi{c DEF
Câu 5. (3 điểm)
a) Trong mặt phẳng với hệ tọa độ Oxy, Cho tam giác ABC. Gọi a b lần lượt là
đường trung tuyến v| đường phân giác trong của tam gi{c. c{c đường thẳng
AD lần lượt có phương trình l|
x - y - 2 = 0, y = 0. Giả sử B(1;3), Viết phương trình đường thẳng AC và xác
định tọa độ điểm C.
b) Trong mặt phẳng với tọa độ Oxy, cho tam gi{c ABC, BE v| CD l| c{c đường
cao của tam gi{c.Giả sử D(2;0), E(1;3) v| đường thẳng bc có phương trình 2 x
+ y - 1 = 0. Tìm tọa độ của điểm B biết B có ho|nh độ dương
ĐỀ 28
PAGE TÀI LIỆU TOÁN HỌC ĐỀ CHỌN HỌC SINH GIỎI TOÁN LỚP 10

MÔN TOÁN
(Thời gian: 180 phút, không tính thời gian giao đề)
Câu 1 (4 điểm)
Cho hàm số y  f  x   x 2  2  m  1 x  m

1. Tìm m để bất phương trình f  x   0 nhận mọi x thuộc R là nghiệm.


31
Page: Tài Liệu Môn Toán
Website: tailieumontoan.com

2. Tìm m để bất phương trình f  x   0 có hai nghiệm x1, x2 lớn hơn 1.

Câu 2 (4 điểm)

1. Giải phương trình: 2 x  1  3 x  3  x 2  4 x  3  6,  x  



x  2x y  2 y  x y  2x y  2x
3 2 2 2

2. Giải phương trình:   x, y  


 2x 1  2 y 1  x  2 y 1

Câu 3 (4 điểm)
1. Giải b}t phương trình: 3x  2  x  3  x3  3x  1, x 
2. Chứng minh răng biểu thức sau không phụ thuộc vào x

A  cos4 x  sin 4 x  2sin 2 x 3 3  sin 4 x  cos4 x   2  sin 6 x  cos6 x 

Câu 4. (6 điểm)
1. Cho tam giác ABC. Chứng minh rằng với G là trọng tâm tam giác ABC ta có
GA.GB  GB.GC  GC.GC   .  AB 2  BC 2  CA2 
1
6
2. Trong mặt phẳng với hệ tọa độ Oxy, cho tam giác ABC vuông tại A (1;2). Đường
thẳng chứa canh BC có phương trình: x + y + 1 = 0. Tìm tọa độ B và C, biết AB =
2AC.
3. Trong mặt phẳng với hệ tọa độ Oxy, cho hai đường tròn
(C1):  x  1   y  3  0 và (C2):  x  2    y  2   5
2 2 2 2

Lập phương trình đường thẳng ∆ đi qua A(1;0), đồng thời ∆ cắt c{c đường tròn
(C1) và (C2) lần lượt tại M, N (M, N không trùng A)
Câu 5. (2 điểm) Cho các số thực dương a, b, c thỏa mãn a + b + c = 3. Chứng minh rằng
a b c
 3 2  3 1
a  b  c b  c  a c  a2  b
3 2

ĐỀ 29
PAGE TÀI LIỆU TOÁN HỌC ĐỀ CHỌN HỌC SINH GIỎI TOÁN LỚP 10

MÔN TOÁN
(Thời gian: 180 phút, không tính thời gian giao đề)

Câu 1. (4 điểm)

32
Page: Tài Liệu Môn Toán
Website: tailieumontoan.com

Hãy tìm tất cả c{c số để khi thêm v|o tích sau ta được một số chia hết cho 2011.

A   20112  1
2010
.20102011

Câu 2. (4 điểm)
 x 1 
Tìm tất cả c{c h|m số f  x  x{c định trên thỏa mãn f  x   f    x  1 , với mọi
 x 
x  0, x  1 .

Câu 3. (4 điểm)
Giải phương trình: 3x2  8x  67  8 4 4 x  4  0 .
Câu 4. (4 điểm)
Cho ba số thực dương a, b, c thỏa mãn điều kiện:
a3 b3 c3
   1.
a 2  ab  b2 b2  bc  c 2 c 2  ca  a 2
Tìm gi{ trị lớn nhất của biểu thức S  a  b  c .
Câu 5. (4 điểm)
Cho tam gi{c đều ABC nội tiếp đường tròn  C  có tâm O và bán kính R. Chứng
minh: M   C   MA2  MB2  MC 2  2BC 2 .

ĐỀ 30
PAGE TÀI LIỆU TOÁN HỌC ĐỀ CHỌN HỌC SINH GIỎI TOÁN LỚP 10
MÔN TOÁN
(Thời gian: 180 phút, không tính thời gian giao đề)
sin B  2sin C
Câu 1 . Cho tam giác ABC có sin A  với A, B, C tương ứng là kí hiệu số
2cos B  cos C
đo của các góc BAC, ABC, ACB của tam giác ABC. Chứng minh rằng tam giác ABC
vuông.

 x  xy  2 y  3 y  1
2 2

Câu 2 . Giải hệ phương trình:   x, y  



 x x  y  x  y  2
a  3c a  3b 2a
Câu 3 . Cho các số thực dương a, b, c. Chứng minh rằng:   5
ab ac bc
Câu 4. (5 điểm) Cho các số nguyên n, m, k thỏa m.n = k2 và k không chia hết cho 3.
Chứng minh rằng (m – n) chia hết cho 3

33
Page: Tài Liệu Môn Toán
Website: tailieumontoan.com

Câu 5. (1 điểm) Cho đường tròn (O1) có tâm O1 v| đường tròn (O2) có tâm O2, biết hai
đường tròn cắt nhau tại hai điểm A, B. Vẽ tuyến chung d của hai đường tròn. Gọi C, D
lần lượt là tiếp điểm của d với (O1), (O2); biết A và C khác phía so với O1 O2. Vẽ đường
thẳng đi đi qau A v| song song với d lần lượt cắt BD, BC tại E, F. Chứng minh rằng AE
= AF.
ĐỀ 31
PAGE TÀI LIỆU TOÁN HỌC ĐỀ CHỌN HỌC SINH GIỎI TOÁN LỚP 10

MÔN TOÁN
(Thời gian: 180 phút, không tính thời gian giao đề)


x  y  3y  9
3 3 2

Câu 1 (4 điểm) Giải phương trình sau:  2


x  y  x  4 y

2

Câu 2 (4 điểm) Cho x, y, z là các số thực dương thỏa mãn xy + yz + zx = 3. Chứng minh
x2 y2 z2
bất đẳng thức:   1
x3  8 y3  8 z3  8

Câu 3 (4 điểm) Trên các cạnh BC, CA, AB và về phía ngoài tam giác ABC ta dựng các
hình vuông BCMN, ACPQ, ABEF. Gọi G là trọng tâm tam giác ABC. Kí hiệu A1 là giao
điểm của AG và FQ; B1 l| giao điểm của BG và NE; C1 l| giao điểm của CG và MP. Ta
x{c định c{c điểm A2, B2, C2 sao cho AGC2F, BGA2N, CGB2P là các hình bình hành.
Chứng minh rằng c{c đường đi qua A2, B2, C2 tương ứng vuông góc với B1C1, C1A1, A1B1
đồng quy.
Câu 4. (4 điểm) Giả sử m, n là các số tự nhiên thỏa mãn: 4m3 + m = 12m3 + n. Chứng
minh rằng m – n là lập phương của một số nguyên.
Câu 5. (4 điểm) Trên mặt phẳng tọa độ Oxy, xét tập hợp M c{c điểm có tọa độ (x; y) với
x, y  * và x  12; y  12 . Mỗi điểm trong M được tô bởi một trong ba màu: m|u đỏ,
màu trắng hoặc màu xanh. Chứng minh rằng tồn tại một hình chữ nhật có các cạnh
song song với các trục tọa độ mà tất cả c{c đỉnh nó thuộc M v| được tô màu.
ĐỀ 32
PAGE TÀI LIỆU TOÁN HỌC ĐỀ CHỌN HỌC SINH GIỎI TOÁN LỚP 10

34
Page: Tài Liệu Môn Toán
Website: tailieumontoan.com

MÔN TOÁN
(Thời gian: 180 phút, không tính thời gian giao đề)
Câu 1 (4 điểm)
Giải phương trình sau trên tập số thực:
 6 x  3 7  3x  15  6 x  3x  2  2 9 x 2  27 x  14  11

Câu 2 (4 điểm)
Cho tam giác ABC (BC < AC). Gọi M l| trung điểm của AB, AP vuông góc với BC tại P,
BQ vuông góc với AC tại Q.Giả sử đường thẳng PQ cắt đường thẳng AB tại T. chứng
minh rằng TH  CM, trong đó H l| trực tâm tam giác ABC.
Câu 3 (4 điểm)

( là tập số thực) thỏa mãn f  f  x    x3  x với mọi


3
Cho hàm số f : 
4
x .
Chứng minh rằn tồn tại 3 số thực phân biệt a, b, c sao cho f(a)+f(b)+f(c)=0
Câu 4. (4 điểm)
Giả sử m, n là các số tự nhiên thỏa mãn: 4m3 + m = 12m3 + n. Chứng minh rằng m – n
là lập phương của một số nguyên.
Câu 5. (4 điểm) Tìm số nguyên dương n nhỏ nhất để 2013n – 1 chia hết cho 22014.
ĐỀ 33
PAGE TÀI LIỆU TOÁN HỌC ĐỀ CHỌN HỌC SINH GIỎI TOÁN LỚP 10
MÔN TOÁN
(Thời gian: 180 phút, không tính thời gian giao đề)
Câu 1 (5 điểm)
Cho Parabol (P) có phương trình y  4 x 2  1 , đường thẳng d có phương trình
y  x3

c) Lập phương trình đường thẳng ∆ song song với đường thẳng d sao cho ∆ cắt
(P) tại hai điểm phân biệt A, B và AB = 1.
d) Gọi I l| đỉnh của (P); A, B l| hai điểm phân biệt thuộc (P) và không trùng với I
sao cho IA vuông góc với IB. Tìm quỹ tích trung điểm N của đoạn AB khi A,
B thay đổi.
Câu 2 (5 điểm)
35
Page: Tài Liệu Môn Toán
Website: tailieumontoan.com

d) Giải phương trình: x 1  x2 1  x x


 x 2  21  y  1  y 2

e) Giải hệ phương trình: 

 y  21  x  1  x
2 2

Câu 3 (5 điểm)
c) Cho tam giác ABC có AC = b, BA = a, AB = c ( b < a). Gọi D, E lần lượt là trung
điểm của AB, AC. Đường phân gisc trong của góc C cắt DE tại P. Đường tròn nội
tiếp của tam giác ABC tiếp xúc với AB, BC lần lượt tại N, M.
a) Tính BM , BN , BP theo hai vecto BA, BC và theo a, b, c

b) Chứng minh rằng P, M, N thẳng hàng


d) Cho tam gi{c ABC có AC = b, BA = a, AB = c l| độ dài ba cạnh của tam giác;
ma , mb , mc l| độ d|i ba đường trung tuyến lần lượt xuất phát từ A, B, C. Gọi R, S
lần lượt l| b{n kính đường tròn ngoại tiếp, diện tích của tam giác ABC. Chứng
1 1 1 3
minh rằng nếu    thì tam gi{c ABC đều.
abmc bcma camb 2RS

Câu 4. (3 điểm)
Trong mặt phẳng với hệ tọa độ Oxy, cho tam giác ABC cân tại A, đường thẳng BC có
phương trình
x + 2y – 17 = 0, đường cao CK có phương trình 4x + 3y – 28 = 0, đường cao BH qua điểm
M(1;6). Tìm tọa độ đỉnh A và tính diện tích tam giác ABC.
Câu 5. (2 điểm)
Cho ba số dương a, b, c thỏa mãn a 2  b2  c2  12 . Chứng minh rằng:
1 1 1 8 8 8
   2  2  2
a  b b  c c  a a  28 b  28 c  28
ĐỀ 34
PAGE TÀI LIỆU TOÁN HỌC ĐỀ CHỌN HỌC SINH GIỎI TOÁN LỚP 10
MÔN TOÁN
(Thời gian: 180 phút, không tính thời gian giao đề)

Câu 1 . Giải phương trình 2 x  x 2  2 x  7  2  4 2 x  1

36
Page: Tài Liệu Môn Toán
Website: tailieumontoan.com

4 x 2  2 y 2  4 x  4 y  2 xy  x  y

Câu 2 . Giải hệ phương trình: 
 2 x  1  2  x  y   3  8x y  4 y  4  x  y   1

2 2

Câu 3
a) Chứng minh rằng nếu các cạnh của ta giác ABC thỏa mãn.
sin 2014 A  sin 2014 B  sin 2014 C thì tam gi{c đó nhọn.
b) Cho tam giác ABC có góc C nhọn, AH, BK l| hai đường cao, HK = 7 , diện tích
tứ giác ABHK
bằng 7 lần diện tích ta giác CHK. Tính b{n kính đường tròn ngoại tiếp tam giác
ABC.
Câu 4. Trong mặt phẳng với hệ tọa độ Oxy, cho điểm E(3;-1) v| đường tròn (C) có
phương trình x2  y 2  2 x  8x  14  0. Viết phương trình đường tròn (C’) có tâm E và
cắt (C) theo một d}y cung có độ dài bằng 3 .
Câu 5. Cho ba số dương x, y, z thỏa mãn x  y  z  3xyz .
1 1 1 3
Chứng minh:  2  2 
x  2 y z  1 y  2x z  1 z  2x y  1 4
2 2 2 2 2 2 2

ĐỀ 35
PAGE TÀI LIỆU TOÁN HỌC ĐỀ CHỌN HỌC SINH GIỎI TOÁN LỚP 10

MÔN TOÁN
(Thời gian: 180 phút, không tính thời gian giao đề)

Câu 1 (5 điểm)

 y  3x y  28
3 2

Giải phương trình sau:  2


 x  6 xy  y  6 x  10 y

2

Câu 2 (5 điểm)
Cho tia Ax v| điểm B cố định sao cho góc Bax nhọn, điểm C chạy trên tia Ax. Đường
tròn nội tiếp tam giác ABC tiếp xúc với BC va fAC theo thứ tự của M và N. Chứng minh
rằng, đường thẳng MN luôn đi qua một điểm cố định.
Câu 3 (4 điểm)

37
Page: Tài Liệu Môn Toán
Website: tailieumontoan.com

Cho x, y, z ϵ (0;1). Chứng minh rằng:

 x  x  y  y  z  z    x  yz  y  xz  z  yx 
2 2 2

Câu 4. (4 điểm)
Tìm tất cả các cặp số nguyên dương (m; n) sao cho m2  n2  p là số nguyên tố và
m3  n3  4 chi hết cho p.
Câu 5. (2 điểm)
Trên mạng lưới ô vuông vô hạn người ta điền v|o mỗi ô vuông cơ sở 1 số thực sao
cho mỗi số n|y bằng trung bình cộng với 4 số ở 4 hình vuông ở cơ sở có cạnh kề với
nó.
a) Chứng minh rằng: nếu c{c số được điện v|o c{c ô vuông cơ sở l| những số
nguyên dương thì c{c số đó phải bằng nhau.
b) Nếu c{c số được điện l| c{c số hữu tỉ thì c{c số được điền v|o c{c ô vuông cơ sở
đó cạnh kề với nó, có nhất thiết phải bằng nhau không? Giải thích?
ĐỀ 36
PAGE TÀI LIỆU TOÁN HỌC ĐỀ CHỌN HỌC SINH GIỎI TOÁN LỚP 10

MÔN TOÁN
(Thời gian: 180 phút, không tính thời gian giao đề)
3x
Câu 1 (5 điểm) Giải phương trình sau:  3x  1 2 x 2  1  5 x 2  3
2
 12
(1  y  3x
) x 2

Câu 2 (5 điểm) Giải hệ phương trình: 
(1  12
) y 6
 y  3x

Câu 3 (3 điểm)
Cho các số thực dương a, b, c thỏa mãn: 9  a 4  b4  c 4   25  a 2  b2  c 2   48  0

a2 b2 c2
Tính giá trị nhỏ nhất của biểu thức: P   
b  2c c  2a a  2b
Câu 4. (5 điểm)

38
Page: Tài Liệu Môn Toán
Website: tailieumontoan.com

Cho tam giác ABC có 3 góc nhọn, ph}n gi{c trong AD, Đường tròn đường kính AD
cắt đường thẳng BC tại H, cắt đường thẳng AB tại M và cắt đường thẳng AC tại N.
Chứng minh rằng c{c đường thẳng CM, BN, AH đồng quy.
Câu 5. (1 điểm)
Chứng mih rằng trong dãy 9; 99; 999;9999;... có vô số số hạng chia hết cho 17.
ĐỀ 37
PAGE TÀI LIỆU TOÁN HỌC ĐỀ CHỌN HỌC SINH GIỎI TOÁN LỚP 10

MÔN TOÁN
(Thời gian: 180 phút, không tính thời gian giao đề)

Câu 1 (4 điểm) Giải phương trình 7 x2  7 x  9  x2  x  6  2 2 x  1


Câu 2: (4 điểm) Cho ABCD là tứ giác nội tiếp có giao điểm P của hai đường phân giác
của các góc BAD, BCD nằm trên đường chéo BD. Gọi Q l| trung điểm của BD. Đường
thẳng qua t song song với AD cắt tia AQ tại K nằm ngoài tứ giác ABCD. Chứng minh
rằng am giác CDK là tam giác cân.
Câu 3 (4 điểm) Cho ba số thực dương x, y v| z thay đổi nhưng luôn thỏa mãn điều
kiện: xy+ yz + zx = 3xyz. Tìm giá trị nhỏ nhất của biểu thức.
y2 z2 x2
S=  
x  y 2  1 y  z 2  1 z  x 2  1

Câu 4 (4 điểm) Mỗi điểm trong mặt phẳng được tô bởi một trong hai màu xanh hoặc
đỏ. Chứng minh rằng tồn tại tam gi{c m| ba đỉnh và trọng tâm cùng màu.
Câu 5 (4 điểm)
Chứng minh rằng tồn tại 16 số tự nhiên liên tiếp sao cho không có số nào trong 16 số đó
có thể biểu diexn được dưới dạng: 7 x 2  9 xy  5 y 2  x, y  Z 

ĐỀ 38
PAGE TÀI LIỆU TOÁN HỌC ĐỀ CHỌN HỌC SINH GIỎI TOÁN LỚP 10

MÔN TOÁN
(Thời gian: 180 phút, không tính thời gian giao đề)
39
Page: Tài Liệu Môn Toán
Website: tailieumontoan.com

Câu 1
Cho bộ số gồm 8 số D = {T, R, A, I, H, E, P, N} và
T  R R  A A  I I  H H  E E  P P  N N  T 
T  ; 
 2 2 2 2 2 2 2 2 

Là một hoán vị của D. Biết rằng T+ R+ A+ I+ H+ E+ P+ N = 2014.


Hãy x{c định các giá trị N
Câu 2
Giải phương trình: x2  x  3  3  2 x
Câu 3

 12  2 x  4  y
2

Giải hệ phương trình:  (1)



 1  2 y  y 2
 5  2 x

Câu 4.
Cho tam giác ABC nội tiếp đường tròn O. Ph}n gi{c trong của góc A cắt BC tai A1 và
cắt đường tròn O tại A2. Tương tự ta thu được c{c điểm B1, B2, C1, C2, tương ứng.
A1 A2 B1B2 C1C2 3
Chứn gminh rằng   
BA2  A2C CB2  B2 A AC2  C2 B 4

Câu 5.
Cho số nguyên tố có 4 chữ số p = abcd . Chứng minh rằng đa thức:
P(x) = ax3 + bx2 + cx +d không ph}n tích được thành tích của hai đa thức với hệ số
nguyên
ĐỀ 39
PAGE TÀI LIỆU TOÁN HỌC ĐỀ CHỌN HỌC SINH GIỎI TOÁN LỚP 10

MÔN TOÁN
(Thời gian: 180 phút, không tính thời gian giao đề)

Câu 1 (3 điểm)

40
Page: Tài Liệu Môn Toán
Website: tailieumontoan.com

e) Cho phương trình bậc hai x2  2mx  3m  2  0 , trong đó x là ẩn, m là tham số.
Tìm tất cả các giá trị thực của m sao cho phương trình đã cho có hai nghiệm
x1.x2 và x12  x22 đạt giá trị nhỏ nhất.
f) Cho tam thức bậc hai f  x   ax 2  bx  c, a  0. Chứng minh rằng nếu
f  x   0 với mọi x  R thì 4a + c ≥ 2b.

Câu 2 (2 điểm)
f) Giải phương trình: x  2  3x  1  2 x  3 (x  R)

 x  y   x  xy  y  3  3  x  y   2
 2 2 2 2

g) Giải hệ phương trình:   x, y  



 x  6  y  3   x 2
 2 x  8

Câu 3 (2 điểm)
a) Cho a, b, c là các số thực dương thỏa mãn a + b + c = 1. Chứng minh rằng
a 2 b2 c 2
   3  a 2  b2  c 2 
b c a
b) Giải bất phương trình: 3
3  x  1 x  2  x  R

Câu 4. (3 điểm)
a) Cho tam giác ABC, dựng về phía ngoài tam giác ABC hai tam giác vuông ABE và
ACF với BAE  CAF  900 , sao cho tam gi{c ABE đồng dạng với tam giác ACF.
Gọi M l| trung điểm BC, chứng minh rằng AM vuông góc với EF.
b) Cho tam giác ABC không vuông với a = BC, b = CA , c = AB. Chứng minh rằng
nếu a 2  b2  2c2 và tan A + tan B = 2tan C thì ABC là một tam giác cân.
Trong mặt phẳng với hệ trục tọa độ Oxy, cho tam giác ABC có t}m đường tròn
11 1
ngoại tiếp và trong tâm lần lượt có tọa độ là I (4;0), G ( ; ). Tìm tọa độ c{c đỉnh A, B,
3 3
C của tam giác ABC biết rằng đỉnh B nằm trên đường thẳng (d): 2x + y – 1 = 0 v| điểm
M(4;2) nằm trên đường cao kẻ từ đỉnh B cảu tam giác ABC.
ĐỀ 40
PAGE TÀI LIỆU TOÁN HỌC ĐỀ CHỌN HỌC SINH GIỎI TOÁN LỚP 10

MÔN TOÁN

41
Page: Tài Liệu Môn Toán
Website: tailieumontoan.com

(Thời gian: 180 phút, không tính thời gian giao đề)
Câu 6.
1. Giải phương trình: 2  x  6  3 x  5  x  3 .

2. C{c số a, b, c thỏa mãn điều kiện: a  2b  5c  0 . Chứng minh phương trình


ax2  bx  c  0 có nghiệm.

Câu 7.

 x  4 xy  x  2 y  0
2

Giải hệ phương trình: 


 x  8 x y  3x  4 y  0

4 2 2 2

Câu 8. Trong mặt phẳng với hệ tọa độ Oxy, cho c{c điểm A 1;3 , B  5; 3 . X{c định tọa
độ điểm M trên đường thẳng d : x  2 y  1  0 sao cho 2MA  MB đạt giá trị nhỏ nhất.
Câu 9.
Tam giác ABC có c{c góc thoả mãn hệ thức: cot A  cot C   cot B .
1
1. X{c định góc giữa hai đường trung tuyến AA1 và CC1 của tam gi{c ABC khi   .
2
2. Tìm gi{ trị lớn nhất của góc B khi   2 .
Câu 10.
1 1 1
Ba số dương a, b, c thỏa mãn: 2
 2  2  1.
a b c
Tìm gi{ trị lớn nhất của biểu thức:
1 1 1
P  
5a  2ab  2b
2 2
5b  2bc  2c
2 2
5c  2ca  2a 2
2

ĐỀ 41
PAGE TÀI LIỆU TOÁN HỌC ĐỀ CHỌN HỌC SINH GIỎI TOÁN LỚP 10

MÔN TOÁN
(Thời gian: 180 phút, không tính thời gian giao đề)

Câu 1. Cho a, b, c dương thỏa mãn (a + b + c)abc = 1. Tìm giá trị bé nhất của biểu thức:
a5 b5 c5
S 3  
a  2b3 b3  2c3 c3  2a3
Câu 2. Cho số nguyên n ≥ 2
42
Page: Tài Liệu Môn Toán
Website: tailieumontoan.com

a) Hãy x{c định tất cả các bộ  a1 ,......an  nguyên dương sao cho
 a1 ! 1 ..... an ! 1  9 là số chính phương.
b) Hãy x{c định tất cả các bộ  a1 ,......an  nguyên dương sao cho
 a1 ! 1 ..... an ! 1 1 là số chính phương.
Câu 3. Cho tam giác ABC không vuông và không cân, O, H theo thứ tự l|m t}m đường
tròn ngoại tiếp và trực tâm của tam giác ABC. E, F theo thứ tự l| giao điểm của BH, CH
và AC, AB. M, N theo thứ tự l| trung điểm của AB, AC. O1,O2 theo thứ tự nằm tâm
đường tròn ngoại tiếp của tam giác BEM, CFN. Chứng minh rằng OH  O1O2
Câu 4. Tìm số nguyên dương K bé nhất sao cho với mỗi tập gồm k số tự nhiên luôn tồn
tại 6 phần tử trong tập có tổng là bội của 6
Ngày thi thứ 2

 x y 2
Câu 1. Giải hệ phương trình 
 x  15  y  15  8

Câu 2. Cho bốn số dương a, b, c, d thỏa mãn a + b + 1 = 7c v| đa thức
P(x) = x3  ax2  bx  c có ba nghiệm thực (không nhất thiết phân biệt).
Đặt Q(x) = x2  2 x  d . Chứng minh rằng:
a) Tích ba nghiệm của đa thức P(x) không vượt quá -1
b) Phương trình P(Q(x)) = 0 có tối đa bốn nghiệm thực phân biệt
Câu 3. Cho tứ giác ABCD nội tiếp đường tròng (O) và không phải là hình tang. M, N
theo thứ tự l| trung điểm cảu AB, CD. Gọi E = AD ∩ BC, F = AC ∩ BD, P = EF ∩ AB v| Q
= EF ∩ CD. Chứng minh rằng:
a) M, N, P, Q cùng thuộc một đường tròn có tâm là T.
b) MQ, NP, OT đồng quy.
Câu 4.Cho ba số nguyên dương a, m, n trong đó a l| số chẵn và n > 1.
1. Chứng minh rằng mọi ước nguyên tố của A = a 2 +1 đều có dạng 2n1 k  1 , với k
n

là số tự nhiên.
m
2. Giả sử rằng a m  1 chia hết cho A. Chứng minh rằng n là số nguyên dương lẻ
2
ĐỀ 42
PAGE TÀI LIỆU TOÁN HỌC ĐỀ CHỌN HỌC SINH GIỎI TOÁN LỚP 10

MÔN TOÁN
(Thời gian: 180 phút, không tính thời gian giao đề)

43
Page: Tài Liệu Môn Toán
Website: tailieumontoan.com

Câu 1 (2,0 điểm)


2014 2015
Tìm tập x{c định của hàm số: f  x   
 x2  2 x  3 x2  2x
Câu 2 (1,0 điểm)
x
a) Chứng minh rằng hàm số f  x   đồng biến trên khoảng  1;  
x 1
b) Chứng minh rằng hàm số f  x   2015  x  2015  x là một hàm số lẻ

Câu 3 (1,0 điểm)

Giải phương trình 19  3x  4  x2  x  6  6 2  x  12 3  x


Câu 4. (1,0 điểm)

 x  2 y  3xy  y  1  0
2 2

Giải hệ phương trình:  2


x  y  y  3  0

2

Câu 5. (1,0 điểm)


Tìm tất cả các giá trị m sao cho bất phương trình

 m 1  2  m  2   2m  2  0 vô nghiệm (x là ẩn, m là tham số)


2

Câu 6. (1,0 điểm)


Cho tam giác ABC không cân nội tiếp đường tròn tâm O và G là trọng tâm của
tam giác ABC. Gọi M, N, P lần lượt là trọng t}m tam gi{c OBC, OCA, OAB, v| G’ l|
trọng tâm tam giác MNP. Chứng minh rằng O, G, G’ thẳng hàng.
Câu 7. (1,0 điểm)
Cho tam giác ABC không vuông và có cạnh BC = a, CA = b, AB = c. Chứng minh rằng
nếu tam giác ABC thỏa mãn a 2  b2  2c2 v| tanA + tanC = 2 tanB thì tam gi{c ABC đều.
Câu 8. (1,0 điểm)
Trong mặt phẳng với hệ tọa độ Oxy, cho tam giác ABC không là tam giác vuông
và nội tiếp đường tròn (I) (đường tròn (I) có t}m l| I); điểm H(2;2) là trực tâm tam giác
ABC. kẻ c{c đường kính AM, BN của đường tròn (I). tìm tọa độ c{c đỉnh của tam giác
ABC biết M (5;3) N (1;3) v| đường thẳng BC đi qua điểm P (4;2).
Câu 9. (1,0 điểm)
Cho a b c l| c{c số thực dương thỏa mãn điều kiện A + B + C = 2015 chứng minh
rằng

44
Page: Tài Liệu Môn Toán
Website: tailieumontoan.com

2015a  a 2 2015b  b2 2015c  c 2  2015  a 2015  b 2015  c 


   6  2 2    
bc ca ab  a b c 
ĐỀ 43
PAGE TÀI LIỆU TOÁN HỌC ĐỀ CHỌN HỌC SINH GIỎI TOÁN LỚP 10

MÔN TOÁN
(Thời gian: 180 phút, không tính thời gian giao đề)

Câu 1 (4,0 điểm) Giải phương trình sau trên tập số thực:

1  x2  x2  x  1  3 1  x  1 .
Câu 2 (4,0 điểm) Cho tam giác ABC . Gọi (O1 ) l| đường tròn đi qua B và tiếp xúc với
AC tại A ; (O2 ) l| đường tròn đi qua C và tiếp xúc với AB tại A . P l| giao điểm thứ
hai của (O1 ) và (O2 ) ; K , L theo thứ tự l| giao điểm thứ hai của (O1 ), (O2 ) với đoạn
thẳng BC . Gọi ( S ) l| đường tròn ngoại tiếp tam giác PKL .
a) Chứng minh rằng: AK , AL tiếp xúc với ( S ) .

b) Gọi Q l| giao điểm thứ hai của ( S ) và AP ; E l| giao điểm của QK và AB ; F


l| giao điểm của QL và AC . Chứng minh rằng c{c điểm A, K , L, S , E, F cùng thuộc
một đường tròn. (Chú ý. Ta kí hiệu ( X ) là đường tròn có tâm X ).

Câu 3 (4,0 điểm) Cho đa thức f ( x)  x 4  x3  mx2  nx  p , trong đó m, n, p là các số


nguyên đôi một phân biệt, khác không, sao cho f (m)  m4  m3 và f (n)  n4  n3 . Tìm
m, n, p .
Câu 4 (4,0 điểm) Tìm tất cả các cặp số nguyên dương (a, b) thỏa mãn đồng thời hai điều
kiện sau:
i) a  b2 l| lũy thừa của một số nguyên tố;
ii) a 2  b chia hết cho a  b2 .
Câu 5 (4,0 điểm) Cho tập S  {1,2,3,...,2025} . Tìm số nguyên dương nhỏ nhất n sao
cho: Với mọi tập con T của S gồm n phần tử, tồn tại hai phần tử phân biệt u, v  T sao
cho u  v  20.
ĐỀ 44
PAGE TÀI LIỆU TOÁN HỌC ĐỀ CHỌN HỌC SINH GIỎI TOÁN LỚP 10
45
Page: Tài Liệu Môn Toán
Website: tailieumontoan.com

MÔN TOÁN
(Thời gian: 180 phút, không tính thời gian giao đề)

Câu 1 (1,5 điểm)

Cho hàm số y  x  3x  2 và hàm số y   x  m . Tìm m để đồ thị các hàm số


2

đó cắt nhau tại hai điểm phân biệt A, B đồng thời khoảng cách từ trung điểm I của đoạn
thẳng AB đến các trục tọa độ bằng nhau.
Câu 2 (2,0 điểm)

a) Giải phương trình sau trên : 3x  1  x  1  9  x .


9
b) Giải bất phương trình sau:  x2 .
x 5 3
Câu 3 (1,0 điểm)

2 x  y  3xy  3x  2 y  1  0
 2 2

Giải hệ phương trình 


4 x  y  x  4  2 x  y  x  4 y

2 2

Câu 4 (2,5 điểm)


a) Trong mặt phẳng tọa độ Oxy cho tam giác ABC có B(1;2). Đường thẳng  là
đường phân giác trong của góc A có phương trình 2x  y  1  0 , khoảng cách từ C đến
 gấp 3 lần khoảng cách từ B đến  . Tìm tọa độ của A và C biết C nằm trên trục tung.
b) Cho tam giác ABC vuông ở A; BC = a; CA = b; AB = c. X{c định điểm I thỏa
mãn hệ b2 IB  c2 IC  2a 2 IA  0 . Tìm điểm M sao cho biểu thức
thức:
( b MB  c MC  2a MA ) đạt giá trị lớn nhất.
2 2 2 2 2 2

Câu 5 (2,0 điểm)


a) Chứng minh rằng c{c biểu thức sau không phụ thuộc v|o a.

E  sin 4 a  4cos2 a  cos4 a  4sin 2 a


b) Cho tam gi{c ABC v| điểm K thuộc cạnh BC sao cho KB=2KC, L là hình chiếu
của B trên AK, F l| trung điểm của BC, biết rằng KAB  2KAC . Chứng minh rằng
FL vuông góc với AC.

46
Page: Tài Liệu Môn Toán
Website: tailieumontoan.com

Câu 6 (1,0 điểm)


Cho x, y, z là các số thực dương thỏa mãn x  y  z  xyz . Chứng minh rằng:

1  1  x2 1  1  y 2 1  1  z 2
   xyz .
x y z
ĐỀ 45
PAGE TÀI LIỆU TOÁN HỌC ĐỀ CHỌN HỌC SINH GIỎI TOÁN LỚP 10

MÔN TOÁN
(Thời gian: 180 phút, không tính thời gian giao đề)

Câu 1 (4 điểm)

 y  2 xy  7 y   x  7 x  8
4 2 2 2

Giải hệ phương trình sau: 


 3  x  y 1  x  x  4 y  3

2 3 2 2

Câu 2 (4 điểm)
Cho đường tròn (w1) và (w2) cắt nhau tại P v| Q, một đường thẳng d thay đổi đi qua
B Cắt w1 tại A v| c{c (w2) tại B sao cho P nằm giữa a v| b; C, D l| hai điểm cố định
lần lượt thuộc (w1) và (w2) sao cho P thuộc tia đối của tia DC. tia BD v| đoạn AC cắt
nhau tại X, điểm y thuộc (w1) sao cho đường thẳng PY song song với đường thẳng
BD, điểm Z thuộc (w2) sao cho đường thẳng PZ song song với đường thẳng AC. Gọi
I v| J lần lượt l| t}m của c{c đường tròn ngoại tiếp tam gi{c ABQ v| CDQ.
a) chứng minh rằng đường thẳng IJ vuông góc với đường thẳng XQ
b) chứng minh rằng đường thẳng YZ luôn đi qua một điểm cố định khi d thay đổi.
Câu 3 (4 điểm)
Cho số nguyên tố p và ba số nguyên dương x, y, z thỏa mãn x < y < z < p. Chứng
minh rằng nếu x3  y3  z 3 (mod p) thì x 2  y 2  z 2 chia hết cho x + y + z.

Câu 4. (4 điểm)
3
Xét c{c số thực dương x,y v| z thỏa mãn x + y + z ≤ .
4
Tìm gi{ trị nhỏ nhất của biểu thức:

47
Page: Tài Liệu Môn Toán
Website: tailieumontoan.com

P  x y  y z   1 1 1
z x   
x y z
Câu 5. (4 điểm)
Có 42 học sinh tham iga một buổi giao lưu. Biết rằng cứ 3 học sinh bất kỳ, đều có ít
nhất một cặp đôi gồm hai học sinh có trao đổi kinh nghiệm học tập với nhau. Kí
hiệu k l| số cặp đôi như thế. Tìm gi{ trị nhỏ nhất cuả k.

ĐỀ 46
PAGE TÀI LIỆU TOÁN HỌC ĐỀ CHỌN HỌC SINH GIỎI TOÁN LỚP 10

MÔN TOÁN
(Thời gian: 180 phút, không tính thời gian giao đề)

Câu 1 (4 điểm)

Giải phương trình sau trên tập số thực x3  x2  3x  1  x3  6 x  2  5


Câu 2 (4 điểm)
Cho ∆ABC có ba góc nhọn v| nội tiếp đường tròn (O). Goin H l| trực t}m ∆ABC v|
P l| điểm trên đoạn BC (P ≠ B; P ≠ C). Đường thẳng AH cắt đường tròn ngoại tiếp
∆BHC tại T (T ≠ H). Đường thẳng TP cắt đường tròn ngoại tiếp ∆BHC tại K (K ≠ T).
Giả sử BK cắt AC tại M; CK cắt AB tại N. Gọi X, Y lần lượt l| trung điểm của BN,
CM
a) chứng minh rằng tứ gi{c ANKM nội tiếp.

b) chứng minh rằng XPY có số đo không đổi khi P di động trên BC


Câu 3 (4 điểm)
1 1 1
Xét c{c số thực dương x,y v| z thỏa mãn   3 .
x y z
x y z 3
Chứng minh rằng:  4  4 
x  1  2 xy y  1  2 yz z  1  2 zx 4
4

Câu 4. (4 điểm)

48
Page: Tài Liệu Môn Toán
Website: tailieumontoan.com

Với tam thức bậc hai a 2  bx  c cho phép thực hiện các phép biến đổi sau:
(i) Đổi chỗ a và c cho nhau hoặc,
(ii) Thay đổi x bởi x + t với t là một số thực bất kì.
Bằng cách lặp lại các phép biến đổi trên có thể biến đổi tam thức x2  8x  2015 thành
tam thức 2016 x2  8 x 1 hay không?
Câu 5. (4 điểm)
Tìm tất cả c{c số tự nhiên n sao cho số n7  n5  2n4  n3  n2  1 có đúng một ước số
nguyên tố.
ĐỀ 47
PAGE TÀI LIỆU TOÁN HỌC ĐỀ CHỌN HỌC SINH GIỎI TOÁN LỚP 10

MÔN TOÁN
(Thời gian: 180 phút, không tính thời gian giao đề)

 5 x  2 xy  2 y  2 x  2 xy  5 y  3  x  y 
2 2 2 2

Câu 1 (4 điểm) Giải hệ phương trình sau: 


 2 x  y  1  2 7 x  12 y  8  2 xy  y  5
 3

Câu 2 (4 điểm)
Cho đường tròn (O) đường kính AB, C l| điểm di động trên (O) không trùng với
A và B. Các tiếp tuyến của (O) tại B và C cắt nhau tại N, AN cắt (O) tại D khác A. Tiếp
tuyến của (O) tại D cắt CN tại P. Chứng minh rằng P di động trên một đường cố định
khi C di động trên (O).
Câu 3 (3 điểm)
Cho a, b, c là ba số thực dương tùy ý. Chứng minh:
a b c
  1
7a  b  c
2 2 2
a  7b  c
2 2 2
a  b 2  7c 2
2

Câu 4. (3 điểm)
Tìm tất cả c{c số nguyên dương k sao cho phương trình: x2  y 2  x  y  kxy có
nghiệm nguyrn dương x,y.
Câu 5. (1,0 điểm)
Cho trước số nguyên dương n  2 . Trong một giải đấu cờ vua có 2n động viên tham
gia mỗi người đối với người kh{c đúng một v{n.Tại thời điểm trong giải, người ta

49
Page: Tài Liệu Môn Toán
Website: tailieumontoan.com

thấy có n2  1 v{n đấu đã diễn ra. Chứng minh rằng khi đó có thể chọn ra 3 vận
động viên sao cho hai người bất kỳ trong ba người được chọn đều đã thi đấu với
nhau.
Câu 6. (1,0 điểm)
Cho hàm số f:N* → N* \ {1} (N* là tập hợp các số nguyên dương) thỏa mãn:
f(n) + f(n + 1) = f(n + 2)f(n + 3) – 168
Tính f(2014)

ĐỀ 1.
Câu 1.
(3,0 điểm)
1a (2,0 điểm)
m  2
Phương trình đã cho có hai nghiệm  '  m2  3m  2  0  
m  1
Theo định lí Vi – ét ta có x1  x2  2m, x1 x2  3m  2

Do đó x12  x22   x1  x2   2 x1 x2  4m2  2  3m  2   4m2  6m  4


2

Lập bảng biến thiên của hàm số f  m   4m2  6m  4 trên  ;1 2;   ta được

m - 1 2 +

+ +

f m

2 8

Từ bảng biến thiên ta được f  m   4m2  6m  4 đạt giá trị nhỏ nhất khi m  1 .

1b (1,0 điểm)

50
Page: Tài Liệu Môn Toán
Website: tailieumontoan.com

Do f  x   0 với mọi x  nên f  0   0  c  0 .

a  0 a  0
Mặt khác f  x   0 với mọi x     2
  b  4ac  0 b  4ac
2

Ta có 4a  c  2 4ac  2 b2  2 b  2b .

Câu 2. 2a (1,0 điểm)



3 x  0 x  0
 
Đkxđ  x  2  0   x  2  x  2
2 x  3  0 
 x  
3
 2
Phương trình đã cho tương đương với:

x  2  2 x  3  3x  1
 x  2  2x  3  2  x  2  2 x  3  3x  1  2 3x

  x  2 2 x  3  3x

  x  2  2 x  3  3x
 x  1
 2 x 2  x  6  3x  2 x 2  4 x  6  0  
x  3
Kết hợp với đkxđ ta được x  3 . Vậy tập nghiệm của phương trình l| S  3 .

2b (1,0 điểm)
Đkxđ: x  6, y  3

Từ phương trình đầu của hệ ta có:

 x  y   x 2  xy  y 2  3  3  x 2  y 2   2
  x  y   x 2  xy  y 2   3  x  y   3x 2  3 y 2  2
 x3  y 3  3x  3 y  3x 2  3 y 2  2
  x  1   y  1  x  1  y  1  y  x  2  x  1
3 3

Thay v|o phương trình thứ hai của hệ ta được:

51
Page: Tài Liệu Môn Toán
Website: tailieumontoan.com

x  6  x  1   x2  2x  8
 x  6  3  x  1  2  x2  2x  3  0
x 3 x 3
    x  3 x  1  0
x6 3 x 1  2
 1 1 
  x  3    x  1  0
 x6 3 x 1  2 
 x3
So sánh với đkxđ ta được  x, y    3,1 .

Câu 3.
3a (1,0 điểm)
a b c 3
Ta có   
 a  1 b  1  b  1 c  1  c  1 a  1 4

 4a  c  1  4b  a  1  4c  b  1  3  a  1 b  1 c  1

 4  ab  bc  ca   4  a  b  c   3abc  3  3  ab  bc  ca   3  a  b  c 

 ab  bc  ca  a  b  c  6
Áp dụng bất đẳng thức Cô Si cho ba số dương ta được:
ab  bc  ca  3 3 ab.bc.ca  3
a  b  c  3 3 abc  3
Cộng từng vế hai bất đẳng thức trên ta được ab  bc  ca  a  b  c  6 .
Dấu đẳng thức xảy ra khi và chỉ khi a  b  c  1 . Vậy bđt được chứng minh.
3b (1,0 điểm)
Đkxđ x  2 . Đặt t  x  2, t  0 suy ra x  t 2  2 , thay vào bất phương trình ta được:

1  t 2  1  t  1  t 2  1  t 
3 3

1  t 2  1  t  1  t 2  1  t   t 3  4t 2  3t  0  t  t  1 t  3  0
3 3

t  3  x2 3  x  11
  
0  t  1 0  x  2  1  2  x  3

Kết hợp với đkxđ ta được tập nghiệm là S   2;3 11;   .


Câu 4.

52
Page: Tài Liệu Môn Toán
Website: tailieumontoan.com

(3,0 điểm)
4a (1,0 điểm)

M
G
O

 
2
Ta có OA  OB  OC  3.OG  9.OG 2  OA  OB  OC

 OA2  OB 2  OC 2  2.OA.OB  2.OB.OC  2.OC.OA


 3R 2  OA2  OB 2  AB 2  OB 2  OC 2  BC 2  OC 2  OA2  CA2
 9R2  a 2  b2  c2 .
Do 4 điểm A, G, O, M cùng nằm trên một đường tròn nên OG vuông góc với GA hay

OG 2  GA2  OA2 
1
9
 9 R 2  a 2  b2  c 2    2b2  2c 2  a 2   R 2
1
9
 9 R 2  a 2  b2  c 2  2b2  2c 2  a 2  9 R 2
 b 2  c 2  2a 2
4b (1,0 điểm)
2S
sin A 4S
Ta có tan A   2 bc2  2 2 . Tương tự ta tính được tan B, tan C
cos A b  c  a 2
b  c  a2
2bc
Theo giả thiết
4S 4S 4S
tan A  tan B  2 tan C   2 2 2
b c a c  a b
22 2 2 2
a  b2  c 2
53
Page: Tài Liệu Môn Toán
Website: tailieumontoan.com

2 2

 a 4   b2  c 2   b4   c 2  a 2   2 c 4   a 2  b2 
2

 a4  b4  c4  2b2c2  b4  c4  a4  2c2a2  2c4  2a 4  2b4  4a 2b2

 2c 4   a 2  b 2   c 2  a 2  b 2 
2

 2c 4   a 2  b 2   2c 4  a  b
2

Hay tam giác ABC cân


4c (1,0 điểm)

A
M

N
H

G I
(d)
C
B

Ta chứng minh IH  3.IG


x4 y2
Suy ra H  3;1  pt MH :   x y20.
3  4 1 2
Do B là giao của (d) v| đường thẳng MH nên tọa độ B là nghiệm của hệ:
x  y  2  0 x  1
   B 1; 1 .
2 x  y  1  0  y  1
3
Gọi N l| trung điểm của AC. Khi đó BN  BG  N  5;1
2
Ta có nAC  uMN  1;1  pt AC :1 x  5  1.  y  1  0  x  y  6  0 .

Do A thuộc đường thẳng AC nên A  t;6  t  , kết hợp với I l| t}m đường tròn ngoại tiếp
t  3
tam giác ABC nên IA2  IB 2   t  4    6  t   10  
2 2

t  7
+) Với t  3  A 3;3  , C 7; 1 

+) Với t  7  A  7; 1 , C 3;3 


54
Page: Tài Liệu Môn Toán
Website: tailieumontoan.com

Vậy A  3;3 , B 1; 1 , C  7; 1 hoặc A  7; 1 , B 1; 1 , C 3;3 

ĐỀ 2
Câu 1
1.1
2,0 điểm

(2 x  3) 4 x  1  (2 y  3) 4 y  1  2 (2 x  3)(2 y  3) (1)

 y  x  4 xy
 (2)
1 1
Điều kiện x{c định: x  ; y 
4 4
x y
(2)  x  y(4 x  1)   4 x  1   4 y  1 thay v|o (1) ta được
y x

x y
(2 x  3)  (2 y  3)  2 (2 x  3)(2 y  3)
y x

x y
Do (2 x  3)  (2 y  3)  2 (2 x  3)(2 x  3)
y x

Suy ra (1)  x(2 x  3)  y(2 y  3)  ( x  y)(2 x  2 y  3)  0  x  y thay v|o (2) ta được


 x  0 (lo¹i)
2 x2  x  0  
x  1  y  1
 2 2
1 1
Vậy hệ phương trình có nghiệm  ;  .
2 2
1.2
1,0 điểm
Ta có: f  x  y   f  x   y  f ( y)  f (0)  y y  .  f ( x)  a  x với a  f (0) .

1 1 1
f    f (0)   a  x  0.
 x x x
 1  f ( x) f (0)  x a  x
Mặt khác f    2   2 x  0 .
 x x x2 x
1 ax
a  2 x  0  ax 2  a x  0  a  0.
x x
55
Page: Tài Liệu Môn Toán
Website: tailieumontoan.com

Vậy f ( x)  x x  .

Câu 2. 2,0 điểm


p , q đều khác 2, 7 . Không mất tính tổng quát ta giả sử q  p . Khi đó từ giả thiết ta
được 7 p  4 p p hoặc 7q  4q p

TH1. 7 p  4 p p , theo định lí Fermat ta có: 7 p  4 p  3  mod p   3  0  mod p   p  3.

TH2. 7q  4q p , ta có  p  1, q   1  tồn tại 2 số nguyên dương u , v sao cho


qv   p  1 u  1  7q  4q  mod p   7qv  4qv  mod p   7    4    mod p 
1 p 1 u 1 p 1 u

 7  4  mod p   3  0  mod p   p  3.

Với p  3 , từ giả thiết ban đầu ta được:

7 3
 43  7q  4q  3q  9.31.  7q  4q  3q  q  3, q  31.

Vậy  p , q   3, 3 ,  31, 3 ,  3, 31 .

Câu 3.
2,0 điểm
A

I1

H
K
I3

D
I2
L
C
F

Giả sử tiếp tuyến qua H song song với CD của đường tròn  I1  cắt BC tại K v| đường
thẳng qua H song song với BC cắt đường thẳng CD tại L, suy ra CKHL là một hình bình
hành.
Do các tứ giác ABCD, ABKH ngoại tiếp, nên

56
Page: Tài Liệu Môn Toán
Website: tailieumontoan.com

AD  HL  AD  CK  AD  BC  BK
 AB  CD  BK  AB  BK  CD  AH  HK  CD
 AH  LC  CD  AH  DL
Suy ra tứ giác ADLH ngoại tiếp, hay HL tiếp xúc với ( I 3 )

Vì FD  KH ; FH  HA nên c{c đường phân giác HI1 của góc AHK và FI 3 của góc
HFD vuông góc với nhau; hay I1H  I 2 I3 (Do F , I 2 , I 3 thẳng hàng) (1)

Chứng minh tương tự, cũng được HI3  EI 2 hay I3 H  I1I 2 (2)

Từ (1) v| (2) suy ra điều phải chứng minh.


Câu 4.
2,0 điểm
Theo bất đẳng thức AM-GM, ta có

4 4 3 4
a  2 a·  4   a    3, dấu đẳng thức xảy ra khi và chỉ khi a  2
a a 4 a

9 9 1 9
b  2 b·  6   b    3, dấu đẳng thức xảy ra khi và chỉ khi b  3
b b 2 b

16 16 1  16 
c  2 c·  8   c    2, dấu đẳng thức xảy ra khi và chỉ khi c  4
c c 4 c 
3a b c 3 9 4
Cộng ba bất đẳng thức cùng chiều, thu được      8 (1)
4 2 4 a 2b c
a b 3c
Mặt khác, do a  2b  3c  20 nên    5 (chia hai vế cho 4) (2)
4 2 4
3 9 4
Cộng (1) và (2), vế đối vế, ta được L  a  b  c     13.
a 2b c
Dấu đẳng thức xảy ra khi và chỉ khi a  2, b  3, c  4. Vậy giá trị nhỏ nhất của biểu thức
L bằng 13, đạt được khi a  2, b  3, c  4.

Câu 5.
1,0 điểm
Giả sử tìm được tập hợp X thỏa mãn và m  n là hai phần tử bé nhất của X. Khi đó, do
c{ch x{c định X nên tồn tại k  X sao cho n  mk 2 . Suy ra m  k  n v| do đó k  m hoặc
k  n.
Với k  n  n  m.n 2  m.n  1 vô lí.

57
Page: Tài Liệu Môn Toán
Website: tailieumontoan.com

Với k  m  m  n  m3  m  1


+) Nếu | X | 2 thì tập hợp X  m, m3 m  1 . 
+) Nếu | X | 3 , gọi q là phần tử bé thứ ba của X (tức là m  n  q ). Khi đó tồn tại  X
sao cho q  m 2 .
Do q  nên hoặc  m hoặc  n.

Nếu  m thì q  m3  n , vô lý. Vậy  n  m3 và q  m 2


 m7 .
Nhưng tồn tại t  X sao cho q  nt 2 , do đó t  m2 . Mà m  m2  m3  m2 
 X , vô lý.


Vậy | X | 2 và X  m, m3 m  1 . 
Đề 3
Câu 1. a) Điều kiện: y  0, xy  2  0, xy  y  7  0 .
y  0 không thoả mãn hệ. y  0 chia hai vế phương trình thư nhất trong hệ cho y và
chia hai vê phương trình thứ hai trong hệ cho y ta được

 7  7
 x  3 y  y  8  3xy  6  x  3 y  y  8  3xy  6
 
 
 x  7  1  3xy  1  5  x  3 y  7  2  2 x  3 y  7  3 xy  21  1  25
 y  y y

 7
a  x  3 y  y
Đặt 
b  3xy  6

a  8  b

Hệ có dạng 
a  2  2 a  b  16  25

2

8 3
  a; b   11;3   x; y   1;1 ,  ; 
3 8
b) Nhận xét f  x   x 2  2 thoả mãn f 1  3, f  3  11, f  5  27

Xét đa thức Q  x   P  x   f  x  l| đa thức bậc 4 có các nghiệm là x  1, x  3, x  5

Nên Q  x    x  1 x  3 x  5 x  m 

Ta có P  2  Q  2  f  2  216  105m, P  6  Q  6  f  6   128  15m

Vậy P  2  7 P  6  216  105m  7 128  15m  1112

58
Page: Tài Liệu Môn Toán
Website: tailieumontoan.com

Câu 2.

PT   x 2  4  y 2  7   17  x 4   y 2  7  
2

 

 16 x 4  8 x 2  y 2  7    y 2  7   0  4 x 2   y 2  7   0
2 2

 4 x2  y 2  7  0   2 x  y  2 x  y   7

Do x, y nguyên dương nên 2 x  y  2 x  y và 2 x  y  0

2 x  y  7
Vậy    x; y    2;3
2 x  y  1
Vậy phương trình có nghiệm  x; y    2;3

Câu 3
1)
A

I O

B K
H L C

A'
M
N

1
Ta có OAC  900  AOC  900  ABC  BAH mà AI là phân giác góc A nên
2
HAI  OAI , suy ra tam giác ANA' cân tại A.
Gọi L l| giao điểm của MA và BC.
Ta có HKN  900  HNK  HAM  LAA ' , suy ra tứ giác ALA'K nội tiếp.
Do đó MA '.MK  ML.MA (1)
Dễ thấy ngay hai tam giác MCL và MAC đồng dạng, suy ra ML.MA  MC 2 (2)
Do I l| t}m đường tròn nội tiếp tam giác ABC nên MI  MC (3).
Từ (1), (2), (3) suy ra MN .MK  MI 2  NIK  900 . Vậy tứ giác NHIK nội tiếp.

59
Page: Tài Liệu Môn Toán
Website: tailieumontoan.com

T
l O

S K
B H L C

A'
M
N

2)
* Từ tứ giác NHIK nội tiếp suy ra IHK  INK  IA ' M  IAD . Suy ra tứ giác AIHS
nội tiếp. Do đó AIS  IHS  900 .
Gọi T l| trung điểm của cạnh SA. Khi đó TIA  TAI  INK  MIK , suy ra ba điểm
T , I , K thẳng hàng (4).
* Tiếp theo ta sẽ chứng minh L l| trung điểm của SK.
AI AB BL AB BL AB AB 1
Ta có  và      BL  AB
IL BL LC AC BC AB  AC 2BC 2
AI
Do đó  2 (5)
IL
Áp dụng định lý Menelaus cho tam giác ASL với cát tuyến TIK ta có:
TA KS IL
. .  1 (6). Từ (5) và (6) suy ra KS  2KL , tức L l| trung điểm của SK (7).
TS KL IA
Từ (4) và (7) suy ra I là trọng tâm tam giác AKS (đpcm).

Câu 4.
c2
Ta có 2ac  4a 2  1
4
d2
bd  b2   2
4

c  d 
2
cd
cd    3
8 2
Cộng vế 1 ,  2  ,  3 ta có
c 2 d 2 cd  c  d  3c  d 
2 2

P  2ac  bd  cd  4a  b   2
 2
   4a 2  b 2   8
4 4 2 8 8

60
Page: Tài Liệu Môn Toán
Website: tailieumontoan.com

1 
P  8   a; b; c; d    ;1; 2; 2  . Vậy giá trị lớp nhất cảu P bằng 8.
2 
Câu 5.
Đặt n  2014 . Giả sử các phần tử của M thoả mãn a1  a2  ...  an

Đặt S0  0, Sm  a1  a2  ...  am  0  m  n  .

Gọi P là tập tất cả những số si được x{c định trong đề bài.

Kí hiệu Pm  s  P | Sm1  s  Sm  với m  1, 2,3,...., n . Ta chứng minh cách chia P thành


các tập Pm như vậy thoả mãn điều kiện bài toán. Muốn vậy ta chỉ cần chứng minh
b  Pm thì Sm  2b
h
Thật vậy b  Sm1  a1  a2  ...  am1 và b   aik nên phai tồn tại ik để ik  m
k 1

Vậy b  aik  am  Sm  Sm1  Sm  b  2b  Sm

Đề 4
Câu 1.
x  0
a) Điều kiện: 
2  x  0
2
 x   2;0    0; 2 
1 1
Đặt y  2  x 2  0 . Thay v|o ta được:   2 . Do đó ta có hệ phương trình:
x y

 x2  y 2  2
  x2  y 2  2  x  y 2  2 xy  2

1 1  
x y  2  x  y  2 xy  x  y  2 xy


 x  y  2

 x  y    x  y   2  0
2
 xy  1
 
 x  y  2 xy   x  y  1

  xy  0,5

x  y  2  y  2  x x  1
+)   2 
 xy  1 x  2x 1  0  y  1

61
Page: Tài Liệu Môn Toán
Website: tailieumontoan.com

 1  3
 x 
 x  y  1  x  1  y  2
+)   2  (do y  0 )
 xy   0,5 2 y  2 y  1  0  3 1
 y  2


 1  3 
Vậy phương trình đã cho có tập nghiệm là S   ;1

 2 

b)
Phương trình x2  2mx  m2  2m  4  0 (1) có hai nghiệm không âm
  '  m 2  m 2  2m  4  0

  S  2m  0  m  2.
 P  m 2  2m  4  0

Theo định lý Vi-ét ta có x1  x2  2m; x1 x2  m2  2m  4 . Do đó

   m  1
2
x1  x2  x1  x2  x1  x2  2 x1 x2  2m  2 3
2

Do m  2  x1  x2  8 . Dấu đẳng thức xảy ra khi và chỉ khi m  2 .

Câu 2. Đặt z  y  1 , thay vào hệ ta được:

 x  z   3xz  1 
 x 2  xz  z 2  1   x  z   3  x  z   2  0
2 2

  
 x  xz  z  1  x  z  1  xz
  x  z  1  xz

 x  z  2
 x  z  2 
  xz  1
  x  z  1  

 xz  x  z  1   x  z  1
 
  xz  0

x  z  2 z  2  x x  1 x  1
+)   2  
 xz  1 x  2x 1  0 z  1  y  2
x  z  1 z  1 x  x  1, z  0  x  1, y  1
+)   2  
 xz  0  x  x  0  x  0, z  1  x  0, y  2
Vậy hệ phương trình có tập nghiệm là S  1;2  , 1;1 ,  0;2 

Câu 3.

62
Page: Tài Liệu Môn Toán
Website: tailieumontoan.com

Do a, b, c l| độ dài ba cạnh của một tam giác không nhọn nên có một trong các bất đẳng
thức sau xảy ra: a2  b2  c2 , b2  c2  a 2 , c2  a 2  b2 . Giả sử a 2  b2  c2 , khi đó ta có:

2  1 1 1 2 1 1  b2  c 2 1 1
 a 2
 b 2
 c   a2 b2 c2 
   1  a  2 
b c 
2 

a 2
  b2  c 2   2  2 
b c 
4 b2  c 2
 1 a . 2 2 
2
4
b c a2

3a 2 a2 b2  c 2 a 2 b2  c 2
 1    4  1  3  2 .  4  10 . Do đó
b2  c 2 b2  c 2 a2 b2  c 2 a 2
 1 1 1
a 2
 b2  c 2   2  2  2   10 .
a b c 
Câu 4.
a) Áp dụng quy tắc trọng tâm và quy tắc trung điểm ta có:
OA  OB  OC OB  OC
OG  , OM  . Khi đó
3 2


OG  OM  OG.OM  0  OA  OB  OC OB  OC  0  
 OAOB
.  OAOC
.  2OB.OC  2R2  0

 
2 2 2
a b  a b
  2 R 2  AB 2    2 R 2  AC 2   2 R 2  BC 2  2 R 2  0 (chú ý a.b 
1 1
)
2 2 2
 AB2  AC 2  2BC 2  12R2
b)
ab  c 2S 2S 2S
Kí hiệu a  BC , b  CA, c  AB, p  . Khi đó ta có a  ,b  ,c 
2 m n p

Theo công thức Hê – rông ta có:

S p  p  a  p  b  p  c 

1 1 1  1 1 1 1 1 1 1 1 1
 4S  2S     2S      2S     2S    
m n p  m n p m n p m n p
1
 4S  4S 2 .k  S  , trong đó
k

 1 1 1  1 1 1  1 1 1  1 1 1 
k               
 m n p  m n p  m n p  m n p 

63
Page: Tài Liệu Môn Toán
Website: tailieumontoan.com

2 2 2
Do đó a  ,b  ,c  .
mk nk pk
c)
Do BC vuông góc với đường cao kẻ từ A nên BC có dạng 2 x  y  c  0 . Tọa độ đỉnh B là
2 x  y  c  0  x  2
nghiệm của hệ    B  2;  c  4  ,
x  2  0  y  c  4
tọa độ C là nghiệm của hệ phương trình
2 x  y  c  0  x  c  3
   C  c  3; c  6  .
x  y  3  0 y  c  6
AB đi qua B  2;  c  4  và vuông góc với đường cao kẻ từ C nên

AB : 1.  x  2   1.  y  c  4   0  x  y  c  6  0 . Tọa độ đỉnh C là nghiệm của hệ

 x  y  c  6  0  x  2c  12
   A  2c  12; c  6  .
x  2 y  0 y  c  6
Theo giả thiết ta có
AB. AC.BC AB. AC.BC AB. AC
10   10    2 10
4S ABC 2.d  A, BC  .BC d  A, BC 

 2c  10    2c  10  . 3c  15
2 2
c  7
  2 10  c  5  2  
4c  24  c  6  c c  3
5
+) Nếu c  7  A  2; 1 , B  2;3 , C  4; 1 .

+) Nếu c  3  A  6;3 , B  2; 1 , C 0;3  không thỏa mãn ho|nh độ của A âm.

Vậy A  2; 1 , B  2;3 , C  4; 1 .


Câu 5. Giả sử min MAB, MBC, MCD, MDA  450 (1). 
cos MAB MA2  AB 2  MB 2
MA2  AB 2  MB 2
Ta có cot MAB    .
sin MAB 2.MA. AB.sin MAB 4SMAB

MA2  AB 2  MB 2
Kết hợp với (1) ta được  cot 450  1  MA2  AB 2  MB 2  4S MAB  2 
4SMAB

Tương tự ta được các bất đẳng thức sau đ}y :


MB2  BC 2  MC 2  4SMBC  3
64
Page: Tài Liệu Môn Toán
Website: tailieumontoan.com

MC 2  CD2  MD2  4SMCD  4 

MD2  DA2  MA2  4SMDA  5

Cộng theo vế các bất đẳng thức (2), (3), (4), (5) ta được:
AB2  BC 2  CD2  DA2  4  SMAB  SMBC  SMCD  SMDA   4S ABCD (6)

Mặt khác ta lại có: AB2  BC 2  CD2  DA2  2 AB.BC  2CD.DA  4S ABC  4SCDA  4S ABCD ,
mâu thuẫn với (6). Do đó giả sử ban đầu là sai suy ra tồn tại ít nhất một trong các góc
MAB, MBC, MCD, MDA có số đo không lớn hơn 450 .

Đề 5
Câu 1.
1.1 Điều kiện x, y  0

Đặt x  a  0, y  b  0; viết hệ đã cho về dạng

1 1
 a  2b   a  3b  3a  b 
2 2 2 2
(1)

 1  1  2  b4  a 4  (2)
 a 2b
2
(1)+(2) thu được  a 4  10a 2b2  5b4  a5  10a3b2  5ab 4  2 (3)
a
1
(2)-(1) thu được  5a 4  10a 2b2  b4  5a 4b  10a 2b3  b5  1 (4)
b
Từ (3) v| (4) thu được (a  b)5  3 và (a  b)5  1 .
5
3 1 5
3 1
Từ đó, tìm được a  và b  .
2 2
( 5 3  1) 2 ( 5 3  1) 2
V| do đó, tìm được x  ,y
4 4
1.2
Giả sử phương trình đã cho có ba nghiệm nguyên dương      . Khi đó, theo định lý

Vietta,       a,      b và   3a v| do đó      
3
(1)  3  3  3      3   3  3  9 (2). 2

65
Page: Tài Liệu Môn Toán
Website: tailieumontoan.com

 
Nếu   3 thì   3 và  3        , mâu thuẫn với (1). Vậy 1    3
3 3
Với   3: khi đó   3,  3  3 3  3  3.32  9    1   1  4. Từ đó
    3  a  9, b  27.
Với   2 :   2,  2  3 2  3  3.22  9   2  3 2  3  21. Giải phương trình
này với chú ý     2 ta được  ;   12;2  , 5;3  . Với
  12,   2  a  16, b  52 . Với   5,   3  a  10, b  31.
Với   1:   1,  2  3 2  3  3.12  9   2  3 2  3  12, vô lí

Vậy tất cả các cặp số  a; b   9;27  ,  16;52  ,  10;31 .

Câu 2. + Chứng minh được nhận xét: “Với a,b,x,y,z,t là các số nguyên sao cho a  b là
ước của x  y v| l| ước của z  t thì a  b | xz  yt ”
+ Mặt khác, do (a  c)2  (b  d )2  (a  b  c  d )(a  b  c  d ) (a  b  c  d ) nên suy ra
a  b  c  d | a 2  b2  c 2  d 2  2(ac  bd ) .
Từ đó, do giả thiết nên thu được a  b  c  d | ac  bd (1)

+ Ta sẽ chứng minh kết luận của bài toán bằng phương ph{p quy nạp toán học.
Với n  1, 2 : thì kết luận hiển nhiên đúng.
Giả sử khẳng định đúng tới n, tức là a  b  c  d | a n  bn  c n  d n với n  , n  2
Ta cần chứng minh a  b  c  d | a n1  bn1  c n1  d n1 (2)
Thật vậy, do a  b  c  d | (a  c)  (b  d ) và nhận xét ở trên suy ra a  b  c  d l| ước của
(a  c)(a n  c n )  (b  d )(bn  d n )  a n1  bn1  c n1  d n1  ac(a n1  c n1 )  bd (bn1  d n1 )

Nhưng, do (1), giả thiết quy nạp và nhận xét ở trên suy ra
a  b  c  d | ac(a n1  c n1 )  (bd (bn1  d n1 )

Vậy suy ra a  b  c  d l| ước của


(a  c)(a n  cn )  bd (bn  d n )  ac(a n1  cn1 )  bd (bn1  d n1 )  a n1  bn1  c n1  d n1

(2) được chứng minh.


Từ đó, theo nguyên lý quy nạp, suy ra a  b  c  d | a n  bn  c n  d n với mọi số nguyên
dương n.
Câu 3.
3.1
66
Page: Tài Liệu Môn Toán
Website: tailieumontoan.com

A
E

B C

Không mất tính tổng qu{t, xét trường hợp AB  BC  CA, c{c trường hợp khác xét
tương tự. Khi đó, E nằm trên đoạn CA, F nằm trên tia đối của tia AB, < (hình vẽ)
Từ giả thiết, suy ra F đối xứng với C qua phân giác trong của góc ABC . Do đó
ABC CAB  BCA ABC
CFA  CFB  900  và AIC  1800   900  . Suy ra tứ
2 2 2
giác AFCI nội tiếp.
BCA CAB
Từ đó AFI  ACI  và  IAC  IFC  ICF
2 2
BCA CAB CAB
Do EBA  BEC  CAB  (900  )  CAB   IBE 
2 2 2
Hơn nữa, do tính đối xứng nên IEB  IBE  900  MGC  MCG  ICG suy ra tứ
giác CIEG nội tiếp.
3.2
BCA
Do tứ giác CIEG nội tiếp, nên EGI  ECI   AFI
2
Hơn nữa, do IAB  IEB nên GEI  FAI suy ra GEI đồng dạng FAI
EG EG AF HG AF AI
Suy ra     
BI EI AI GE GE BI
BCA
Nhưng HGE  AEB  900   AIB suy ra HGE đồng dạng AIB
2
CAB
Từ đó EHG  BAI 
2

67
Page: Tài Liệu Môn Toán
Website: tailieumontoan.com

Câu 4.
Đặt f ( x)  x  g ( x) , phương trình h|m đã cho được viết lại về dạng
1 1
xg ( x  )  yg ( y)  yg ( y  )  xg ( x) x, y  0 (1)
y x
1
Cho y  1 thu được xg ( x  1)  g (1)  g (1  )  xg ( x) x  0 (2)
x
1
Trong (2), thay x bởi , ta được
x
1 1 1 1 1 1
g (  1)  g (1)  g (1  x)  g ( )  g (1  )  xg ( x  1)  g ( )  xg (1) x  0 (3)
x x x x x x
1
Từ (2) và (3) suy ra xg ( x)  g ( )  ( x  1) g (1) x  0 (4)
x
Trong (1), cho y  1 , bằng lập luận tương tự, cũng được
1
xg ( x)  g ( )   g (1)( x  1) n  0 (5)
x
b
Từ (4) và (5) suy ra 2 xg ( x)  ( g (1)  g (1)) x  ( g (1)  g (1) x  0 hay g ( x)  a  x  0 ,
x
b
ở đ}y a, b là hai hằng số. Suy ra f ( x)  a   x x  0
x
b
Thử lại ta thấy f ( x)  a   x x  0 thỏa mãn phương trình đã cho.
x
Câu 5.
5.1
Giả sử số dễ thương có hai chữ số lớn nhất là ab,1  a, b  9 . Theo giả thiết ta có
a 2  b2  c2 là số chính phương. Nếu a, b đều không chia hết cho 3 thì
a 2  b2  2  mod 3 , vô lý vì a 2  b2 là số chính phương suy ra ab  0  mod 3 .

+) Nếu a  9  81  b2  c2  c2  b2  81 không có nghiệm nguyên dương với 1  b  9

+) Nếu a  8  b 3  b  3;6;9 , thử trực tiếp ta thấy b  6 thỏa mãn. Vậy số dễ


thương lớn nhất có 2 chữ số là 86.
5.2

68
Page: Tài Liệu Môn Toán
Website: tailieumontoan.com

Xét số A  222211...1 . Khi đó 22  22  22  22  12  ...12  2025  452 suy ra


2009 so1 2009 so12

A  222211...1 là số dễ thương.
2009 so1

Đề 6
Câu 1.
1.1
2 2
 1 3  1 3
Ta có x  x  1   x    , x 2  x  1   x    nên phương trình x{c định với mọi
2

 2 4  2 4
x  . Phương trình đã cho tương đương với

x2  x  1  x2  x  1  2 x 2
 
 x  1 x2  x  1  4

 2 x2  2  2 x4  x2  1  4  x4  x2  1  1  x2

1  x  0
2
1  x  1
 4  4
 
2

x  x 1  1 x x  x 1  1  2x  x
2 2 2 2 4

1  x  1
  x  0 . Vậy pt có nghiệm duy nhất x  0.
x  0
1.2
Phương trình đã cho có hai nghiệm x1 , x2 thỏa mãn x1  x2  4

m  2
 '  0


m m  4  0

2
   2  m  0
   2  m  0  
 x1  x2  4 2  m  1  4
 
m  3
2  m  3

Theo định lí Viet ta có x1  x2  2  m  1 , x1 x2  m3   m  1 suy ra


2

P   x1  x2   8x1 x2  8  m  1  8m3  8  m  1  16m2  40m


3 3 2

Bảng biến thiên

69
Page: Tài Liệu Môn Toán
Website: tailieumontoan.com

m -2 0 2 3
0 16

-144 -24

Từ bảng biến thiên ta được: Pmax  16 khi m  2 , Pmin  144 khi m  2 .

Câu 2.

 x  x y  xy  xy  y  1 
( x  y )  xy ( x  y )  xy  1
2 2
 2 3 2

Ta có  4  2
 
2
 x  y  xy (2 x  1)  1
   xy  1
2

 x y

a  x 2  y a  ab  b  1
Đặt  . Hệ trở thành:  2 (*)
b  xy a  b  1

 a  a  2a  0
3 2

a(a  a  2)  0
2

Hệ (*)   
b  1  a
 b  1  a

2 2

Từ đó tìm ra (a; b) (0; 1); (1; 0); (2;  3)

 x2  y  0
* Với (a; b)  (0; 1) ta có hệ   x  y  1.
 xy  1

 x2  y  1
* Với (a; b)  (1; 0) ta có hệ   ( x; y )  (0; 1);(1;0);(1;0) .
 xy  0
* Với (a; b)  ( 2; 3) ta có hệ
 3  3
 x 2  y  2 y   y  
  x  x  x  1; y  3 .
 xy   3 x  2x  3  0
3 ( x  1)( x  x  3)  0
2
 
Kết luận: Hệ có 5 nghiệm ( x; y) (1; 1);(0; 1);(1; 0);( 1; 0);( 1; 3)  .

Câu 3.
t 2 1
Đặt t  x  1  x2 thì dễ thấy t  0 và x  (1)
2t
2012 20122  t 2
Từ giả thiết ta có y  1  y 2  . Từ đ}y cũng suy ra y  (2)
t 2.2012.t

70
Page: Tài Liệu Môn Toán
Website: tailieumontoan.com

t 2  1 20122  t 2 2011  2012 


Từ (1) và (2) suy ra x  y    t  
2t 2.2012.t 2.2012  t 

2011 2012 2011 2011


Do đó x  y  .2 t.  .2 2012  .
2.2012 t 2.2012 2012
2011
Đẳng thức xảy ra khi và chỉ khi t  2012 . Từ (1) và (2) suy ra x  y 
2 2012
2011 2011
Vậy giá trị nhỏ nhất của P bằng , khi x  y  .
2012 2 2012
Câu 4.
4.1

P N

H
O

B K C

M
D

Kẻ đường kính AD, khi đó tứ giác BHCD l| hình bình h|nh nên trung điểm K của BC
cũng l| trung điểm của HD, trong tam giác AHD có OK l| đường trung bình nên
2OK  AH  OB  OC  OH  OA  OA  OB  OC  OH
Ta có OB  OC  2OK  OM v| c{c đẳng thức tương tự ta được:


OM  ON  OP  2 OA  OB  OC  2OH 
 3OL  2OH suy ra O, H, L thẳng hàng.
4.2
1 AB 2  MA2  MB 2
Trước hết ta có các kết quả sau: S ABCD  AC.BD.sin  ; cot  
2 4SMAB

Tương tự ta được:

71
Page: Tài Liệu Môn Toán
Website: tailieumontoan.com

AB 2  MA2  MB 2 BC 2  MB 2  MC 2 CD 2  MC 2  MD2
cot    
4SMAB 4SMBC 4SMCD

DA2  MD 2  MA2 AB 2  BC 2  CD 2  DA2


 
4S MDA 4  S MAB  S MBC  S MCD  S MDA 
AB 2  BC 2  CD 2  DA2 AB 2  BC 2  CD 2  DA2
 
4S ABCD 2 AC.BD.sin 

4.3

K
I

B C

Đường tròn ngoại tiếp tam giác ABC l| đường tròn đi qua 3 điểm M, N, P nên ta lập
được phương trình n|y l|: x2  y 2  3x  29  0 suy ra tâm K của đường tròn ngoại tiếp
 3 
tam giác ABC có tọa độ là K   ; 0  .
 2 
5
Do AB  KP nên AB có vtpt nAB  KP    2; 1 . Suy ra phương trình
2
AB : 2  x  1  1 y  1  0  2 x  y  3  0 . Do đó tọa độ A, B là nghiệm của hệ phương
2 x  y  3  0  y  2x  3  x  1, y  5
trình  2     x  4, y  5
 x  y  3x  29  0  x  3x  4  0
2 2

5
Suy ra A 1;5 , B  4; 5 . Do AC  KN nên AC có vtpt là nAC  KN   2;1
2
Suy ra pt AC : 2  x  1  y  5  0  2 x  y  7  0 . Khi đó tọa độ A, C là nghiệm của hệ
phương trình:
2 x  y  7  0  y  2 x  7  x  1, y  5
 2     x  4, y  1 . Từ đ}y suy ra C  4; 1 . Vậy
 x  y 2
 3 x  29  0  x 2
 5 x  4  0 
A 1;5 , B  4; 5 , C  4; 1 .

Đề 7
72
Page: Tài Liệu Môn Toán
Website: tailieumontoan.com

Câu 1.
1.a
Đặt S  x  y; P  xy . Khi đó hệ phương trình trở thành

S  m  2 S  m  2
 2  
 S  2 P  2S  m  4 P  m  m  2
2 2

 
Để hệ có nghiệm thì S 2  4P   m  2   4 m2  m  2  m2  4  2  m  2
2

1.b
Ta có A  P  2S  2011  m2  m  2005
Lập bảng biến thiên ta được max A  2011 khi m  2 ; min A  2004,75 khi m  0,5

m 1
-2 - 2
2
2007 2011

2004,75

1.2
Đặt t  x2  0 , thay v|o phương trình ta được t 2   3m  1 t  6m  2  0

t  2
 phương trình đã cho có bốn nghiệm phân biệt khi
t  3m  1
 1
3m  1  0 m 
  3 . Khi đó phương trình đã cho có bốn nghiệm là  2;  3m  1
3m  1  2 m  1

10
Để các nghiệm đều lớn hơn 3 thì  3m  1  3  3m  1  3  m  . Vậy các giá
3
 1 10 
trị của m là m   ;  \ 1
3 3 
Câu 2.
ĐK xy  0 , ta thấy từ pt thứ nhất  x  y  0 , do đó x  0, y  0 . Từ đó ta đặt

u  x  0, v  y  0 thay vào hệ ta được

73
Page: Tài Liệu Môn Toán
Website: tailieumontoan.com


u  v  uv  1  u  v   1  3uv

2 2 2

 4 

 u  3  v 4
 3  4 
u  v  6  2 3u  3v  u v  9  16
4 4 4 4 4 4

 u  v 2  1  3uv

 2 2
  u  v   2uv   2u 2v 2  2 u 4v 4  3  u  v   2uv   6u 2v 2  9  10
2 2

   

Đặt t  uv  0  t  1 (vì 1  3uv   u  v   4uv  uv  1 ). Thế từ phương trình thứ nhất


2

của hệ trên v|o phương trình thứ hai ta được


    
2
4 t  3t  6t  12  t  2t  9
4 2 2
2 t  3t  6t  12  t  2t  9  
4 2 2


t  2t  9  0
2

 3t 4  4t 3  34t 2  60t  33  0   t  1  3t 3  7t 2  27t  33  0 .

u  v  2 u  1  x  1
+) Nếu t  1  uv  1 ta có   
uv  1 v  1 y  1
+) Nếu 3t 3  7t 2  27t  33  0  3t 3  7t 2  6  27 1  t   0 vô lí vì 0  t  1

Kết luận nghiệm của hệ là  x; y   1;1

Câu 3.
Do x, y  0 nên bất đẳng thức đã cho tương đương với

1  x 2  1  y 2  1  xy   1  x 2 1  y 2
 
  2  2 x  2 y  x 2  y 2  1  xy   1  2 x  x 2 1  2 y  y 2 

 xy  x  y    xy  1  0 , bất đẳng thức n|y luôn đúng. Dấu bằng xảy ra khi
2 2

x  y  1.

Câu 4.
4.1
Giả sử tọa độ của M  x;0  . Khi đó MA  1  x; 2  ; MB   4  x;3 .

Theo giả thiết ta có MA.MB  MA.MB.cos 450

74
Page: Tài Liệu Môn Toán
Website: tailieumontoan.com

2
 1  x  4  x   6  1  x  4  x
2
 4.
2
 9.
2
2
 x 2  5 x  10  x 2  2 x  5. x 2  8 x  25.
2

 2  x 2  5 x  10    x 2  2 x  5  x 2  8 x  25  (do x 2  5 x  10  0)
2

 x 4  10 x3  44 x 2  110 x  75  0
  x  1 x  5   x 2  4 x  15   0  x  1; x  5

Vậy ta có hai điểm cần tìm là M 1;0  hoặc M  5;0  .

4.2 Gọi A’, B’, C’ lần lượt l| ch}n đường cao hạ từ c{c đỉnh A, B, C. Do tứ giác BCB’C’
nội tiếp nên FDA  FCA  ABE  ADE  H nằm trên đường phân giác trong hạ từ D
của tam giác DEF, tương tự ta cũng chỉ ra được H nằm trên đường phân giác trong hạ
từ đỉnh E của tam giác DEF. Vậy H l| t}m đường tròn nội tiếp của tam giác DEF.
Ta lập được phương trình c{c đường thẳng DE, DF lần lượt là
DE : 3x  y  5  0; DF : 3x  y  7  0 . Do đó phương trình ph}n gi{c trong và ngoài của
3x  y  5 3x  y  7
đỉnh D là   x  2  0; y  1  0 . Kiểm tra vị trí tương đối của E, F
10 10
với hai đường trên ta được phân giác trong kẻ từ đỉnh D là
d : x  2  0 . Tương tự ta lập được phương trình ph}n gi{c trong kẻ từ đỉnh E là
d ' : x  y  1  0 . Mặt khác H là giao của d và d’ nên H  2;3

5 7
Ta có AC là trung trực của HE nên AC đi qua trung điểm B '  ;  và có vtpt là
2 2
HE  1;1  AC : x  y  6  0

A E

B'
F C'
H
A'
B C

75
Page: Tài Liệu Môn Toán
Website: tailieumontoan.com

4.3 Gọi M là tiếp điểm của AC với đường tròn nội tiếp tam giác ABC. Khi đó ta có
AM  p  a; IM  r . Gọi S là diện tích tam giác ABC, theo công thức Heron ta có
S p  p  a  p  b  p  c  . Áp dụng định lí Pitago trong tam giác AIM ta có

S
2
 p  a  p  b  p  c 
IA  AM  MI   p  a   r   p  a       p  a 
2 2 2 2 2 2 2

 p p

 p  a  bc IA2 b IB 2 c IC 2 a
   . Tương tự ta có  ; 
p c  p  a p a  p  b p b  p  c p

IA2 IB 2 IC 2 abc
Do vậy    2
c  p  a a  p  b b  p  c p

I
C

Đề 8
Câu 1.
1.1
3x  0
+) Nếu y  0 thay vào hệ ta có  2 hệ này vô nghiệm
x  3  0
+) Nếu y  0 thì ta đặt x  ty thay vào hệ ta được


ty  y  3ty  6 y  0
3 3

2 2
t y  ty  3  0

2


 y ty 2  y 2  3t  6  0
 ty 2  y 2  3t  6  0

  2
 
3
   y  2
2 2
 t t y 3
 t t

76
Page: Tài Liệu Môn Toán
Website: tailieumontoan.com

 3 3
 t  1  3t  6  0  3t  6  0 t 2  2t  1  0 t  1
 t  t  1  t  
   2 3  2 3
 y2  3  y2  3 y  2  y  2
   t  t
 t2  t  t t
2

x  y
 3
 3  x y .
y   2
 2
 3 3  3 3
Vậy hệ phương trình có nghiệm là  x; y    ;  ;   ;  
 2 2  2 2 

1.2
1
ĐK x  với điều kiện n|y phương trình được đưa về dạng
2

18 x  16  4  x  3 2 x  1  7  2 x  2  2 x  1  7  2 x  2  x  3
   
x  3  2 x 1  6  2 x  2   0
2
2 x  3  2x 1  7 2x  2

Đặt a  x  3  2 x  1; b  2 x  2 thay v|o phương trình trên ta được


2a 2  7ab  6b2  0   2a  3b  a  2b   0  2a  3b; a  2b

+) a  2b  x  3  2 x  1  2 2 x  2 phương trình n|y vô nghiệm

+) 2a  3b  2 x  3  2 2 x  1  3 2 x  2 giải phương trình n|y được nghiệm x  1 . Vậy


nghiệm của phương trình đã cho l| x  1 .
Câu 2.
Giả sử tìm được bộ ba số  m; n; p  trong đó m, n, p là các số hữu tỉ dương sao cho có
các số nguyên dương a, b, c thỏa mãn
1 1 1
a  m ;b  n  ;c  p 
np pm mn

Từ đó mnp  1  anp  bpm  cmn . Suy ra abc  mnp    mnp  1


2 3

u
Đặt mnp  trong đó u, v  
,  u; v   1 ta được
v
3
u2  u 
abc  2    1  abcu 2v   u  v 
3
(1)
v v 

77
Page: Tài Liệu Môn Toán
Website: tailieumontoan.com

Do  u; v   1 nên nếu p là một số nguyên tố sao cho p | u 2v thì hoặc p | u hoặc p | v do


đó u  v không chia hết cho p . Do đó

u  v  abc   u  v 

3 3

(1)  abc  
u 2v u v  1

2

Suy ra u  v  1, abc  8, mnp  1. Từ đó tìm được  a; b; c   1;1;8  , 1;2;4 , 2;2;2  và các


1 1  1 
hoán vị và vì vậy  m; n; p   1;1;1 ,  ; ; 4  ,  ;1; 2  và các hoán vị.
2 2  2 
Câu 3.
3.1
Ta chứng minh bằng phản chứng. Giả sử không tồn tại số tự nhiên n nào thỏa mãn thì
với mọi số tự nhiên n ta luôn có
a n3 bn3 c n3 2011 a n 2 b n 2 c n 2
     n  n
bn1 c n1 a n1 2010 bn c a
Lần lượt cho n  0,1, 2,..., 2009 và cộng từng vế của 2010 bất đẳng thức ta được
a 2012 b2012 c2012 2011 2 2 2
2010
 2010  2010  2010.  a  b  c  2011 . Mâu thuẫn với giả thiết nên ta có
b c a 2010
đpcm.
3.2
a m2
Áp dụng bđt AM – GM cho 2 số và m số b 2 ta có
bm

a m 2 a 2 m 2 .b2 m
2 m  mb   m  2 
2 m  2
2m
  m  2 a2
b b
Tương tự ta được

bm 2 b2 m 2 .c 2 m
2 m  mc   m  2 
2 m  2
2m
  m  2  b2
c c

c m 2 c 2 m 2 .a 2 m
2 m  ma   m  2 
2 m  2
2m
  m  2 c2
a a
Cộng từng vế c{c bđt trên ta được
a m 2 bm 2 c m 2
m
 m  m  a 2  b2  c 2 (1)
b c a

78
Page: Tài Liệu Môn Toán
Website: tailieumontoan.com

a m2
Áp dụng bđt AM – GM cho m số m
và a 2 ta được
b

a m 3 a m m3 .a 2 a m 2
m m1  a   m  1
2 m 1   m  1 m . Tương tự ta có
b bm m1 b

b m 3 bm m3 .b 2 b m 2
m m1  b   m  1
2 m 1   m  1
c c m m1 cm

c m 3 c m m3 .c 2 c m 2
m  c 2
  m  1 m 1   m  1
a m1 a m m1 am

Cộng từng vế của c{c bđt trên ta được


 a m 3 b m 3 c m 3   a m 2 bm 2 c m 2 
m  m 1  m 1  m 1   m  m  m  m 
b c a   b c a 
m2 m2 m2
a b c
 m  m  m  a 2  b2  c2
b c a
Kết hơp với (1) ta có đpcm. Dấu đẳng thức xảy ra  a  b  c .
Câu 4.
4.1
+) Do các tứ giác BFHD, DHEC và CBFE nội tiếp nên

FDH  FBH  FBE  FCE  HCE  HDE

Suy ra DH là phân giác của góc EDF . Tương tự cũng được EH là phân giác của góc
DEF và FH là phân giác của góc EFD. Từ đó H l| t}m đường tròn nội tiếp của tam
giác DEF .
a a.sin A
+) Do MBT  MCT  BAC; MB  MC   d  M ; BT   d  M ; CT  
2 2
+) Ta có

MEF  HEF  HEM  HAB  HEM  HAB  HBM  900  B  900  C  A và


BC a a.sin A
ME    d  M ; EF   .
2 2 2
a
Do đó d  M ; TB   d  M ; TC   d  M ; EF    sin A nên M l| t}m đường tròn nội tiếp tam
2
giác XTY .
4.2

79
Page: Tài Liệu Môn Toán
Website: tailieumontoan.com

+) Do tứ giác AFDC nội tiếp và TX tiếp xúc với  O  nên

FDB  FAC  BAC  CBT  DBT


Suy ra TX || DF . Tương tự cũng có TY || DE.
DF
+) Từ đó, với k  thì phép vị tự tâm S tỷ số k biến tam giác DEF thành tam giác
TX
TYX . V| do đó biến H (t}m đường tròn nội tiếp của tam giác DEF ) thành M (tâm
đường tròn nội tiếp của tam giác TYX ) suy ra S , H , M thẳng hàng.

Y
A

E
F S
X H O

C
B D M

Câu 5.

Đặt f  2   a . Cho m  n  0  f  0   3  f  0    f  0   0 .
2

Cho m  1; n  0  f 1   f 1   f 1  1 . Cho m  n  1  f  3  3.


2

 
Cho n  0  f m2   f  m   , m  nên f  4   a 2 .
2

Mặt khác với mỗi số tự nhiên

k  3   k  1  2  k  2    k  3  2k 2
2 2 2

  f  k  1   2  f  k  2     f  k  3   2  f  k   1
2 2 2 2

Từ (1) cho k  3 ta có

 f  4  2  f 1    f  0    2  f  3   a 4  16  a  2  f  2   2 .
2 2 2 2

Theo trên ta chứng minh được f  n   n với n  0; 1; 2; 3; 4 . Ta chứng minh bằng quy
nạp f  n   n . Thật vậy, với n  3 từ đẳng thức (1) ta có:

80
Page: Tài Liệu Môn Toán
Website: tailieumontoan.com

 f  n  1  2  f  n  2    f  n  3   2  f  n 
2 2 2 2

  f  n  1    n  3  2n  2  n  2    n  1  f  n  1  n  1
2 2 2 2 2

Do đó f  n   n, n   f  2011  2011.

Đề 9
Câu 1.

a. Cho parabol (P): y   x  4 x  5 v| điểm I (1;4) . Tìm trên (P) hai điểm M, N đối
2

xứng nhau qua điểm I


đường thẳng  qua I v| có hsg k có phương trình y  k ( x  1)  4

Xét pt  x  4 x  5  k ( x  1)  4  x  (k  4) x  k  1  0 (1)
2 2

  (k  4)2  4(k  1)  0  k 2  4k  20  0, k   cắt (P) tại M và N


Gọi 2 nghiệm của (1) là x1 , x2  M ( x1; k ( x1  1)  4), N ( x2 ; k ( x2  1)  4)

M, N đối xứng nhau qua điểm I  I l| trung điểm của MN


 x1  x2
 2  1 4k
  1 k  2
 k ( x1  1)  4  k ( x2  1)  4  4 2
 2

Khi đó (1) x  2 x  3  0  x  1 hoặc x  3 . Vậy M (1;0), N (3;8)


2

b. Tìm m để phương trình x 2  2  m4  m2 có 4 nghiệm phân biệt

Điều kiện cần m4  m2  0  m  1 hoặc m  1 (1)


 x 2  2  m4  m2  x 2  2  m4  m2
Khi đó  2   2
 x  2  ( m  m )  x  2  (m  m )
4 2 4 2

Điều kiện đủ 2  (m4  m2 )  0  1  m2  2

Kết hợp với ĐK (1) ta được 1  m  2 hoặc  2  m  1

Cách khác. Pt có 4 nghiệm  đường thẳng y  m4  m2 cắt đths y  x 2  2 tại 4 điểm.


Từ đồ thị suy ra 0  m4  m2  2  1 | m | 2
Câu 2.
a. ĐK : x  2 .

81
Page: Tài Liệu Môn Toán
Website: tailieumontoan.com

BPT  ( x  1)  
x  2  2  ( x  6)  
x  7  3  x2  2 x  8

x2 x2
 ( x  1)  ( x  6)  ( x  2)( x  4)
x22 x7 3
 x 1 x6 
 ( x  2)    ( x  4)   0
 x22 x7 3 
x 1 x6
Ta có   ( x  4)
x22 x7 3
x2 x2 x6 x6 1
    
x22 2 x7 3 2 x2 2
( x  2) x  2 ( x  6)( x  7  1) 1
    0, x  2
x22 x7 3 x22
BPT  x  2  0  x  2
Vậy tập nghiệm của BPT là S   2;2

b. Trừ vế ta được  x  y  x  y  2xy  7   0

TH 1. x  y . Thế vào pt thứ nhất ta được

x  2
x2  5x  6  0  
x  3
TH 2. x  y  2 xy  7  0  2 xy  x  y  7

Cộng hai pt theo vế ta được


5  x  y    x 2  y 2   12  0  5  x  y    x  y   2 xy  12  0
2

x  y 1
  x  y   6 x  y   5  0  
2

x  y  5
x  y  1  xy  4 (Loại)
 x  2, y  3
x  y  5  xy  6  
 x  3, y  2
Vậy hệ có 4 nghiệm là  2;2  ,  3;3 ,  2;3 ,  3;2 

c. ĐK: x  1 . Chia hai vế cho x  1 ta được

82
Page: Tài Liệu Môn Toán
Website: tailieumontoan.com

x 1 x 1
3  m  24
x 1 x 1
x 1
Đặt t  4 ,0  t  1 ta được 3t 2  m  2t  3t 2  2t  m (2)
x 1
Pt (1) có nghiệm x  1  pt (2) có nghiệm t   0;1

Lập bảng biến thiên của f  t   3t  2t trên  0;1


2

1
Từ BBT suy ra pt (2) có nghiệm t   0;1  1  m 
3
Câu 3.
a. Gọi M l| trung điểm của BC ta có:
2 1 1
AG  AM  AB  AC
3 3 3

DE  DA  AE  2 AB 
2
5
2

AC  5 AB  AC
5
 (1)

1 1 5 1 1
DG  DA  AG  2 AB  AB  AC   AB  AC  5 AB  AC
3 3 3 3 3
  (2)

6
Từ (1) và (2) suy ra DE  DG  D, E, G thẳng hàng
5

b. Ta có MH .MA 
1
2

BA  CA MH  A

H
1

 BA.MH  CA.MH
2


1
2   
BA MC  CH  CA MB  BH 
   B A' M C


1
2
BA.MC  BA.CH  CA.MB  CA.BH 
Vì BA  CH  BACH
.  0; CA  BH  CA.BH  0
1 1
 MH .MA  BA.MC  CA.MB
2 2
Mặt khác ta có BA.MC  BA '.MC; CA.MB  CA '.MB và MB  MC

83
Page: Tài Liệu Môn Toán
Website: tailieumontoan.com

Nên MH .MA 
1
2
1 1
BA '.MC  CA '.MC  MC BA '  CA '
2 2
 
1 1 1 1
 MC.BC  . BC.BC  BC 2 (đpcm)
2 2 2 4
c.

Chứng minh được HM  ME từ đó suy ra E (5;1)

Chứng minh được HG  2GF từ đó suy ra F (3;5)


Giả sử B( x; y) . Từ giả thiết suy ra B, E, F thẳng hàng và BE  BH
Tìm được tọa độ B(1;3)
Câu 4.
x 2  2 xy  2 y 2
Thế x 2  y 2  1 v|o S ta được S 
xy  x 2  y 2
TH 1. y  0  x2  1  S  1
2
x x
 y 2 y 2
t 2  2t  2
TH2. y  0  S   
x
. Đặt t   S 
x x
2
y t2  t 1
 1
y  y 

 S (t 2  t  1)  t 2  2t  2  (S  1)t 2  (S  2)t  S  2  0

Với S  1 , tồn tại t    (S  2) 2  4( S  1)( S  2)  0


Biến đổi ta được (S  2)(3S  6)  0  2  S  2

Do S  1  2; 2 nên max S  2, min S  2

Câu 5.

84
Page: Tài Liệu Môn Toán
Website: tailieumontoan.com

A  (1  x)2  y 2  ( x  1)2  y 2  y  2  (1  x  x  1)2  ( y  y)2  y  2

Vậy A  4  4 y2  y  2 .

TH 1. y  2  A  2 1  y  2 5
2

TH 2. y  2  A  2 1  y  2  y
2

     12  12  y 2   2  y  3.1  1. y  2  y  3  2
2
3
 
1
A  2  3 khi và chỉ khi x  0, y 
3
Ta có 2  3  2 5  min A  2  3

Đề 10
5 10
Câu 1. *  (1). Điều kiện: x  2, x  1 (*).
x2 x 1
1 4
* (1)  
( x  2) 2
( x  1)2
 ( x  1)2  4( x  2)2 (do ( x  1)2 .( x  2)2  0 với mọi x thỏa mãn (*))
 3x 2  18x  15  0
 x  (; 5]  [  1; ) .
* Kết hợp với điều kiện có tập nghiệm của (1) là S  (; 5]  [  1;1)  (1; ) .

CHÚ Ý: Nếu thí sinh không nêu điều kiện (*) mà biến đổi quy đồng bỏ mẫu thì không cho điểm
của câu kể từ bước biến đổi đó.

Câu 2. (2):  x  4  x  1  3 x  5 x  2  6 hay x 2  5 x  3 x 2  5 x  2  2  0


2

* Đặt t  x 2  5 x  2 , điều kiện t  0 .


Ta có x 2  5 x  t 2  2 , phương trình cho trở thành t 2  3t  4  0
hay t=-1 hoặc t=4.

* Kết hợp điều kiện có t=4 hay x2  5x  2  4

85
Page: Tài Liệu Môn Toán
Website: tailieumontoan.com

Giải được x=-7, x=2.


* Kết luận: (2) có tập nghiệp là {-7; 2}.

2 x  4 xy  2 y  3 x  3y  2  0 (3)
2 2

Câu 3.  2
 x  y  2 y(2 x  1)  0

2
(4)

* (3)  2( x  y )2  3( x  y )  2  0
 x  y  2

x  y  1
 2
* Với x  y  2 , thay y  2  x vào (4) có:

x 2  (2  x)2  2(2  x)(2 x  1)  0  2 x 2  6 x  0  x 3;0 .

x=-3 thì y=1, với x=0 thì y=-2.

1 1
* Với x  y  , thay y   x vào (4) có:
2 2

1 1 5  1  11 1  11 
x 2  (  x )2  2(  x )(2 x  1)  0   x  2 x 2  0  x   ; 
2 2 4  4 4 

1  11 3  11 1  11 3  11
x thì y  ; x thì y 
4 4 4 4
* Kết luận: Hệ cho có 4 nghiệm (x; y) là:

1  11 3  11 1  11 3  11
(-3; 1), (0 ; -2), ( ; ), ( ; )
4 4 4 4
Câu 4.

2 x  4  0
* x 2  2 x  m2  2 x  4   2
 x  2 x  m  (2 x  4)
2 2

 x  2
 2 (*)
3 x  18 x  16 - m  0 (5)
2

* (5) l| phương trình bậc hai có  '  3m2  33  0, m .

9  3m2  33 9  3m2  33
(5) có hai nghiệm là x1  , x2  .
3 3

86
Page: Tài Liệu Môn Toán
Website: tailieumontoan.com

33  9
* Nhận xét: x2   2, m nên hệ (*) luôn có nghiệm với mọi m.
3
Vậy phương trình cho luôn có nghiệm với mọi giá trị của m.

CHÚ Ý : -Thí sinh có thể trình bày theo phương pháp đồ thị hoặc sử dụng bảng biến thiên của
hàm số bậc hai trên [-2; +∞).
-Nếu thí sinh dùng phép biến đổi bình phương hai vế mà không nêu được điều kiện
2 x  4  0 thì không cho điểm câu này.
Câu 5.

x 2  mx  m  1  0 (6)
Xét tam thức bậc hai f ( x )  x 2  mx  m  1 có hệ số bậc hai a=1>0, biệt số   m2  4m  4 .
* Trường hợp 1:   0 , khi đó f ( x )  0, x 
(6) có tập nghiệm là S   : không thỏa mãn yêu cầu.

m
* Trường hợp 2:   0 , khi đó f ( x )  0, x  ; f ( x)  0  x   .
2
Tập nghiệm của (6) chỉ có một phần tử: không thỏa mãn yêu cầu.

* Trường hợp 3:   0 .

m   m  
 f ( x ) có hai nghiệm phân biệt là x1  , x2  .
2 2
 Tập nghiệm của (6) là S   x1 ; x2  .

 Biểu diễn của S trên trục số l| đoạn có độ dài bằng 1 khi và chỉ khi
x2  x1  1    1 (thỏa mãn điều kiện   0 )

 m2  4m  4  1  m 1;5 .

* Kết luận: m  1;5 .

LƯU Ý: Nếu thí sinh làm theo cách trên nhưng chỉ nêu được trường hợp 3 và giải đúng giá trị
m thì chỉ cho 1điểm.
Câu 6.
87
Page: Tài Liệu Môn Toán
Website: tailieumontoan.com

Chứng minh 4S(cot A  cot B  cot C)  a2  b2  c2 .

b2  c 2  a2
* Chứng minh được hệ thức cot A  (7).
4S
c 2  a2  b2 a2  b2  c 2
* Tương tự có cot B  (8), cot C  (9).
4S 4S
Cộng theo vế c{c đẳng thức (7), (8), (9) ta được:
a2  b2  c 2
cot A  cot B  cot C  hay 4S(cot A  cot B  cot C)  a2  b2  c2 .
4S
Câu 7.
7.1 Xét d1: x+y-6=0 và d2: x+2y-5=0.
1.1  1.2 3
* cos  cos(d1 , d2 )  
12  12 . 12  22 10
*  là góc giữa hai đường thẳng nên 00    90o , sin   0
1
do đó sin   1  cos2  .
10
9 1
  10
3 cos   sin   10
* Vậy m   10 10  4.
2 cos   sin  6

1
10 10
7.2 * I thuộc đường thẳng d1: x+y-6=0 v| có ho|nh độ bằng 2  I  (2;4) .
* Gọi M, N là c{c giao điểm của d2 với (C), H là hình chiếu vuông góc của I lên MN thì :
2  2.4  5
 IH  d ( I ; d2 )   5.
12  22
1
 H l| trung điểm của MN nên HM  MN  1 .
2

 Tam giác IHM vuông tại H có IM  IH 2  HM 2  6 .

* Đường tròn (C) có t}m l| I v| b{n kính l| IM nên có phương trình :


( x  2)2  ( y  4)2  6
7.3 * B l| giao điểm của d1 và d2 nên tìm được B=(7; -1).
* Đặt u(a; b) là một vectơ chỉ phương của đường thẳng AC ( a2  b2  0 ).

Đường thẳng BC hay d2 có vectơ chỉ phương v  (2; 1) .


88
Page: Tài Liệu Môn Toán
Website: tailieumontoan.com

* Tam giác ABC cân tại A nên: góc giữa đường thẳng AB và BC bằng góc giữa đường
thẳng AC và BC
3 2a  b
 cos   cos(u, v)  
10 12  22 . a2  b2

 3 a2  b2  2 2a  b  9(a2  b2 )  2(2a  b)2  a2  8ab  7b2  0

 a  b hoặc a  7b .
* Với a=-b, chọn a=1, b=-1, khi đó u(1; 1) nên đường AC v| đường BC cùng vectơ chỉ
phương : Điều này không thể xảy ra.
* Với a=-7b, chọn b=-1, a=7, u(7; 1) (t/mãn đkiện AC và AB cắt nhau).
Đường thẳng chứa đường cao kẻ từ B của tam gi{c ABC l| đường thẳng đi qua B v|
vuông góc với AC nên nhận u(7; 1) l|m vectơ ph{p tuyến, do đó nó có phương trình
là : 7(x-7)-1(y+1)=0 hay 7x-y-50=0.
* Kluận: Phương trình cần tìm là 7x-y-50=0.
Câu 8. * Áp dụng bất đẳng thức Cauchy cho ba số ta có:
( a 2  7 b)  8  8
3
a2  7b .2.2  ,
3
(b2  7c)  8  8
3
b2  7c .2.2  ,
3
(c 2  7 a )  8  8
3
c  7a .2.2 
2
.
3
a2  b2  c 2  7(a  b  c)  48
Cộng theo vế các bất đẳng thức trên có: 4S 
3
7(a  b  c)  51
hay 4S  (10).
3
a2  1 b2  1 c2  1
* Lại có: a  a.1  , b , c
2 2 2
a2  b2  c 2  3
nên a  b  c  hay a  b  c  3 (11).
2
* Từ (10), (11) suy ra 4S  24 hay S  6 .
 3 a2  7b  3 b2  7c  3 c2  7a  2
Dấu đẳng thức xảy ra khi và chỉ khi  hay a=b=c=1.
a  b  c  1

89
Page: Tài Liệu Môn Toán
Website: tailieumontoan.com

* Kluận: Giá trị lớn nhất của S l| 6, đạt được khi a=b=c=1.
Đề 11

 x 2  m (2)
Câu 1. Phương trình tương đương với 
 x  (1;5)
TH1: m  0 phương trình vô nghiệm

TH2: m  0 (2) có nghiệm x  m , x m


 1  m  5  m  25
phương trình đã cho có nghiệm khi  
 1   m  5  m  1
KL: 0  m  25 phương trình đã cho có nghiệm
Câu 2.
a. Điều kiện 32  x  2

Phương trình đã cho tđ với: 8 x  30 x  64  x  30 x  64  9


2 2

 x 2  30 x  64  1 (VN )

 x 2  30 x  64  9

 x 2  30 x  145  0
 x  15  370(tm)

 x  15  370(tm)
3
b. Điều kiện x 
2
x 3  2 x  1  0  x 3  2 x  1 (2)
3
+) 0  x  là nghiệm
2
+) x  0 thì (2)  x (3  2 x)  1
2

 2 x3  3x 2  1  0
 ( x  1)(2 x 2  x  1)  0
 ( x  1)( x  1)(2 x  1)  0

90
Page: Tài Liệu Môn Toán
Website: tailieumontoan.com

 1
x  
 2
 x  1

 1 3
KL: x    ;  là các nghiệm
2 2 
Câu 3.


x  x y  2 y
3 2

 x  x y  2 x y  2 y (1)
3 2 2 3

a.   2

 x 2
y  y 3
 y x y  y  y

3

(1)  x3  x 2 y  2 y 3  0
 ( x  y )( x 2  2 xy  2 y 2 )  0
x   y
 2  x  y
 x  2 xy  2 y 2
 0
Khi đó  x  x   x  x  0, y  0
3 3

hệ có nghiệm duy nhất ( x; y ) là (0;0)

b.

5 x  2 y  0 x  0
Điều kiện  ; nhận xét  không thỏa mãn hệ
7 x  y  0  y  0

nên 2 x  5 x  2 y  3 y  7 x  y

 (2 x  3 y )  7 x  y  5 x  2 y  0
7 x  y  5x  2 y
 2x  3y  0
7 x  y  5x  2 y
 1 
 (2 x  3 y ) 1  0
 7 x  y  5 x  2 y 
 2x  3y

3y2 y  2
Khi đó  6  y2  4  
2  y  2
y  2 thì x  3 (thỏa mãn)

91
Page: Tài Liệu Môn Toán
Website: tailieumontoan.com

y  2 thì x  3 (loại)
KL: hệ có nghiệm duy nhất ( x; y ) là (3;2)

Câu 4.

m 1
D  m
2m 1
3m 1
Dx   2m  3
m3 1
m 3m
Dy   5m 2  3m
2m m  3
Hệ có nghiệm duy nhất  D  0  m  0

 2m  3
 x  m 
 x  2 
3
khi đó nghiệm là   m
y  5m 2
 3m  y  5m  3
 m
m  1
x   m  1

y khi 
m  m  3
 
 m  3
Câu 5.
Từ giả thiết ta có:
1 1 1
1 1
x y z

92
Page: Tài Liệu Môn Toán
Website: tailieumontoan.com

1 y 1 z 1 y 1 z 1
  2 .
x y z y z
1 z 1 x 1
2 .
y z x
1 x 1 y 1
2 .
z x y
1 ( x  1)( y  1)( z  1) 2
 8. ( )
xyz xyz
1
 ( x  1)( y  1)( z  1) 
8
3
Dấu bằng xảy ra khi x  y  z 
2
Câu 6.
AB  4 đặt BC  x, AD  y ( y  x)
( x  y).4
Diện tích hình thang là 24 nên : 24   x  y  12
2
Chu vi hình thang là : 16  4 2 nên:

16  4 2  4  x  y  16  ( y  x)2 nên

y  x  4
4 2  16  ( y  x) 2   khi đó
 y  x  4(loai )
 x  y  12  x  4
 
 y  x  4 y  8
AD  8 phương trình AD : y  2 , D( x0 ;2) ; x0  2

AD  8  ( x0  1) 2  8
 x0  1  8  x0  9
 
 x0  1  8  x0  7( L)
D(9;2)
93
Page: Tài Liệu Môn Toán
Website: tailieumontoan.com

1
BC  AD Tìm được C (5;6)
2
Câu 7.

a.
(C ) có tâm I (1; 2) bán kính R  1  4  3  8
 / /d ;  có phương trình x  y  c  0 (c  0)
31
 cắt (C ) theo d}y cung có độ dài bằng 1 nên tính được d( I ; ) 
2
 31
 c 1
1 2  c 31 2
Do đó:   (tm)
2 2  31
c   1
 2
b. Nhận xét IA  R nên A nằm trong đường tròn
Tia AI cắt (C ) tại M, tiếp tuyến của (C) tại M là d1

d( A;d1 )  IA  R  AM
Gọi d 2 là tiếp tuyến bất kì của (C ) tại M 1

d( A;d2 )  AM1  AM
Vậy d1 là tiếp tuyến có khoảng cách từ A đến tiếp tuyến là lớn nhất

IM   8 IA
M ( x; y) ; IM  ( x  1; y  2) ;
IA  (1;0)
 x  1   8(1)  x  8  1
 
 y  2  0  y  2
M ( 8  1; 2)

vec to pháp tuyến d1 là IA(1;0)

94
Page: Tài Liệu Môn Toán
Website: tailieumontoan.com

Phương trình 1( x  8  1)  0  x  8  1  0

c, Gọi n(a; b) l| vec tơ ph{p tuyến của d' (a 2  b2  0)

n1 (1;1) l| vec tơ ph{p tuyến của d

n.n1 1 ab
cos600     a 2  b2  2(a 2  b2  2ab)
n n1 2 a b . 2
2 2

 a  (2  3)b
 a 2  b 2  4ab  0  
 a  (2  3)b

Chọn b  1 , a  2  3

Kl: C{c đường thẳng thoả mãn là: (2  3) x  y  0; (2  3) x  y  0

Đề 12

Câu 1. * PT ho|nh độ giao điểm: x  3x  1  m  0. (1)


2

* d cắt (P) tại 2 điểm phân biệt khi và chỉ khi PT (1) có 2 nghiệm phân biệt
13
   0  13  4m  0  m  . (*)
4
A( x1;  x1  m); B( x2 ;  x2  m) với x1; x2 là nghiệm của pt (1).
 x1  x2  3
Theo hệ thức Viet: 
 x1.x2  m  1
Theo giả thiết ta có OAB vuông tại O

 OA.OB  0  2 x1 x2  m  x1  x2   m 2  0
 m  1
 m2  m  2  0  
m  2
Đối chiếu đk (*) ta có 2 gi{ trị của m là m  1 và m2
Câu 2.

2.1 * Đặt t  x  0 , pt đã cho trở thành:


2

95
Page: Tài Liệu Môn Toán
Website: tailieumontoan.com

 m  2 t 2  2  m  1 t  2m  1  0 (*)
* Phương trình đã cho có 4 nghiệm phân biệt khi và chỉ khi pt (*) có hai nghiệm dương
phân biệt hay

m  2  0

 '   m  1   m  2  2m  1  0
2

 m 1
 0
m  2
 2m  1
 0
 m2
m  2

7  3 5  m  7  3 5
 2 2

  m  1 73 5
  2m
m  2 2
 1
m 
 2

m  2
 73 5
* Kết luận: m   2; 
 2 

2.2 * Ta có 3sin
4
  8cos2   5  3sin 4   8 1  sin 2    5  0

 3sin 4   8sin 2   3  0
 2 1
sin  

1
3  sin 2  
 2 3
sin   3( vôlí)

* Với 0    , ta có cos  0
2
1 2 6
Khi đó cos   1  sin   1  
2

3 3 3

96
Page: Tài Liệu Môn Toán
Website: tailieumontoan.com

3
1  6  1 2 6
2

* Ta có P  sin   cos  =      
4 3

3  3  9
Câu 3.
3.1* TXĐ:
4 2

Ta có x  x  1  x  x  1 x  x  1
2
 2

 
* pt  x  x  1  2 x  x  1  4 
2 2
x 2
 x  1 x 2  x  1

* Đặt a  x 2  x  1, b  x 2  x  1
Pt đã cho trơ th|nh: b  2a  ab  4
2

b  2
  b  2  b  a  2   0  
b  a  2
1  13
* Với b  2 , ta có x2  x  1  2  x2  x  3  0  x 
2
* Với b  a  2 , ta có x2  x  1  x2  x  1  2  x2  x  1  2  x2  x  1

 2 x2  x  1  x  2
x  2 x  2
 2   vô nghiệm.
 4 x  4 x  4  x 2
 4 x  4  x  0

1  13 1  13
* Kết luận: Phương trình đã cho có hai nghiệm là x  ; x
2 2

 x  5x  y  9
2

 x  2 x  3x  y  9
2

3.2   2

3 x 3
 6 x 2
 x 2
y  2 xy  18 ( x  2 x)(3x  y )  18

Đặt u  x  2 x; v  3x  y
2

u  v  9 u  6 u  3
Hpt trở thành   hoặc 
uv  18 v  3 v  6
u  3  x2  2 x  3  x  1  x  3
* Với  , ta có   hoặc 
 v  6 3x  y  6 y  3  y  15

97
Page: Tài Liệu Môn Toán
Website: tailieumontoan.com

u  6  x2  2 x  6 
 x  1  7  x  1  7
* Với  , ta có   hoặc 
 v  3 3x  y  3 
 y  6  3 7  y  6  3 7

 
KL:hpt có 4 nghiệm là 1;3 ,  3;15 , 1  7;6  3 7 , 1  7;6  3 7 
3.3 * Đặt t  3x  2 x  t  0  . Ta có bất phương trình:
2

3  t 2  2t  t 2  2t  3  0  3  t  1
* Kết hợp với điều kiện t  0 ta được: 0  t  1

 2
 x  3
3x 2  2 x  0  
* Với 0  t  1 , ta có 0  3 x  2 x  1     x  0
2

3x  2 x  1
2
 1
  x  1
 3
 1
 3  x  0

2  x 1
 3
 1  2 
* Kết luận tập nghiệm của bất pt đã cho l| : S    ;0   ;1
 3  3 
Câu 4
2
* Ta có 3MA  2MC  0  5MA  2 AC  0  MA   AC
5


Suy ra M nằm giữa A và C nên AM , AB  MAB  60  0

2
AM  AM  AC  4 , BM  AM  AB
5

 
2
BM 2  AM  AB  AM 2  AB 2  2 AM . AB
*
 AM 2  AB 2  2 AM . AB.cos600  21
2 1 1
* Chỉ ra BM  AC  AB; AD  AB  AC
5 2 2

98
Page: Tài Liệu Môn Toán
Website: tailieumontoan.com

2  1 1 
* BM . AD   AC  AB  . AB  AC 
5  2 2 
3 1 1
  AB. AC  AC 2  AB 2  0
10 5 2
Suy ra BM  AD
Câu 5
5.1 * Ta có a  5; b  9, suy ra c  4 , MF1  MF2  2a  10, F1 F2  2c  8

MF1  MF2  F1 F2
* p 9
2
4
 Ta có S MF1F2  pr  9.  12
3
1
 Mặt khác S MF1F2  .d  M ,Ox .F1F2  4.d  M ,Ox 
2
Từ đó ta có d  M ,Ox   3  yM  yM  3

* Do đó M  xM ;3 hoặc M  xM ; 3

Vì M   E  nên xM  0 . Khi đó ta có M  0;3 hoặc M  0; 3

Kết luận : M  0;3 hoặc M  0; 3

5.2a

* M  2a  3; a   d , đường tròn (C) có tâm I  2; 1 , bán kính R  2 2

* Từ M kẻ được 2 tiếp tuyến tới (C) khi và chỉ khi M nằm ngoài hình tròn (C) khi và chỉ
khi IM > R hay IM > 2 2 (*)
* Giả sử từ M kẻ được 2 tiếp tuyến tới (C) và 2 tiếp tuyến đó vuông góc với nhau khi đó
IM = R 2  4 ( thỏa mãn (*))

 2a  5   a  1  4   2a  5   a  1  16
2 2 2 2
hay

99
Page: Tài Liệu Môn Toán
Website: tailieumontoan.com

 5a 2  18a  10  0
 9  31
a 
5

 9  31
a 
 5
9  31  3  2 31 9  31 
* Với a  , ta có M  ; 
5  5 5 

9  31  3  2 31 9  31 
* Với a  , ta có M  ; 
5  5 5 

 3  2 31 9  31   3  2 31 9  31 
* Kết luận : M  ;  hoặc M  ; 
 5 5   5 5 
5.2b * Từ BD  2 AC , ta có IB  2IA
1 1 1
* Trong tam giác vuông IAB ta có 2
 2 ( H là tiếp điểm của AB với (C))
IA IB IH 2
5 1
suy ra 2
  IA  10
4 IA 8
* Giả sử A  2t  3; t   d và xA  1 nên t  2 . Ta có
t  2(tm)
IA  10   2t  5   t  1  10   8
2 2

t   không tm 
 5
* t  2 , ta có A 1;2 

Kết luận : A 1;2 

Câu 6.
* Với a, b, c là 3 số dương. Chứng minh được :

 a  b  c  
1 1 1
    9 (1)
a b c
1 1 1 9
Áp dụng bđt (1) ta có A=   
xy  2 yz  2 zx  2 xy  yz  zx  6

100
Page: Tài Liệu Môn Toán
Website: tailieumontoan.com

 x  y   0
2



Ta có  y  z   0   x  y  z  xy  yz  zx
2 2 2 2


 z  x   0 
2

Theo giả thiết x  y  z  3


2 2 2

x  y  z
Suy ra A  1 .Dấu = xảy ra khi và chỉ khi   x  y  z 1
x  y  z  3
2 2 2

Vậy giá trị nhỏ nhất là A = 1 khi và chỉ khi x  y  z  1

Đề 13
Câu 1. * PT ho|nh độ giao điểm: x2  4 x  1  m  0. (1)
* d cắt (P) tại 2 điểm phân biệt khi và chỉ khi PT (1) có 2 nghiệm phân biệt
  '  0  4  m  1  0  m  5 (*)
A( x1;2 x1  1); B( x2 ;2 x2  1) với x1; x2 là nghiệm của pt (1).

 x1  x2  4
Theo hệ thức Viet: 
 x1.x2  m  1
* Ta có

AB 2   x1  x2    2 x1  1  2 x2  1  5  x1  x2 
2 2 2

 5  x1  x2   4 x1 x2   5 16  4  m  1  5  20  4m 
2
 
24
* AB  2  AB 2  4  5  20  4m   4  m 
5
24
Đối chiếu đk (*) ta có gi{ trị của m là m 
5
Câu 2.
2.1
* Đặt t  x  0 , pt đã cho trở thành:
2

 m  2 t 2  2mt  m2  4  0 (*)
* Phương trình đã cho có 3 nghiệm phân biệt khi và chỉ khi pt (*) có một nghiệm dương
và một nghiệm bằng 0
* Phương trình (*) có nghiệm t=0 khi và chỉ khi
101
Page: Tài Liệu Môn Toán
Website: tailieumontoan.com

m2  4  0 hay m  2
 Với m=2, phương trình (*) trở thành -4t=0. Trong trường hợp này, (*) chỉ có một
nghiệm là 0.
 Với m=-2, phương trình (*) trở thành 4t 2  4t  0 . Phương trình n|y có hai
nghiệm t=0 và t=1
* Kết luận: vậy PT đã cho có ba nghiệm phân biệt khi và chỉ khi m=-2
2.2 * Ta có
sin  a  b   sin a cos b  cos a sin b
cos  a  b   cos a cos b  sin a sin b
cos  a  b   cos a cos b  sin a sin b

2  sin a cos b  cos a sin b 


 VT=  tan b
cos a cos b  sin a sin b  cos a cos b  sin a sin b
2  sin a cos b  cos a sin b  sin b
 
2cos a cos b cos b
sin a cos b  cos a sin b sin b
 
cos a cos b cos b
sin a cos b
  t ana  VP
cos a cos b
Câu 3.
3.1

1  4 x 2  0  1 1
* ĐK:  hay x    ;  \ 0
x  0  2 2
* Thực hiện phép nhân liên hợp ta thu được BPT

4 x  3(1  1  4 x 2 )  3 1  4 x 2  4 x  3 .

102
Page: Tài Liệu Môn Toán
Website: tailieumontoan.com

 3 y
  x  B
 4 x  3  0 
4
  x  1
 1  4 x  0
2

  
1
 2  x A
4x  3  0 
    3 C O2 x
x
 9(1  4 x 2 )  (4 x  3) 2   4

 9(1  4 x )  (4 x  3)
2 2 D

 1 1
Kết hợp ĐK thu được nghiệm   ;  \ 0
2 2  
3.2
1
ĐK: 4 x  1  0  x 
4
Biến đổi phương trình:

4 x 2  4 x  10  2  4 x  1 x 2  3  4  x 2  3  4 x  2  2  4 x  1 x 2  3 1

 
Đặt 2 x 2  3  t t  2 3 . Khi đó phương trình 1 có dạng:

t  1 l 
t 2   4 x  1 t  4 x  2  0  
t  4 x  2
 1
x  2  10
Với t  4 x  2  2 x  1  x  3  
2
2 x tm  .
3x  4 x  2  0
2 3

2  10
Vậy phương trình có nghiệm duy nhất x  .
3

4  x  0
2
x 2

3.3 Điểu kiện:   
3  2 y  y  0
 1  y  3

2

Biến đổi phương trình đầu của hệ, ta có:

103
Page: Tài Liệu Môn Toán
Website: tailieumontoan.com

 x  1  3  x  1  y 3  3 y
3

  x  1  y   x  1  y 2   x  1 y  3  0
2
 
 y  x 1

 x  1  y   x  1 y  3  0
2 2

*) Với y  x  1 thay v|o phương trình thứ hai ta có:

 2
 2
 x
x   3
4  x 2  2  3x   3   x0
10 x  12 x  0
2 6
 x  0  tm   x   l 
 
 5
Vậy hệ phương trình có nghiệm là  x, y    0,1 .
2
 y  3y2
*) Với  x  1  y   x  1 y  3  0   x  1     3  0 VN 
2 2

 2 4
Câu 4.

* Ta có AB. AC  AB. AC .cos BAC  AB. AC.cos BAC (1)

* Theo định lí cosin trong tam giác ABC ta có


AB 2  AC 2  BC 2
cos BAC  (2)
2 AB. AC
* Thế (2) vào (1) suy ra điều phải chứng minh.
Câu 5.
x2 y 2
5.1 Gọi (E) có dạng   1 a  b  0 
a 2 b2
Theo gt: AC=2 BD=2b
Tâm của hình thoi ABCD là gốc toạ độ O cũng l| t}m đường tròn nội tiếp hình thoi

Ta có:
1 1
SAOB  OA.OB  2 AB
2 2
 ab  2. a 2  b 2
 a 2b 2  2(a 2  b 2 )(*)

104
Page: Tài Liệu Môn Toán
Website: tailieumontoan.com

c
Mặt khác : e   0,5  a  2c
a
3a 2
b  a c 
2 2 2
(**)
4
14 7
Thay (**) v|o (*) ta được 3a 4  14a 2  a 2  (do a  0)  b 2 
3 2
x2 y 2
Vậy ptct của (E) là :  1
14 7
3 2
Gọi I = d1  d2

5.2 A
2 x  y  1  0 x  1
I   . Vậy I(1;1) I 
2 x  y  3  0  y  1
B
Từ gt d1, d2 có VTPT n1  (2; 1); n2  (2;1);
IB=3IA
Gọi  là góc của d1 và d2
4 1 3 4
 cos     sin  
5 5 5
2
1 4 6 IA
 SIAB  IA.3IA. 
2 5 5
Từ gt: SIAB  6  IA  5  IB  45
2 2

A  d1  A(a,2a  1) với a > 0, a  1

a  0  l 
Pt IA  5  (a  1)  (2a  2)  5  5(a  1)  5  
2 2 2 2

 a  2  tm 
Vậy a = 2  A(2;3)
B  d2  B(a,3  2b)  IB2  (b  1)2  (2  2b)2  5(b  1)2

b  4  B(4; 5)
IB 2  45  (b  1)2  9  
b  2  B(2;7)
Với A(2;3); B(4;5) pt cần tìm là 4 x  y  11  0

105
Page: Tài Liệu Môn Toán
Website: tailieumontoan.com

Với A(2;3); B(-2;7) pt cần tìm là x  y  5  0

5.3
Đường tròn (C) có tâm I  2;1 bán kính R=3.

Do M thuộc d nên M(a ;1-a)


Điều kiện M nằm ngoài (C) : IM>R  IM 2  9  2a 2  4a  5  0 (*)

Ta có MA2  MB2  MI 2  IA2  2a 2  4a  5

A, B thuộc đường tròn tâm M bán kính MA :  x  a    y  a  1  2a 2  4a  5


2 2

(học sinh có thể chọn A, B thuộc đường tròn khác)


A, B thuộc đường tròn  C  : x 2  y 2  4 x  2 y  4  0

Suy ra phương trình AB:  a  2  x  ay  3a  5  0

Do (J) tiếp xúc với AB nên (J) có bán kính d(J,AB)


Chu vi của (J) lớn nhất khi và chỉ khi d(J,AB) lớn nhất

 5 11 
AB luôn đi qua điểm cố định K  ; 
2 2 
d(J,AB) lớn nhất khi K là hình chiếu vuông góc của J trên AB.

Đường thẳng AB có vecto chỉ phương AB  a; a  2 

JK . AB  0  a  2 (thỏa mãn (*))


Vậy M  2; 1

Áp dụng bất đẳng thức Cô-si, ta có:


Câu 6.

a6  b6  1  3 3 a6b6  3a 2b2  a6  b6  1  3a 2b2  3ab 1

c6  b6  1  3 3 c6b6  3c 2b2  c6  b6  1  3c 2b2  3cb  2 

a6  c6  1  3 3 a6c6  3a 2c 2  a6  c6  1  3a 2c2  3ac  3

Cộng 1 ,  2  ,  3 theo các vế tương ứng ta có:

P  a 6  b6  1  c 6  b6  1  a 6  c6  1  3  ab  bc  ca 
P3 3

106
Page: Tài Liệu Môn Toán
Website: tailieumontoan.com

Vậy GTNN của P bằng 3 3 . Dấu bẳng xảy ra khi và chỉ khi a  b  c  1.

Đề 14
Câu 1.
 x 3  12 x  7 1

x2  x  2 2
 2 x 3  24 x  14  x 2  x  2
 0
x2  x  2
 ( x  4)( x  3)(2 x  1)
 0
( x  1)( x  2)
( x  4)( x  3)(2 x  1)
  0 (1)
( x  1)( x  2)
Bảng xét dấu:

x 1
- -4 -1 2 3 +
2
x4 - 0 + + + + +

x3 - - - - - 0 +

2x  1 - - - 0 + + +

x 1 - - 0 + + + +

x2 - - - - 0 + +

VT (1) - 0 + - 0 + - 0 +

VT (1)  0

 4  x  1
1
  x2
2
x  3

107
Page: Tài Liệu Môn Toán
Website: tailieumontoan.com

1
Vậy tập nghiệm của bất phương trình l|: S=  4;1   ;2)  3;
2
m  1x 2  2 x  m  1  0
Câu 2.
Phương trình có 2 nghiệm phân biệt

m  1  0

1  (m  1)(m  1)  0
 2  m  2

m  1
2 m 1
Khi đó S  x1  x2  ; P  x1 x2 
m 1 m 1
A= 2 x1  3x1 x2  2 x2  3x1 x2
3 2 3 2

= 2( x1  x2 ) 3  9 x1 x2 ( x1  x2 ) = 2S  9PS  2
3

m 1 2
3
 2 
 2  9 . 2
 m  1  m  1 m  1
16  18(m 2  1)
 2
(m  1) 3
 16  18m2  18  2m3  6m2  6m  2
 m3  12m2  3m  16  0

108
Page: Tài Liệu Môn Toán
Website: tailieumontoan.com

 2  m  2
Kết hợp điều kiện 
m  1
 13  105
Vậy m=1; m=
2
( x  1)( y  1)( x  y  2)  6
Câu 3.  (I)
x  y  2x  2 y  3  0
2 2

Đặt u = x -1 ; v = y -1

uv.(u  v)  6
(I) trở thành 
u  v  5
2 2

uv.(u  v)  6

(u  v)  2uv  5
2

u  v  3

uv  2
u  2

v  1

u  1

v  2
 x  3

y  2
Khi đó  KL
 x  2

 y  3
Câu 4.
a. Đường tròn tâm A tiếp xúc với đường thẳng d có bán kính là
2.1  2  4 8
R  d ( A; d )  
5 5
64
Phương trình đường tròn đó l|: ( x  1)  ( y  2) 
2 2

5
b.

109
Page: Tài Liệu Môn Toán
Website: tailieumontoan.com

Gọi n(a; b) l| véc tơ ph{p tuyến của đường thẳng  (a  b  0) .


2 2

Đường thẳng d có một véc tơ ph{p tuyến là n1 (2;1)

2a  b
Ta có cos 45 
0
(1)
a2  b2 . 5
Giải (1) tìm được: a  3b hoặc b = -3a

Với a = 3b chọn b = 1; a = 3 khi đó phương trình  là:


3x + y - 8 = 0
Với b = -3a chọn a = 1; b = -3 khi đó phương trình  là:
x - 3y - 6 = 0

c. OA  5; OB  10; AB  5

Chu vi tam giác OAB : P  10  2 5


Tam giác OAB cân tại A

3 1 5
Gọi K l| trung điểm OB ta có K ( ; ) , AK 
2 2 2
Diện tích tam giác OAB :
1 5
S  . AK.OB  (đvdt)
2 2
Lưu ý : Học sinh có thể tính theo công thức hêrông
d. Đường tròn (C) có tâm J(2; 1) bán kính 1
3
Gọi I l| trung điểm AB ta có I (2; ) ; IA  IB  0
2
F  MA2  MB2  (MI  IA) 2  (MI  IB) 2  2MI 2  IA2  IB2
5
Nhận xét: IA  IB không đổi. IJ   1. Điểm I nằm ngo|i đường tròn (C)
2 2

2
F nhỏ nhất khi MI nhỏ nhất khi M, I, J thẳng hàng và M thuộc (C) và nằm giữa I, J
3
Khi đó MI  MJ
2
Tìm được M(2; 0)

110
Page: Tài Liệu Môn Toán
Website: tailieumontoan.com

1 4
Câu 5. Ta có sin   cos  nên sin  . cos 
3 9
1 9
tan   cot  
sin  . cos 4
49
tan 2   cot 2   (tan   cot ) 2  2 
16
13
Vậy F 
16
Câu 6.
 
x 2  5x  4 1  x 3  2 x 2  4 x (1)
x  5  1
Đk 
 1  5  x  0
(1)  x 2  2 x  4  3x  4( x( x 2  2 x  4)
TH1 : 1  5  x  0
x 2  2x  4  0
Khi đó  (Hai biểu thức không đồng thời bằng 0)
3x  0
 x 2  2 x  4  3x  0  4 x( x 2  2 x  4)
 1  5  x  0 thoả mãn bất phương trình
TH 2 : x  1  5  x 2  2 x  4  0
Đặt a  x  2 x  4  0 ; b  x  0 Khi đó:
2

a 2  3b2  4ab
 (a  b)(a  3b)  0
 b  a  3b
Hay x  x2  2x  4  3 x

x 2  x  4  0
 2
x  7 x  4  0
 1  17 7  65
 x
2 2

Vậy S  
  1  17 7  65 
; 
   1  5;0 
 2 2 
Câu 7.
111
Page: Tài Liệu Môn Toán
Website: tailieumontoan.com

 2 x 2  4 y 2 4 ( 2 x  3 y )( x  y ) (1)
  1
 xy xy

 2( x  y  3)  x  y  3 ( 2)
(2 x  3 y )( x  y )  0
Điều kiện: 
 x  0; y  0
(1)  2 x  4 y  xy  4 (2 x  3 y)( x  y ) xy
2 2

 2 x 2  3xy  4 y 2  4 xy  2 (2 x 2  3xy )(4 xy  4 y 2 )  0

 2 x 2  3xy  4 xy  4 y 2
 2 x 2  3xy  4 xy  4 y 2
 2 x 2  7 xy  4 y 2  0
2
 x x
 2   7  4  0 (vì y > 0)
 y y
x  4 y

2 x   y
Kết hợp x; y > 0 suy ra x = 4y
x  y  3  2( x  y  3)
 x  y  3  2 x( y  3)  0
 ( x  y  3) 2  0
 x y3
x  4
Kết hợp x  4 y suy ra 
y 1
Thử lại vào hệ, ( x; y)  (4;1) thỏa mãn hệ.

Vậy ( x; y)  (4;1)
1 1 1 1
Câu 8. P    
a  b  c ab(a  b) bc(b  c) ca(c  a)
2 2 2

1
1  a  b  c  33 abc  abc 
27
1 1 1 3
Có   
ab(a  b) bc(b  c) ca(c  a) 3 abc(ab  bc)(bc  ca)(ca  ab)

112
Page: Tài Liệu Môn Toán
Website: tailieumontoan.com

3

1 (ab  bc  bc  ca  ca  ab)
3 .
27 3
27
=
2(ab  bc  ca )

1 27
M  
a b c
2 2 2
2(ab  bc  ca)
1 1 1 23
   
a b c
2 2 2
ab  bc  ca ab  bc  ca 2(ab  bc  ca)
9 3.23
 2 
a  b  c  2ab  2bc  2ca 2(a  b  c) 2
2 2

9 69 69 87
  9 
(a  b  c) 2
2(a  b  c) 2
2 2
87 1
Vậy M  . Dấu bằng xảy ra khi a=b=c= .
2 3

Đề 15
Câu 1.
1.1 Hàm số y có tập x{c định D  (10; 10) là tập đối xứng qua điểm x  0.
Kiểm tra: x  D, f ( x)  f ( x)  f chẵn
f không lẻ (vì nó không đồng nhất bằng 0 trên D), kết luận
1.2 C  [b; b  2)  (a; a  1] là một đoạn  b  a  b  2  a  1
 b  1  a  b  2. (*)
Khi đó, C  [b; b  2)  (a; a  1]  [b; a  1] l| đoạn có độ dài a  b  1.
Câu 2.
2.1 Ta có: m4  m2  1  0
 x 2  m4  m2  2 (1)
PT  
 x  m  m  m (1  m )
2 2 4 2 2
(2)

(1) có 2 nghiệm phân biệt với mọi m vì m4  m2  2  0


(2) có 2 nghiệm phân biệt  m  0 và 1  m2  0  m (1; 1)\{0}
113
Page: Tài Liệu Môn Toán
Website: tailieumontoan.com

PT có 4 nghiệm phân biệt  m (1;1)\{0} và m4  m2  2  m2  m4

 m (1;1)\{0} và m4  m2  1  0  m (1;1)\{0} , kết luận


(m  1)( x  2)  (1  m) x  2 x  (m  2)
2.2 BPT  0  0
x2 x2
Nếu m = 0 thì BPT nghiệm đúng với mọi x  2
Nếu m > 0 thì m + 2 > 2 nên BPT nghiệm đúng với mọi x  (;2)  ( m  2; )
Nếu m < 0 thì m + 2 < 2 nên BPT nghiệm đúng với mọi x  (; m  2)  (2; )
Câu 3.
3.1 Điều kiện: x ≥ 0
PT  x2  1  7 x  7  2  2 x  0  ( x  1)( x x  x  6 x  8)  0

 ( x  1)( x x  8  x  6 x  16)  0

 ( x  1)( x  2)( x  2 x  4  x  8)  0

 ( x  1)( x  2)( x  x  4)  0

x  1
 x 1  0  2
     1  17  9  17
 x  x  4  0  x    
  2  2

7 x  y  0 u  7 x  y  0 u 2  7 x  y u 2  v2
3.2 Điều kiện  ; Đặt   x và
2 x  y  0 v  2 x  y  0 v  2 x  y
2 5
7v 2  2u 2
y
5

u  v  5
 u  v  5

HPT trở thành:  2 2   2
u  v  7v  2u  5v  5
 3u  8v  5v  5  0

2 2 2

u  5  v
 u  5  v
 u  5  v

      2
3(5  v)  8v  5v  5  0
 5v  25v  70  0
 v  5v  14  0 (*)

2 2 2

u  3
(*)  v = 2 (nhận) hoặc v = 7 (loại) ; nên HPT trên  
v  2
7 x  y  9 x  1
Do đó HPT đã cho trở thành   (phù hợp)
2 x  y  4 y  2
Câu 4.
114
Page: Tài Liệu Môn Toán
Website: tailieumontoan.com

4.1 Ta có: MC  2MB  AC  AM  2( AB  AM )  3 AM  2 AB  AC

Tương tự ta cũng có: 3CN  2CA  CB


Vậy: AM  CN  AM  CN  0  (2 AB  AC )(2CA  CB)  0

 (2 AB  AC )( AB  3 AC )  0  2 AB2  3 AC 2  5 AB. AC  0
5bc
 2c 2  3b2  0  4c2  6b2  5bc  0
2
4.2 Ta có các công thức tính diện tích: 2Sa  AC ' AB 'sin A; 2S  AB  AC sin A

Sa AC ' AB ' 1  AC ' AB ' 


       (BĐT Cauchy)
S AB AC 2  AB AC 

Sb 1  BA ' BC '  Sc 1  CB ' CA ' 


Tương tự ta cũng có:     và    
S 2  BC BA  S 2  CA CB 

Sa S S 1  AC ' BC ' BA ' CA ' CB ' AB '  3


Do đó:  b  c         (đpcm)
S S S 2  AB BA BC CB CA AC  2
 AC ' AB '
 AB  AC
 C ' B ' //BC
 BA ' BC ' 
Dấu bằng xảy ra      A ' C ' //CA  A’, B’, C’ l| trung điểm của BC, CA, AB
 BC BA  B ' A ' //AB
 CB ' CA ' 
 CA  CB

Câu 5. Dựa v|o tính đối xứng, ta giả sử A  a;0  , B  0; b  với a  0, b  0. (*) Suy ra
ab
SOAB  .
2
1 1 1 1 a 2  b2
Mà 2
 2
 2
(**) 2
 2 2  a 2b2  R 2 (a 2  b2 )  2 R 2 ab
a b R R ab
ab
 SOAB   R 2 không đổi (dấu bằng xảy ra khi và chỉ khi a = b)
2
Kết hợp với (*) và (**): dấu bằng xảy ra khi và chỉ khi a  b  R 2

   
Kết luận: A  R 2;0 ; B 0;  R 2 (4 cặp điểm)

Đề 16
Bài 1. 1) Tìm a; b<<.

115
Page: Tài Liệu Môn Toán
Website: tailieumontoan.com

b 3
Do Parabol nên a  0 và có trục đối xứng x     3a  b  0
2a 2

Tọa độ đỉnh có tung độ là y   mà   b2  4a nên ta có: b2  4a  22a hay
4a
b2  18a  0
 3a  b  0
Ta có hệ pt  2 thế v|o ta được: b2  6b  0  b  0; b  6
b  18  0
Nếu b  0  a  0 loại.
Nếu b  6  a  2 thỏa mãn.
Vậy a = 2 ; b = 6 là giá trị cần tìm.
2) Tìm k < với parabol y  2 x 2  6 x  1
Để đường thẳng cắt Parabol tại hai điểm phân biệt thì pt
2 x2  6 x  1  kx  6 x  1 có hai nghiệm phân biệt x1 ; x2

hay pt: 2 x  kx  2  0 có hai nghiệm phân biệt x1 ; x2 có   k 2  16  0


2

Khi đó, giao điểm M  x1;  k  6  x1  1 , N  x2 ;  k  6  x2  1

 x  x  k  6  x2  1   k  6  x2  1 
nên trung điểm của đoạn MN là I  1 2 ; 
 2 2 

 1 
 2  3k  k 2 
k k 2
Theo định lý Viet ta có x1  x2  nên I  ; 
2 4 2 
 
Do I thuộc đường thẳng 4x + 2y – 3 = 0 nên k  8k  2  0 hay k  4  18 thì thỏa
2

mãn bài toán.


Bài 2.
2
Cho tam gi{c đều ABC v| c{c điểm M , N , P thỏa mãn BM  k BC , CN  CA ,
3
4
AP  AB . Tìm k để AM vuông góc với PN .
15
+) BM  k BC  AM  AB  k ( AC  AB) A

 AM  (1  k ) AB  k AC . P
N

116
Page: Tài Liệu Môn Toán

B M C
Website: tailieumontoan.com

4 1
+) PN  AN  AP   AB  AC
15 3
Để AM vuông góc với PN thì AM .PN  0

 4 1 
 (1  k ) AB  k AC    AB  AC   0
 15 3 
4(1  k ) k 1  k 4k
 AB 2  AC 2  (  ) AB AC  0
15 3 3 15
4(1  k ) k 1  k 4k
  (  )cos600  0
15 3 3 15
1
k
3
1
KL: k 
3
Bài 3.
3m  1
1) Tìm m để phương trình x  6 x 9  m x  2 x 9 8  x 
2
Ta có:
PT Û x - 9 + 3 + m  
x - 9 +1 = x +
3m +1
2
đặt t = x - 9, t  0

3m  1
PT trở thành : t  3  m  t  1  t 2  9   2t 2  2  m  1 t  m  13  0 (1)
2
PT ban đầu có nghiệm x1  10  x2

 '  0

 (1) có nghiệm 0  t1  1  t 2   t1  1 t 2  1  0

 t1  t 2  0
 m  1 2  2  m  13   0
 m 2  25  0
 m  13 
  m  1  1  0  13  m  0  m  13
 2  m  1
m  1  0 

2) Giải phương trình x  3  x . 4  x  4  x. 5  x  5  x. 3  x

117
Page: Tài Liệu Môn Toán
Website: tailieumontoan.com

Điều kiện: x  3

Đặt 3  x  a ; 4  x  b ; 5  x  c với a, b, c là số thực không âm.


Ta có x  3  a  4  b  5  c  a.b  b.c  c.a
2 2 2

Do đó

3  a 2  ab  bc  ca  a  b  c  a   3
 
4  b  ab  bc  ca   b  c  a  b   4
2

 
     c  a  b  c   5
2
5 c ab bc ca

Nhân từng vế ba phương trình ta được  a  b  b  c  c  a   2 15

 2 15
a  b 
 5
 2 15 15 15 15
Suy ra b  c  abc  
 3 5 4 3
 2 15
c  a 
 4
671 671
Suy ra x  . Thử lại x  thỏa mãn phương trình.
240 240
671
Vậy phương trình có nghiệm duy nhất là x 
240
 x2  y 2  2 y  6  2 2 y  3  0
3) Giải hệ phương trình 
 .
( x  y )( x  xy  y  3)  3( x  y )  2

2 2 2 2

Giải
 x2  y 2  2 y  6  2 2 y  3  0
Giải hệ phương trình 
(1)
 .
( x  y )( x  xy  y  3)  3( x  y )  2 (2)
2 2 2 2

ĐKXĐ: y  1,5 .
(2)  x3  y3  3x  3 y  3  x2  y 2   2   x  1   y  1  x  1  y  1  y  x  2
3 3

Thay vào pt thứ nhất ta được:


2
1  1
2
 2x 1  1  x
x  3x  1   2 x  1   x     2 x  1    
2

 2  2  2 x  1  x

118
Page: Tài Liệu Môn Toán
Website: tailieumontoan.com

(Có thể bình phương được pt:  x  1 ( x 2  4 x  2)  0)


2

Giải hai pt n|y ta được x  1, x  2  2


Vậy hệ có hai nghiệm là  x; y   1; 1 , 2  2,  2 . 
Bài 4. 1. Tính EA, CE theo a.
2a
Ta có CE = ;
3

2 a 2 10a 2
2 2 a 10
Ta có AE = AB + BE = a + = nên AE =
9 9 3
Mặt khác: EA.CE = EA.CE.cosAEB
BE 1
Trong tam giác vuông BAE ta có cosAEB  
AE 10
a 10 2a 1 2
Nên EA.CE = EA.CE.cosAEB  . .  a 2.
3 3 10 9
2.

Chứng minh AIC  90o


1
Ta có AE  AB  AD . Giả sử BI  k.BF, k  R
3

  k

AI  AB  BI  AB  kBF  AB  BC  CF   1   AB  kAD
 2
 k 1 2
Do A;E;I thẳng h|ng nên:  1   :1  k :  k 
 2 3 5
6 2 6 2 1 3
Nên AI  AB  AD và CI  AI  AC  AB  AD  AB  AD  AB  AD
5 5 5 5 5 5
6 2  1 3  6 6
Nên AI.CI   AB  AD   AB  AD   a 2  a 2  0 nên AIC  90o.
5 5  5 5  25 25

Bài 5. Cho các số dương a, b, c có a+b+c=3. Tìm gi{ trị nhỏ nhất của biểu thức
a a b b c c
P   .
2c  a  b 2a  b  c 2b  c  a
Giải

119
Page: Tài Liệu Môn Toán
Website: tailieumontoan.com

a a a3 1 a3 a3 c3 c3
  (   )
2c  a  b c  (a  b  c ) 2 c  3 c3 8 16

1 a3 a 3 c  3 c  3 3a c  3
 33   
2 c3 c3 8 16 4 16

a a 3a c  3
Suy ra:  
2c  a  b 4 16

b b 3b a  3 và c c 3c b  3
Tương tự    
2a  b  c 4 16 2b  a  c 4 16
3
Cộng các vế tương ứng của ba BĐT cùng chiều ta được P  ,
2
3
P khi a=b=c=1. KL
2

Đề 17
Câu 1.

 x  1
2
1) Pt   4  x  3 . Đặt x  3  y  1, ( y  1)

 ( y  1)2  x  3

( y  1)  x  3 ( x  y )( x  y  1)  0
 2

Ta có hệ phương trình:  
( x  1)  y  3 ( y  1)  x  3

2 2

x  y

x  y  0 x  y   y  3  17 (loai )
•   2   2
( y  1)  x  3  y  3 y  2  0
2 

  y  3  17  x  3  17
  2 2
x  1 y

x  1 y x  1 y   y  1  13 (loai )
•   2   2
( y  1)  x  3  y  y  3  0
2 

  y  1  13  x  1  13
  2 2

3  17 1  13
KL: Phương trình có hai nghiệm là x  và x 
2 2

120
Page: Tài Liệu Môn Toán
Website: tailieumontoan.com

 x  y  x  y  2(1)

2) Giải hệ phương trình 

 x  y  x  y  4(2)
2 2

ĐKXĐ: y  0; x  y ; x 2  y.

NX: Ta thấy VT(1)  0


x  2
• pt(1)  2 x  2 x 2  y  4  x 2  y  x  2  
 y  4x  4

• Thay y  4 x  4 v|o pt(2) ta được:

x2  4 x  4  x2  4 x  4  4  x  2  x 2  4 x  4  4 ( vì x  2)

x  6 5
 2 x
 x  4 x  4  (6  x)
2
2

5 5 
Với x   y  6. KL: Hệ có nghiệm ( x; y)   ;6  .
2 2 

Câu 2.
1) Tìm tham số m<
• Để bất phương trình có tập nghiệm ta cần có mx2  4 x  m  3  0 với x 
( m =0 không thỏa mãn)
m  0 m  0  m  1
  2 
  0 m  3m  4  0 m  4
• Với m  1
Khi đó ta có mx2  4 x  m  3  0 với x 
Bpt  x  1  mx2  4 x  m  3  mx2  5x  m  4  0 (1)
 4  41
m 
2
Bpt có tập nghiệm   (1)  0  4m2  16m  25  0  
 4  41
m 
 2
4  41
Mà m  1  m 
2

121
Page: Tài Liệu Môn Toán
Website: tailieumontoan.com

• Với m  4
Khi đó ta có mx2  4 x  m  3  0 với x 
Bpt  x  1  mx2  4 x  m  3  mx2  5x  m  4  0 (2)
 4  41
m 
2
Bpt có tập nghiệm   (2)  0  4m2  16m  25  0  
 4  41
m 
 2
4  41
Mà m  4  m 
2
4  41 4  41
KL: m  ; m
2 2
2) Tìm tham số m để hệ có hai nghiệm
Hệ pt
 y   x
 x 2  y 2  2( x  y )  2m  2 (I )
   x  m (1)
  x  y  4 
y  4 x
 x  y  0  
  ( x  2) 2  m(2)
( II )

NX: Nghiệm của hệ (I) thỏa mãn x y 0 và nghiệm của hệ ( II ) thỏa mãn
x y 4 hai hệ không thể có nghiệm chung
Hệ phương trình có đúng hai nghiệm phân biệt
 pt(1) có hai nghiêm phân biêt và pt(2) vô nghiêm( không xay ra)
  pt(2) có hai nghiêm phân biêt và pt(1) vô nghiêm( không xay ra)
 pt(1) và pt(2) có nghiêm kép  m=0

KL: m  0 .
Câu 3.
Tìm các hệ số b; c .
.• ĐK cần:

122
Page: Tài Liệu Môn Toán
Website: tailieumontoan.com

 1  1 1
 f (0)  2  2  c  2 (1)
 
 1  3 1
Ta có  f (1)     b  c   (2)
 2  2 2
 1  3 1
 f (1)  2  2  b  c   2 (3)
 

3 1 1
Từ (2) và (3)    c   kết hợp với (1)  c  
2 2 2
1 1  b  0
Với c   thay v|o (2) v| (3) ta được  b0
2 1  b  0
.• ĐK đủ:
1 1
Với b  0; c   ta có f ( x)  x 2 
2 2
1 1
1  x  1  0  x 2  1  f ( x)   b  0; c   thỏa mãn
2 2
Câu 4.
.• Ta có x2  1  x2  xy  yz  zx  ( x  y)( x  z )

x x x 1 1 
     
x2  1 ( x  y )( x  z ) 2  x  y x  z 

Tương tự ta có
y y 1 1 
    
y2 1 2 yx yz

z z 1 1 
    
z 1 2  z  x z  y 
2

1 x y 1 x z 1 z y 3
• VT          
x y2 2 x z2 2 z y2 2 2
1
Dấu bằng xảy ra  x  y  z 
3
Câu 5.
1) Chứng minh thẳng hàng<<
• Ta có

123
Page: Tài Liệu Môn Toán
Website: tailieumontoan.com

1 1
CN  BC  AN  AC  ( AC  AB )
2 2
1 3
 AN   AB  AC (1)
2 2
4
• 3MA  4MB  0  3 AM  4( AB  AM )  0  AM  AB (2).
7

• G là trong tâm tam giác ABC  GA  GB  GC  o  AG 


1
3

AB  AC  (3)

15 3
Từ (1) và (2)  MN  AN  AM   AB  AC
14 2
5 1
Từ (2) và (3)  MG  AG  AM   AB  AC
21 3
9
 MN  MG  M , N , G thẳng hàng.
2
b/ ( 2đ)
• Gọi P  MN  AC và E l| trung điểm BC
Đặt S1  SNPA ; S2  SNPC
1 1 S PA
Kẻ NH  AC,( H  AC ) Khi đó S1  PA.NH ; S2  PC.NH  1 
2 2 S2 PC

• Kẻ CK / / AG,( K  MN )
PA AG 2 EG EN
Ta có    2. 4
PC CK CK NC
S1 PA
  4
S2 PC

2)

124
Page: Tài Liệu Môn Toán
Website: tailieumontoan.com

• Ta có
1
AP.AF.sin A
SAPF 2 AP.AF AP.AF
  
SABC 1 AB.AC.sin A AB.AC bc
2
AP AC b AP b bc
Áp dụng t/c đường phân giác      AP 
BP BC a AP  PB a  b a b
bc
Tương tự  AF 
ac
SAPF AP.AF bc bc
    SAPF  SABC
SABC bc (a  b)(a  c) (a  b)(a  c)

Ho|n to|n tương tự ta có


ac ab
SBPE  SABC và SCEF  SABC
(a  b)(b  c) (a  c)(b  c)
Mà SEFP  SABC  (SAPF  SBPE  SCEF )

 bc ac ab  2abc
 1     SABC  SABC
 (a  b)(a  c) (a  b)(b  c) (a  c)(b  c)  (a  b)(b  c)(c  a)

Đề 18

Câu 1.

1) 3 3 4x - 3 - 4 6 - 2x + 5 = 0  3 3 4x - 3 + 5 = 4 6 - 2x
Dễ có x = 1 là một nghiệm của phương trình.
VT là hàm số đồng biến trên (- ∞ ; 3],
VP là hàm nghịch biến trên (- ∞ ; 3],
Vậy phương trình có nghiệm duy nhất x = 1
4(x 2 - 3xy + 2y 2 ) 4x 2 -12xy + 8y 2
2) Ta có M = =
4x 2 + y2 4x 2 + y2
+ y = 0 thì M = 1

125
Page: Tài Liệu Môn Toán
Website: tailieumontoan.com

4t 2 -12t + 8 x
+ y ≠ 0 thì M = 2
, (t  ) (*) gọi M là một giá trị bất kỳ của nó thì (*)4(M-
4t +1 y
1)t2 + 12t + M - 8 = 0 có nghiệm t
*, M = 1 , t = 7/12 thoả mãn
*, M ≠ 1để (*) có nghiệm thì ’ = - 4 (M2 – 9 M – 1) ≥ 0
 9  85 9  85 
 M  ; 
 2 2 

9  85
Vậy giá trị nhỏ nhất của M =
2
9  85
Vậy giá trị lớn nhất của M =
2
Câu 2.
u  x 2  x v  v  8
1) Đặt  hệ cho trở thành  vậy u, v là 2 nghiệm của phương trình
v  y  y uv  12
2

t2 – 8t + 12 = 0  t = 2 hoặc t = 6
TH1: Nếu u = 6 thì v= 2, khi đó ta có hệ:
 x 2  x  2
 2 có các nghiệm: (1; -3), (1; 2), (-2; -3), (-2; 2).
 y  y  6

TH2: Nếu u = 2 thì v= 6, khi đó ta có hệ:


 x 2  x  6
 2 có các nghiệm: (-3; 1), (2; 1), (-3; -2), (2; -2).
 y  y  2
Vậy hệ cho có các nghiệm (1; -3), (1; 2), (-2; -3), (-2; 2),(-3; 1), (2; 1), (-3; -2), (2; -2).
2) Đk x ≥ 3/4
Đặt .
phương trình đã cho trở thành :2010x2- 2009xt – t2 = 0
Giải ra : hoặc x = - t/2010 (loại)
* ta có :

Vậy x = 1 , x = 3 là các nghiệm của phương trình đã cho .


x y 5 2
Câu 3. Từ giả thiết ta có (d):  1 .Vì M  (d) nên:   1 (1)
a b a b
126
Page: Tài Liệu Môn Toán
Website: tailieumontoan.com

5 2 1 1
Theo BĐT Bunhiacopski 1 = (  )2  (25  8)( 2  2 )
a b a 2b
 33
1 1 1 a  5
Hay + ≥ đẳng thức xảy ra  
OA 2 2OB2 33  b   33
  4
Khi đó đường thẳng (d) có phương trình: 5x  4y  33  0
Câu 4. Phương trình đường thẩng AB: 4x + 3y -7 = 0
Giả sử C(x ;y). theo giả thiết ta có: x – 2 y -1= 0 (1)
4x  3y  7  4 x  3 y  37  0 (2a)
d(C, (AB)) = 6  6
5  4 x  3 y  23  0 (2b)
Giải hệ (1), (2a) ta được C1 (7; 3)
Giải hệ (1), (2b) ta được C2 (-43/11; -27/11)
Câu 5.
2010 - y 2010 - x 1 1
Ta có P = + = 2010( + ) - ( x + y) (1)
y x x y

1 1 4
Theo BĐT Côsi ta có +  (2)
x y x+ y
Đẳng thức xảy ra khi x = y

 
2
Theo BĐT Bunhiacopski ta có x y  2( x  y)  4020  x  y  4020 (3)

Đẳng thức xảy ra khi x = y


4.2010
Từ (1) , (2), (3) ta có P ≥   4020  4020 đẳng thức xảy ra khi x = y
4020
Vậy P đạt giá trị nhỏ nhất là 4020 khi x = y =1005
Đề 19
Câu 1.
1
2) Điều kiện: x  . Đặt t  2 x  1 ( t  0 ) thì 2 x  t 2  1. Khi đó ta có
2
x2  6 x  2  2(2  x)t  0  x2  2tx  4t  3(t 2  1)  2  0

 ( x  t )2  (2t  1)2  0  ( x  3t  1)( x  t  1)  0

127
Page: Tài Liệu Môn Toán
Website: tailieumontoan.com

1
 x  1  t (do x  3t  1  0; x  ; t  0 ).
2
x  1
Với x  1  t ta có x  1  2 x  1   2  x  2  2.
 x  2x  1  2x 1
Đối chiếu điều kiện ta có tập nghiệm của bất phương trình l| S  [2  2; ).
2)

 x  xy  y  y (1)
5 4 10 6
5
 Điều kiện: x  
 4 x  5  y  8  6 (2)
 4
2

Th1: y  0  x  0 không thỏa mãn


Th2: y  0 ta có:
5
x x
(1)      y 5  y  (t  y)(t 4  t 3 y  t 2 y 2  ty 3  y 4 )  0 với t=x/y
 y y
 (t  y) (t 2  y 2 )2  (t  y)2 (t 2  yt  y 2 )  2  0

 t=y hay y 2  x

Thay vào (2): 4 x  5  x  8  6  2 4 x2  37 x  40  23  5x


 23
x 
 5  x  1  y  1
 x  42 x  41  0
2

Đối chiếu đk ta được nghiêm hệ là: ( x; y)  (1;1);(1;1)

Câu 2.

my  y  m  0 (1)
2

Hệ đã cho tương đương với:  2


 x  yx  y  0 (2)

y  0
Phương trình (2) (ẩn x ) có nghiệm là  x  y 2  4 y  0  
 y  4
Th1: m  0, ta có y  0, x  0. Suy ra m  0 thỏa mãn.
Th2: m  0. Phương trình (1) (ẩn y ) không có nghiệm thuộc khoảng (; 4]  [0; ) (*)
là (1) vô nghiệm hoặc (1) có 2 nghiệm đều thuộc (4;0), điều kiện là

128
Page: Tài Liệu Môn Toán
Website: tailieumontoan.com

   1  4m 2  0  1 1
  m  (;  2 )  ( 2 ; )
   1  4m 2  0  
     1  4m 2  0   1  m  0
    1  4m  0  
2

 4  y  0   1  1  4m 2   2 (B)
  4  0 
    1  4m  1  8m ( A)
1 2
2m
 4  y2  0  
 1  1  4m 2   1  4m  1  8m
2

 4  2m
0  

(với y1 , y2 là 2 nghiệm của phương trình (1)).

 1 1
 2  m   8 1 4 4 1
(A)      m    (B)  m  (;  )  ( ; )
 1  4m2  1  8m 2 17 17 2

Hệ phương trình đã cho có nghiệm khi và chỉ khi phương trình (1) (ẩn y ) có ít nhất một
nghiệm thuộc khoảng (; 4]  [0; ) hay (*) không xảy ra, điều kiện là
4 1 4 1
 m  ; m  0. Vậy tất cả các giá trị m cần tìm là m .
17 2 17 2
Câu 3.
Gọi hình chiếu của I trên d1 , d 2 lần lượt là E , F . khi đó
2 6
IE  d( I ;d1 )  ; IF  d( I ;d2 )  .
5 5
6
Gọi R là bán kính của đường tròn (C ) cần tìm ( R  )
5

4 36
AB  2 AE  2 R 2  ; CD  2CF  2 R 2 
5 5

 4  36  4 36
Theo giả thiết ta có: 4  R 2    4  R 2    16  20 R 2  R2  .
 5  5  5 5

 8R2  16  4 (5R2  4)(5R2  36)  2R2  4  (5R2  4)(5R2  36)


6 6
 (2R2  4)2  (5R2  4)(5R2  36) (do R  )  R  2 2 ( do R  )
5 5
Vậy phương trình đường tròn (C ) cần tìm là (C ) : ( x  2)2  ( y  4)2  8.
Câu 4.
b c
1) Ta có: AL  AB  AC
bc bc

129
Page: Tài Liệu Môn Toán
Website: tailieumontoan.com

CA  CB AB  2 AC
CM  
2 2
Theo giả thiết: AL  CM  AL.CM  0


 bAB  c AC  AB  2 AC   0  bc 2
 bc 2 cos A  2cb 2 cos A  2cb 2  0
  c  2b 1  cos A  0  c  2b (do cos A  1)

b2  a 2 c 2 a 2  b2
Khi đó: CM 2   
2 4 2
1
  1
 
AB 2  AC 2  2 AB. AC   9b2  a 2 
2
2
AL2  AB  AC 
9 9 9
CM 2 9 a 2  b2 9
CM 3

AL 2
52 5 
AL2
 . 2  52 5
4 9b  a 2 4
 
a 2  b2 a2
  5  2 5   6 5
9b2  a 2 b2
b2  c 2  a 2 5b 2  a 2 5 1
cos A   2

2bc 4b 4
2)
a b
C/M được : a 2  b2  c2  d 2  (a  c)2  (b  d )2 . ấu bằng xẩy ra khi: 
c d
2 2
p  a2   a2  (a 2  4b 2 )2
Áp dụng (1) ta có :  1     1  b4  4    b2   4 
4  4  4  16

9 5
Mặt khác: (1  2a)(1  b)   a  2b  ab  (2)
2 2

 a 2  1  2a
 3(a 2  4b 2 )
Mà: 4b 2  1  4b   2  2a  4b  2ab  a 2  4b 2  2 (3)
 a 2  4b 2 2
  2ab
 2
1
Từ (1) và (3) suy ra: p  2 17 .Dấu “=” xẩy ra khi: a=1 và b 
2
1
Vậy: MinP  2 17 Đạt được khi a=1 và b  .
2

130
Page: Tài Liệu Môn Toán
Website: tailieumontoan.com

Câu 5.
3 số f(m),f(n),f(p) hoặc cùng dương, }m hoặc có 2 số cùng dấu nên:
Th1: f(m),f(n),f(p) cùng bằng 7 hoặc -7  loại vì phương trình f(x)-7=0 có 3 nghiệm
phân biệt
Th2: f (m)  f (n)  7 và f ( p)  7

Không mất tính tổng quát,giả sử m>n và m  p  n  p ta có: m,n là nghiệm pt:
x2  ax  b  7  0 và p là nghiệm pt: x2  ax  b  7  0 nên :
 n  p  2
m  n  a   n  m  9(l )
  pm  7
(n  p)(n  p  a)  14  (n  p)( p  m)  14  
(m  p)(m  p  a)  14 n  p  2
   n  m  9(l )
  p  m  7
Th3: f (m)  f (n)  7 và f ( p)  7 ,khiđó ho|n to|n tương tự ta có:

m  p  7 m  p  7
( p  n)(m  p)  14   hoặc 
pn  2  p  n  2
Do m,n,p  1;9 nên tìm được 4 bộ là: (a;b)= (11;17),(13;29),(7; 1),(9;7) .

Chú ý: Mọi cách giải đúng khác đều cho điểm tương ứng.

Đề 20
Câu 1.

a) Tìm m: y  x  2mx  3m và y  2 x  3 cắt nhau tại hai điểm phân biệt và


2

ho|nh độ dương
Yêu cầu bài toán  PT sau có hai nghiệm dương ph}n biệt

x2  2mx  3m  2 x  3  x 2  2(m  1) x  3m  3  0
 '  0

 3(m  1)  0
2(m  1)  0

 m  1
'  0  
 m  4
Kết hợp nghiệm, kết luận m  4

131
Page: Tài Liệu Môn Toán
Website: tailieumontoan.com

b) Giải bất phương trình:  x 2  8x  12  10  2 x


TXĐ:  x  8x  12  0  2  x  6
2

Nếu 5  x  6 thì  x 2  8x  12  0  10  2 x , bất phương trình nghiệm đúng với


mọi x: 5  x  6

10  2 x  0

Nếu 2  x  5   bất pt đã cho
  x  8 x  12  0

2

28
  x2  8x  12  4 x2  40 x  100  5 x 2  48 x  112  0  4  x 
5
Kết hợp nghiệm, trường hợp này ta có: 4  x  5
Tập nghiệm của bpt đã cho: (4;6]

3
Câu 2. 1) Giải phương trình: (4 x  x  3)  x 
3 3 3
(1)
2

2 y  2 x  3
3 3

Đặt y  4 x  x  3 . (1) có dạng: 


3
( I ) Khi đó nghiệm của (1) là x ứng

 4 x 3
 x  3  y
với (x;y) là nghiệm của (I)


2 y  2 x  3
3 3 
2 y  2 x  3(2)
3 3

(I)    
     ( x  y )(2 x  2 xy  2 y  1)  0(3)

3 3 2 2
 2 x 2 y ( x y ) 0

3
TH1: y = -x kết hợp(2), có nghiệm của (1): x   3
4
TH2: 2 x 2  2 xy  2 y 2  1  0;  'x  2  3 y 2 . Nếu có nghiệm thì y  2 . Tương tự cũng
3
3
2  2 8 2
có x  . Khi đó VT (2)  4     3 . Chứng tỏ TH2 vô nghiệm. KL (1) có 1
3  3  3 3

nghiệm x   3 3
4

2) Giải phương trình: 2 x  11x  23  4 x  1


2

ĐK: x  1 . (1)  2( x  6 x  9)  ( x  1  4 x  1  4)  0
2

2( x  3)2  ( x  1  2)2  0 (*)

132
Page: Tài Liệu Môn Toán
Website: tailieumontoan.com

 x  3  0
Do a  0(a) nên pt(*)  
2

 x  1  2  0
 x  3 . Vậy pt đã cho có 1 nghiệm x=3
Câu 3.
a) M (1;4) . Đg thẳng d qua M, d cắt trục hoành tại A; d cắt trục tung tại B. Tìm giá
trị nhỏ nhất của diện tích tam giác OAB( xA ; yB  0 )

x y
Giả sử A(a;0); B(0;b), a>0; b>0. PT đường thẳng AB:  1
a b
1 4 4 16
Vì AB qua M nên  11 2 1
a b ab ab
ab 1 4 1 a  2
  8;"  "     
2 a b 2 b  8
1 1
Diện tích tam giác vuông OAB( vuông ở O)là S  OA.OB  ab  8 . Vậy S nhỏ nhất
2 2
bằng 8 khi d qua A(2;0), B(0;8)
b)

A
M N
H

(C): ( x  2)  ( y  3)  9 ; A(1; 2) .  qua A,  cắt (C) tại M và N. Tìm giá trị nhỏ
2 2

nhất của độ d|i đoạn thẳng MN.


(C) có tâm I(2;-3), bán kính R=3. Có A nằm trong đường tròn(C) vì
IA2  (1  2)2  (2  3)2  2  9
Kẻ IH vuông góc với MN tại H ta có
IH 2  HN 2  IN 2  9  MN 2  4HN 2  4(9  IH 2 )
Mà IH  AH  IH  IA  2  MN 2  4(9  2)  28  MN  2 7

133
Page: Tài Liệu Môn Toán
Website: tailieumontoan.com

Vậy MN nhỏ nhất bằng 2 7 khi H trùng A hay MN vuông góc với IA tại A
Câu 4.
a) Chứng minh rằng tứ giác lồi ABCD là hình bình hành khi và chỉ khi
AB2  BC 2  CD2  DA2  AC 2  BD2
Tứ giác lồi ABCD là hình bình hành  AB  DC  AB  DC  0

 
2 2 2
 AB  DC  0  AB  DC  2 AB.DC  0

 AB2  DC 2  2 AB.( AC  AD)  0


 AB2  DC 2  ( AB2  AC 2  BC 2 )  ( AB2  AD2  BD2 )  0 (*)

   
2 2 2 2 2 2
( vì a  b  a  2a.b  b  2a.b  a  b  a  b )

(*)  AB  BC  CD  DA  AC  BD (Đpcm)
2 2 2 2 2 2

( Chú ý: nếu chỉ làm được 1 chiều thì cho 0,75 đ)


1 1 1
b) Tìm tất cả các tam giác ABC thỏa mãn: 2
 2  2 (1)
ha b c
Có a.ha  2S  bc sin A

1 a2 4R2
 2 2 2 2  2 2
ha b c sin A b c
(1)  b  c  4R  sin B  sin C  1
2 2 2 2 2

 1  cos2B  1  cos2C  2  cos2B  cos2C  0


 2cos( B  C )cos( B  C )  0
  
 B  C  2 hay A  2
  0  B  C   ;0  B  C   
 BC  
 2

Vậy tam giác ABC vuông ở A hoặc có B  C 
2
 a  b    b  c    c  a  ; a, b, c  0
2 2 2
2a 2b 2c
Câu 5. CMR :    3
bc ca ab a  b  c
2

134
Page: Tài Liệu Môn Toán
Website: tailieumontoan.com

2a 2b 2c a b a c bc ba c a c b


XétM= 1 1 1   
bc ca ab bc ca ab
1 1 1 1 1 1
 (a  b)(  )  (b  c)(  )  (c  a)(  )
bc ca ca ab ab bc
1 1 1
 ( a  b) 2  (b  c)2  (c  a ) 2
(b  c)(c  a) (c  a)(a  b) (a  b)(b  c)
1 4 4 1
Vì    ;
(b  c)(c  a) (a  b  2c) 2
(2a  2b  2c) 2
( a  b  c) 2
1 ( a  b) 2
(a  b)  0  (a  b)
2 2
 ;"  "  a  b
(b  c)(c  a) (a  b  c)2
L|m ho|n to|n tương tự với hai biểu thức còn lại

 a  b  b  c   c  a 
2 2 2

Suy ra M  (Đpcm); “=”  a  b  c


a  b  c
2

Lưu ý: Học sinh làm theo cách khác đúng vẫn cho điểm tối đa.

Đề 21

Câu 1.

a) Cho hàm số y  x  3x  2 và hàm số y   x  m . Tìm m để đồ thị các hàm số


2

đó cắt nhau tại hai điểm phân biệt A, B đồng thời trung điểm của đoạn thẳng AB
c{ch đều các trục tọa độ.
Yêu cầu bài toán  PT sau có hai nghiệm phân biệt
x2  3x  2   x  m hay x2  2 x  2  m  0 (*)có  '  0  m>1
xA  xB
Gọi x A ; x B là 2 nghiệm của (*), I l| trung điểm AB ta có x I   1;
2
yI  x I  m  m  1

Yêu cầu bài toán  yI  x I

 m  1  1  m  2;m  0

Kết hợp ĐK, kết luận m  2

135
Page: Tài Liệu Môn Toán
Website: tailieumontoan.com

1 1
b) Giải bất phương trình:   0 (1)
x  4x  3
2 2x  4

 x 2  4 x  3  0
TXĐ:   1  x  2;2  x  3
 x  2

(1)  1 1

x  4x  3
2 2x  4

Nếu 1  x  2 thì  x 2  4 x  3  0  2 x  4 , bất phương trình nghiệm đúng với mọi x:


1 x  2
2 x  4  0
Nếu 2  x  3  
  x  4 x  3  0
2

bất pt đã cho  2x  4  x 2  4x  3
 4 x2  16 x  16   x2  4 x  3  5x2  20 x  19  0
5 5
x  2 ;x  2
5 5
5
Kết hợp nghiệm, trường hợp này ta có: 2  x3
5
5
Tập nghiệm của bpt đã cho: (1;2)  (2  ;3)
5
Câu 2.
1) Trong mặt phẳng tọa độ Oxy cho tam giác ABC có B(1;2) . Đường thẳng  là
đường phân giác trong của góc A có phương trình 2x  y  1  0 ; khoảng cách từ
C đến  gấp 3 lần khoảng cách từ B đến  . Tìm tọa độ của A và C biết C nằm
trên trục tung.
3 y 1 y 1 9
D(B;  )= ; C(0:y0) ; D(C;  )= 0 , theo bài ra ta có 0   y0  10; y0  8
5 5 5 5
Vẽ hệ trục tọa độ, điểm B, chú ý C khác phía B đối với  suy ra C(0;-8)
Gọi B’(a;b) l| điểm đối xứng với B qua  thì B’nằm trên AC.
Do BB'  u   (1; 2) nên ta có: a  2b  3  0 ;

Trung điểm I của BB’ phải thuộc  nên có: 2a  b  2  0


Từ đó ta có: a= -7/5; b=4/5

136
Page: Tài Liệu Môn Toán
Website: tailieumontoan.com

3
Theo định lý Ta - Let suy ra CA  CB'
2
 7 44 
A(x; y);CA   x; y  8 ;CB'    ; 
 5 5 
21 26
Từ đó suy ra A( ; ) ;C(0;-8)
10 5
2) Xét các tam giác vuông ABC vuông ở A, gọi  là góc giữa hai đường trung tuyến
BM và CN của tam giác. Chứng minh
B
3
rằng sin  
5
Gọi a, b v| c tương ứng l| độ dài các cạnh
N
đối diện các góc A, B và C của tam giác. Có
G
c2
CN  b 
2 2

4
A C
b2
BM  c 
2 2
M
4
BG 2  CG 2  BC2
Gọi G là trọng tâm tam giác ABC, ta có cos BGC 
2BG.CG
2(b 2  c2 ) 2(b 2  c2 )
= ; Do đó cos 
(4c2  b 2 )(4b 2  c2 ) (4c2  b 2 )(4b 2  c2 )

5(b2  c2 )
Có (4c2  b2 )(4b 2  c2 )  ;"  "  4c2  b 2  4b 2  c 2  b  c
2
2(b2  c2 ) 2(b 2  c2 ).2 4
Do đó cos   
(4c2  b2 )(4b2  c2 ) 5(b 2  c2 ) 5

3
Hay sin   1  cos 2  . Dấu bằng có khi tam gi{c vuông c}n đỉnh A
5
Câu 3.
2 1
a) Cho tam giác ABC. Gọi D, E lần lượt là các BD  BC; AE  AC . Tìm vị trí
3 4
của điểm K trên AD sao cho 3 điểm B, K, E thẳng hàng.
1 1 3 A
Vì AE  AC  BE  BC  BA(1)
4 4 4
E

Giả sử AK  x.AD  BK  x.BD  (1  x)BA K

137
B D C
Page: Tài Liệu Môn Toán
Website: tailieumontoan.com

2 2x
Mà BD  BC nên AK  x.AD  BK  BD  (1  x)BA
3 3
Vì B, K, E thẳng hàng(B  E ) nên có m sao cho BK  mBE
m 3m 2x
Do đó có: BC  BA  BC  (1  x)BA
4 4 3
 m 2x   3m 
Hay    BC  1  x   BA  0
4 3   4 

Do BC; BA không cùng phương nên


m 2x 3m 1 8
  0 &1  x   0 Từ đó suy ra x  ; m 
4 3 4 3 9
1
Vậy AK  AD
3
b) Cho tam giác ABC vuông ở A; BC = a; CA = b; AB = c.
X{c định điểm I thỏa mãn hệ thức: 2a 2 IA  b2 IB  c2 IC  0 ; Tìm điểm M: biểu thức
2a 2MA2  b2MB2  c2MC2 đạt giá trị lớn nhất.

Kẻ đường cao AH, ta có b2  a.CH;c2  a.BH nên A

b2 .BH  c2 .CH . Do đó:

B H C

b .BH  c .CH  0
2 2

Suy ra b2 .IB  c2 .IC  b2 .IH  c2 .IH  a 2 .IH


Kết hợp giả thiết suy ra 2a 2 .IA  a 2 .IH hay 2.IA  IH
Do đó điểm I thỏa mãn gt là I thỏa mãn A l| trung điểm IH

Với x, y, z tùy ý thỏa mãn: x.IA  y.IB  z.IC  0 (*) bình phương vô hướng 2 vế (*), chú ý
rằng 2IA.IB  IA2  IB2  AB2 ta có:
(x.IA2  y.IB2  z.IC2 )(x  y  z)  xyc2  xzb2  yza 2

Từ đó có (2a 2 .IA2  b2 .IB2  c2 .IC2 )  3b2c2

Mặt khác xMA2  x(IA  IM)2  x(IM2  IA2  2IA.IM)

138
Page: Tài Liệu Môn Toán
Website: tailieumontoan.com

Tương tự cho yMB2; zMC2 rồi cộng c{c đẳng thức đó lại ta có
xMA2  yMB2  zMC2  (x  y  z)IM2  xIA2  yIB2  zIC2
Thay số có:
2a 2 MA2  b2 MB2  c2 MC2  a 2 IM2  3b2c2  3b2c2
Dấu bằng xảy ra khi M trùng I
Câu 4.


1) Giải phương trình: 1   6 x  2  2 x  1  2 5 x  4 x
2 2

(*)
1 1
ĐK: x  ;x  
2 2

(*)  (3x  1)2  (2x 2  1)  2(3x  1) 2x 2 1 1  (3x  1) 2  (2x 2 1)  (10x 2  8x)

 
2
 3x  1  2x 2  1   x  1
2

 2x 2  1  2x  2(a)

 2x 2  1  4x(b)

4  6
Giải(a) v| đối chiếu ĐK có 1 nghiệm x 
2
4  6
Giải (b) vô nghiệm. Kết luận (*) có 1 nghiệm x 
2
b) Cho x, y, z là các số thực dương thỏa mãn x  y  z  xyz . Chứng minh rằng:

1  1  x2 1  1  y 2 1  1  z 2
   xyz (I)
x y z
1 1 1
Giả thiết suy ra:    1 . Ta Có:
xy yz zx
1 x2 1 1 1 1  1 1  1 1  1  2 1 1 
 2              ;"  "  y  z
x x xy yz zx  x y  x z  2  x y z 
Viết hai BĐT tương tự rồi cộng lại ta được:

1  1  x2 1  1  y 2 1  1  z 2  1 1 1
   3     ;"  "  x  y  z
x y z x y z

139
Page: Tài Liệu Môn Toán
Website: tailieumontoan.com

 1 1 1
Ta sẽ CM: 3      xyz  3  xy  yz  zx    xyz    x  y  z 
2 2

x y z
  x  y    y  z    z  x   0 Điều n|y luông đúng
2 2 2

Dấu bằng có khi và chỉ khi x=y=z


Vậy (I) được CM, dấu bằng có khi và chỉ khi x=y=z= 3

Lưu ý: Học sinh làm theo cách khác đúng vẫn cho điểm tối đa.

Đề 22
Câu 1.
Cho parabol (P): y  x 2 – 2 x  4 v| c{c đường thẳng (dm): y  3x  2m  1 (m là
tham số)
1) Biện luận số giao điểm của (P) và (dm) theo tham số m.
Xét phương trình ho|nh độ: x2 – 2x + 4 = 3x + 2m + 1
 x2 – 5x + 3 – 2m = 0 (1). Ta có:  = 8m + 13
13
+) Nếu m   ( >0) thì (1) có hai nghiệm phân biệt, do đó (dm) cắt (P) tại hai điểm
8
phân biệt.
13
+) Nếu m      0  thì (1) có 1 nghiệm kép, do đó (dm) cắt (P) tại một điểm.
8
13
+) Nếu m      0  thì (1) vô nghiệm, do đó (dm) không cắt (P).
8
2) Khi (dm) cắt (P) tại hai điểm A, B (A và B có thể trùng nhau), tìm tập hợp trung điểm I
của AB khi m thay đổi.
13
+) (dm) cắt (P) tại hai điểm A, B (A và B có thể trùng nhau)  m   .
8
+) Gọi hai nghiệm của phương trình (1) l| x1, x2 thì ta có: x1 + x2 = 5, x1.x2 = 3 – 2m

140
Page: Tài Liệu Môn Toán
Website: tailieumontoan.com

+) x1, x2 cũng l| ho|nh độ giao điểm A, B nên trung điểm I của AB có tọa độ:
 x x 5
 xI  1 2 
 2 2

 y  3x  2m  1  2m  17


I I
2
13 y 17 13 21
+) Do m   nên ta có: I     yI  .
8 2 4 8 4
5 21
Kết luận: Tập hợp điểm I là phần đường thẳng x  ứng với y  .
2 4
Câu 2.

1) Giải bất phương trình:  2x  5  x 2  x  25  x2  5x  6  0

x  3
Điều kiện x{c định đúng: 
x  2
*) Nếu x = 3 hoặc x = 2 thì bất phương trình nghiệm đúng.
x  3
thì bất PT đã cho  2 x  5  x  x  25  0 (a)
2
*) Nếu 
x  2

(a)  x 2  x  25  2 x  5
 2 x  5  0 (Do x 2  x  25  0) (1)

  2 x  5  0
  x 2  x  25  4 x 2  20 x  25 (2)

+) Giải (1) và kết hợp nghiệm ta được: x < 2.

 5
 5 x
x   2
+) Giải (2): (2)   2  
3x  19 x  0
2 0  x  19
  3

Kết hợp nghiệm ta được:


19
3 x 
3
141
Page: Tài Liệu Môn Toán
Website: tailieumontoan.com

 19 
Kết luận: Bất PT đã cho có tập nghiệm là: S   ;2  3; 
 3
2. 3 x  3 y  3 5(8 x  y ) (1)
2) Giải hệ phương trình:  2
 x  y  2 x  4 y  31  0 (2)
2


3 y  3 5y y  0
TH 1 x  0 : Thay vào HPT ta có   , không thỏa mãn.

 y 2
 4 y  31  0  31  0

y 3 y
TH 2 x  0 : Chia hai vế của pt (1) cho 3
x ta được: 2  3  5(8  )
x x
y
Đặt t  3 ta được phương trình: 2  t  3 5(8  t 3 )  (2  t )3  5(8  t 3 )
x
 (t  2)(4t 2  14t  16)  0  t  2
t  2  y  8x . Thế vào (2) ta
được : x2  64 x2  2 x  32 x  31  0  65x2  34 x  31  0

x  1
 31 248 
 31 . Vậy hệ có 2 nghiệm là : (1; 8);   ; 
x    65 65 
 65

Câu 3.
1) Trong mặt phẳng với hệ tọa độ Oxy, cho tam giác ABC cân tại A có phương trình
đường thẳng chứa cạnh AB là: x  2 y  2  0 , phương trình đường thẳng chứa cạnh AC
là: 2x + y + 1 = 0. Điểm M(1;2) thuộc đoạn thẳng BC. Tìm tọa độ điểm D sao cho DB.DC
có giá trị nhỏ nhất.
+) C{c đường phân giác góc A là tập hợp c{c điểm c{ch đều AB, AC nên có phương
A
trình:
x  2y  2 2x  y  1 x  y  3  0
 
5 5 3 x  3 y  1  0 B M C

la
+) Do Δ ABC cân tại A nên phân giác trong ( la )

của góc A vuông góc với BC.

+) TH1: (la ) : x  y  3  0 , khi đó BC đi qua M(1;2) và có vtpt n1  (1;1) ;

142
Page: Tài Liệu Môn Toán
Website: tailieumontoan.com

 Phương trình BC : x  y  3  0

x  2 y  2  0 x  4
Tọa độ B là nghiệm hệ PT:    B(4; 1)
x  y  3  0  y  1
2 x  y  1  0  x  4
Tọa độ C là nghiệm hệ PT:    C (4;7)
x  y  3  0 y  7
3
Khi đó MB   3; 3 ; MC   5;5  MB   MC  B,C nằm về hai phía ( la ) ( thỏa
5
mãn)
+) TH2: (la ) : 3x  3y  1  0 , khi đó BC đi qua M(1;2) v| có vtpt n2  (1; 1)

 Phương trình BC : x  y  1  0

x  2 y  2  0 x  0
Tọa độ B là nghiệm hệ PT:    B(0;1)
x  y 1  0 y 1
 2
 x
2 x  y  1  0  3 2 1
Tọa độ C là nghiệm hệ PT:    C ( ; )
x  y 1  0 y  1 3 3

 3
 5 5
Khi đó MB   1; 1 ; MC    ;   cùng hướng (loại)
 3 3
Với B(4; 1) ; C  4;7  . Gọi D  x; y   DB   4  x; 1  y , DC  4  x;7  y 

x  0
 DB.DC  x 2  y 2  6 y  23  x 2   y  3  32  32 . Dấu ''  ''  
2

y  3
Vậy D(0;3) thì DB.DC nhỏ nhất bằng -32.

2) Cho tứ giác ABCD, hai điểm M, N thay đổi sao cho AM  k AB; DN  k DC (0  k  1) .
Gọi I l| điểm thỏa mãn 3IM   2IN . Tìm tập hợp c{c điểm I khi M, N thay đổi.
Gọi E, F l| c{c điểm thỏa mãn: 3EA   2ED; 3FB   2 FC (*)
2 2
Ta có: EI  EA+ AM  MI =  AD  k AB  MN (1)
5 5
3 3
EI  ED+ DN  NI = AD  k DC  MN (2)
5 5
143
Page: Tài Liệu Môn Toán
Website: tailieumontoan.com

Nhân hai vế (1) với 3, nhân hai vế (2) với 2 rồi cộng lại ta được :

5EI  k (3 AB  2DC ) (3)


Tương tự:
2 2
EF  EA+ AB  BF =  AD  AB  BC (4)
5 5
3 3
EF  ED+ DC  CF = AD  DC  BC (5)
5 5
Từ (4) và (5) ta có:

5EF  3 AB  2DC (6)


Từ (3) v| (6) ta được:

EI  kEF (7)
Ngược lại, với mỗi I’ thỏa mãn hệ thức

EI '  mEF (0  m  1)
Gọi M, N, I l| c{c điểm thỏa mãn:

AM  mAB; DN  mDC ; 3IM   2IN


Theo chứng minh trên thì

EI  mEF
Suy ra: I’ trùng với I.
E, F cố định do thỏa mãn (*) và 0  k  1 nên tập hợp c{c điểm I l| đoạn EF.
Câu 4.
1) Tam giác ABC có S  b2  (a  c)2 với S là diện tích tam giác; a = BC; b = AC; c = AB.
Tính tanB.
Ta có:
1
S  b2  (a  c)2  ac sin B  a 2  c 2  2ac cos B  a 2  c 2  2ac
2
1 1
 ac sin B  2ac(1  cos B)  sin B  4(1  cos B)  cos B  1  sin B(*)
2 4
Ta có

144
Page: Tài Liệu Môn Toán
Website: tailieumontoan.com

2
 1  17 1
sin B  cos B  1  sin B  1  sin B   1  sin 2 B  sin B  0
2 2 2

 4  16 2
8
 sin B  (do sinB > 0)
17
15 8
Kết hợp với (*) ta được: cos B   tan B  .
17 15
2) Cho a, b, c là các số thực dương thỏa mãn: a 2  b2  c2  3
Tìm giá trị lớn nhất của biểu thức:
ab bc ca
M  
5a  10ab  10b
2 2
5b  10bc  10c
2 2
5c  10ca  10a 2
2

Ta có: 5a 2  10ab  10b2  (2a  3b)2  (a  b)2  (2a  3b)2 , dấu “=” có khi a=b
ab ab
Suy ra : 5a 2  10ab  10b2  2a  3b   .
5a  10ab  10b
2 2 2a  3b
ab 3a  2b
Ta chứng minh:  (*)
2a  3b 25
Thật vậy : (*)  25ab  (2a  3b)(3a  2b)  6(a  b) 2  0 (luôn đúng) ; dấu “=” có khi a=b.
ab 3a  2b
Do đó : 
5a 2  10ab  10b2 25
bc 3b  2c ca 3c  2a
Tương tự:  ; 
5b2  10bc  10c 2 25 5c 2  10ca  10a 2 25
3a  2b 3b  2c 3c  2a 1
Cộng c{c BĐT trên ta được: M     (a  b  c)
25 25 25 5
Ta có:
(a  b  c)2  a 2  b2  c 2  (2ab  2bc  2ca)  a 2  b2  c 2  (a 2  b2 )  (b2  c 2 )  (c 2  a 2 )
= 3a2  3b2  3c2  9 . Do đó: a  b  c  3 .
3 3
Vậy M  , giá trị lớn nhất của M bằng khi a = b = c = 1.
5 5

Đề 23

145
Page: Tài Liệu Môn Toán
Website: tailieumontoan.com

Câu 1. 1) Cho parabol (P): y   x v| đường thẳng (d) đi qua điểm I (0; 1) và có hệ số
2

góc là k . Gọi A và B l| c{c giao điểm của (P) và (d). Giả sử A, B lần lượt có ho|nh độ là
x1; x2 .
1) Tìm k để trung điểm của đoạn thẳng AB nằm trên trục tung.
+ Đường thẳng (d) có pt: y kx 1
+ PT tương giao (d) và (P): x2 kx 1 x2 kx 1 0(*)
+ (*) luôn có 2 nghiệm phân biệt x1; x2 vì k2 4 0 k
x1  x2 k
+ Trung điểm M của AB có ho|nh độ là  ; M nằm trên trục tung
2 2
k
0k 0
2
2)

Chứng minh rằng x1  x2  2  k  R 


3 3

Theo Vi et có: x1  x2  k , x1 x2  1

Ta có: x13  x23  ( x1  x2 ) ( x1  x2 )2  x1 x2  = x1  x2 . ( x1  x2 )2  x1 x2

2 2
Có x1 x2 x1 x2 4 x1x2 k2 4

 x13  x23 = k 2  4(k 2  1)  2 , k  R . Đẳng thức xảy ra khi k = 0

Câu 2.

1) Giải phương trình: 3x  1  5x  4  3x2  x  3 (1)


1
Điều kiện: x  
3
(1)   3x  1  1    
5 x  4  2  3x 2  x

3x 5x
  x  3x 1
3x  1  1 5 x  4  2
 x  0(TM )

 3 5
  3x 1 (*)
 3x  1  1 5x  4  2
146
Page: Tài Liệu Môn Toán
Website: tailieumontoan.com

Với x=1: VT(*)= 2=VP(*) nên x=1 là một nghiệm của (*)
Nếu x>1 thì VT(*)<2<VP(*)
Nếu x<1 thì VT(*)>2>VP(*). Vậy (1) có 2 nghiệm x=0; x=1


 x  x y  xy  xy  y  1(1)
2 3 2

2) Giải hệ phương trình:  (*)



 x 4
 y 2
 xy (2 x  1)  1(2)

( x  y )  xy ( x  y )  xy  1
2 2

(*)   2
 
2

 x  y  xy  1

a  x 2  y a  ab  b  1
Đặt  . Hệ trở thành:  2 (*)
b  xy a  b  1

 a  a  2a  0
3 2

a(a  a  2)  0
2

Hệ (*)   

b  1  a 2
b  1  a

2

Từ đó tìm ra (a; b) (0; 1); (1; 0); (2;  3)

 x2  y  0
Với (a; b)  (0; 1) ta có hệ   x  y  1.
 xy  1

 x2  y  1
Với (a; b)  (1; 0) ta có hệ   ( x; y )  (0; 1);(1;0);(1;0) .
 xy  0
Với (a; b)  ( 2; 3) ta có hệ
 3  3
 x 2  y  2 y   y  
  x  x  x  1; y  3 .
 xy   3 x  2x  3  0
3 ( x  1)( x  x  3)  0
2
 
Kết luận: Hệ có 5 nghiệm ( x; y) (1; 1);(0; 1);(1; 0);( 1; 0);( 1; 3)  .

Câu 3. 1) Trong mặt phẳng tọa độ Oxy cho tam giác ABC có đỉnh A(2;6) , ch}n đường
 3
phân giác trong kẻ từ đỉnh A l| điểm D  2;   , t}m đường tròn ngoại tiếp tam giác
 2
 1 
ABC l| điểm I   ;1 . Viết phương trình của đường thẳng BC.
 2 
Đường tròn (C) ngoại tiếp tam giác ABC có tâm I và bán kính IA

147
Page: Tài Liệu Môn Toán
Website: tailieumontoan.com

 15 
Đường thẳng AD đi qua A v| có VTCP AD  0;  
 2
 n 1;0 l| véc tơ ph{p tuyến của AD
PT đường thẳng AD là: x  2

A'  AD  (C); A '  A  A’ thuộc AD v| IA’=IA, Tìm được A'  2; 4 

A’ l| trung điểm cung BC không chứa A nên IA’ BC


 5 
đường thẳng BC đi qua D v| có A ' I    ;5  là vecto pháp tuyến
 2 
Từ đó viết được pt đường thẳng BC là: x  2 y  5  0

2) Cho tam giác ABC có BC  a; CA  b; BA  c (b ≠ c) và diện tích là S . Kí hiệu


ma ; mb ; mc lần lượt l| độ dài của c{c đường trung tuyến kẻ từ c{c đỉnh A, B, C. Biết
rằng 2ma  mb  mc (*)
2 2 2

2
a) Chứng minh rằng a 4S.cotA
Viết được công thức các trung tuyến

2 2 a2 c2 a2 b2 a2 b2 c2
(*) b c
2 2 4 2 4
b2 c2 2a 2 (**)
cos A
Ta có 4S .cot A 2bc.sin A.
sin A
2bc.cos A b2 c2 a2
Từ (**) b2 c2 a2 a 2 Hay 4S.cotA a2
2b) Gọi O và G lần lượt l| t}m đường tròn ngoại tiếp và trọng tâm tam giác ABC; M là
trung điểm của BC. Chứng minh rằng góc MGO không nhọn.

Ta sẽ chứng minh GO.GM 0 OG.GM 0


Ta có
3OG  OA  OB  OC ; 6GM =2AM  AB  AC  OB  OC  2OA


 3OG.6GM  OA  OB  OC . OB  OC  2OA  
148
Page: Tài Liệu Môn Toán
Website: tailieumontoan.com

= OB2  OC 2  2OA2  2OB.OC  OA.OC  OA.OB


= 2OB.OC  OA.OC  OA.OB
* Mặt khác ta có

 
2
BC 2  OC  OB  OB 2  OC 2  2OB.OC
( trong đó R= OA = OB = OC ).
 2OB.OC  2 R  a
2 2

Tương tự có 2OAOC
.  2R2  b2 ; 2OAOB
.  2R 2  c 2 .

b2  c 2
Vậy 18.OG.GM   a 2  0  OG.GM  0 ( do có (**))
2

3 3
Câu 4. Cho a; b; c là các số thực dương thay đổi và thỏa mãn a  b  c  . Tìm giá
2
1 1 1
trị lớn nhất của biểu thức M   2  2 .
a  b  3 b  c  3 c  a2  3
2 2 2

* Bđt phụ: Cho các số thực x, y, z > 0, a, b, c là các số thực bất kì. Khi đó

a 2 b2 c 2  a  b  c 
2

   (*)
x y z x yz
a b c
Dấu bằng xảy ra khi   .
x y z
+ Dễ thấy bđt trên suy ra từ bđt Bunhia
* Vào bài chính
Ta sẽ chứng minh
1 1 1 1
M  2 2  2  .
a b 3 b c 3 c a 3 2
2 2 2

1 1  1 1  1 1  1
  2   2 2   2 
3 a b 3 3 b c 3 3 c a 3 2
2 2

a 2  b2 b2  c2 c2  a2 3
P 2   
a b 3 b c 3 c a 3 2
2 2 2 2 2

Giả sử a  b  c .

 a  b   a  b
2 2
a 2  b2
Biến đổi 2  .
 
a  b  3 2 a 2  b2  3 2 a 2  b2  3
2
 
Biến đổi tương tự với 2 số hạng còn lại của P.

149
Page: Tài Liệu Môn Toán
Website: tailieumontoan.com

Sau đó {p dung bđt (*) ta có:

a  b  b  c  c  a a  b  b  c  a  c
2 2

P 

4 a 2  b 2  c 2  18  
4 a 2  b 2  c 2  18
4a  b  c  4a  c 2a  b  c  2a  c
2 2 2 2

P P

4 a 2  b 2  c 2  18  
2 a 2  b2  c2  9 
Ta sẽ chứng minh

2a  b  c  2a  c
2 2


3

 4  a  b  c   4  a  c   6 a 2  b2  c 2  27
2 2


2 a b c 9
2 2 2
 2
2

 4  a  b  c   4  a  c   6 a 2  b2  c2  2  a  b  c 
2
 2

 2  a  b  c  2  a  c   3 a   a  b  c
2 2 2
2
 b2  c2
 b 2  ab  bc  ca  0   a  b  b  c   0

Bđt cuối cùng đúng, suy ra đpcm.


Lưu ý: Học sinh l|m theo c{ch kh{c đúng vẫn cho điểm tối đa.
Đề 24
Bài 1: (4,0 điểm) Câu hỏi đề xuất của trường THPT chuyên Trần Phú- Hải Phòng.
Giải phương trình x2  2 2 x  7  2 3  2 x  5 .
7 3
LG: Điều kiện   x  .
2 2
Phương trình ban đầu tương đương với phương trình
6  2  
2 x  7  2 3  2 x  2  x2 1 
4  4x 4  4x
   x2  1
3  2x  7 3  2x 1
x  1
 4 4
    x  1 1
 3  2 x  7 3  2x 1

 4   4 
Phương trình (1)    1    1   x  3
 3  2x  7   3  2x 1 
2 x  6 2x  6
   x  3
 3  2x  7 1  2x  7   2 x  3  1  2 x  3  3 
150
Page: Tài Liệu Môn Toán
Website: tailieumontoan.com

 x  3

 1  2 
 2 2


 3  2x  7 1 2x  7
    2 x  3  1  2 x  3  3 
2
Phương trình (2) vô nghiệm vì VT  .
3  
2x  7 1  2x  7 
2 2
Lại có:   VP
3  
2x  7 1  2x  7  3

Vậy phương trình có nghiệm x  1 hoặc x  3 .


7 3
Cách khác. Điều kiện   x  .
2 2
3 3
Với điều kiện trên ta có: x  5  2 2 x  7  và 2 2 x  3  x  3    0 .
2 2
1 1 2 1
Do đó :  1  1    0
x  5  2 2 x  7 2 2 x  3  x  3 3 3

   
PT  x  5  2 2 x  7  2 2 x  3  x  3  x 2  2 x  3  0

x2  2 x  3 x2  2x  3
   x2  2 x  3  0
x  5  2 2 x  7 2 2 x  3  x  3
(Mỗi nhân liên hợp cho 0,5 điểm)
 1 1 
 ( x 2  2 x  3)    1  0
 x  5  2 2 x  7 2 2 x  3  x  3 
x  1

 x  3
Vậy phương trình có nghiệm x  1 hoặc x  3 .
Bài 2: (4,0 điểm) Câu hỏi đề xuất của trường THPT chuyên Lam Sơn- Thanh Hóa.
Cho tam giác ABC ( AB  AC ) nhọn, không cân nội tiếp đường tròn  O  . Các đường cao
AD , BE và CF cắt nhau tại H . Gọi M là trung điểm cạnh BC . Đường tròn  J  ngoại
tiếp tam giác AEF cắt đường tròn  O  tại điểm thứ hai là K ( K  A) . Đường thẳng
AM cắt đường tròn  J  tại điểm thứ hai là Q (Q  A). EF cắt AD tại P . Đoạn PM
cắt đường tròn  J  tại N .

151
Page: Tài Liệu Môn Toán
Website: tailieumontoan.com

a) Chứng minh các đường thẳng KF , EQ và BC đồng quy hoặc song song và ba điểm K,
P, Q thẳng hàng
b) Chứng minh rằng đường tròn ngoại tiếp tam giác DMN và đường tròn ngoại tiếp tam
giác BNC tiếp xúc nhau.

E
K J
S

P
Q
F
N
H

C
L B D M

A'

a) Gọi A’ l| điểm đối xứng với H qua M, suy ra BHCA’ là hình bình hành.

Do đó A ' C BH ; A ' B CH , suy ra A ' CA  A ' BA  900  AA ' l| đường kính của đường
tròn ngoại tiếp tam giác ABC. Suy ra A ' K  AK (1).
Dễ thấy AH l| đường kính của đường tròn (J), suy ra HK  AK (2) .

Từ (1) và (2) suy ra K, H, A’ thẳng hàng.


Mà H, M, A’ thẳng hàng nên suy ra K, H, M, A’ thẳng hàng.
Gọi L l| giao điểm của AK và BC.
Từ c{c kết quả trên v| giả thiết, suy ra H l| trực t}m của tam gi{c ALM, suy ra LH
vuông góc với AM, gọi Q '  LH AM  Q '  ( J )  Q '  Q. suy ra c{c tứ gi{c ABDE,
ALDQ nội tiếp, suy ra HL.HQ  HA.HD  HB.HE  LBQE nội tiếp.

Ta có: AF . AB  AE. AC  AK . AL  AH . AD  AQ. AM . Suy ra các tứ giác KLBF, CMQE nội


tiếp.
Như vậy: LB l| trục đẳng phương của hai đường tròn (LBQE) và (KLBF);
KF l| trục đẳng phương của hai đường tròn (KLBF) và (J);
EQ l| trục đẳng phương của hai đường tròn (J) và (LBQE).

152
Page: Tài Liệu Môn Toán
Website: tailieumontoan.com

Do đó ba đường thẳng KF, EQ và BC đồng quy hoặc song song.


EF l| trục đẳng phương của hai đường tròn (BC) và (J)
KQ l| trục đẳng phương của hai đường tròn (J) và (LM)
PA/( LM )  PA/( BC ) nên A thuộc trục đẳng phương của (LM) và (BC). Do AD vuông góc với
đường nối t}m hai đường tròn (LM) và (BC) nên AD l| trục đẳng phương của hai
đường tròn (LM) và (BC). Lại có, P l| giao điểm của EF với AD nên suy ra P thuộc KQ.
Cách khác : Ta có AF . AB  AE. AC  AK .AL  AQ.AM  AF .AB  AH .AD ,

Qua phép nghịch đảo  ( A, AH . AD) , tâm A phương tích k  AH . AD :

Đường thẳng KF biến th|nh đường tròn (ABL);


đường thẳng EQ biến th|nh đường tròn (ACM);
đường thẳng BC biến th|nh đường tròn (AEF).
Ba đường tròn (ABL); (ACM); (AEF) có chung nhau điểm A.
Do đó trục đẳng phương của ba đường tròn đó đồng qui tại A hoặc trùng nhau.
Vậy ba đường thẳng KF, EQ và BC song song hoặc đồng quy.
b) Ta có: AK là trục đẳng phương của hai đường tròn (O) và (J);
EF là trục đẳng phương của hai đường tròn (J) và (BFEC);
BC là trục đẳng phương của hai đường tròn (BFEC) và (O), mà AK cắt BC tại L, suy ra
AK, EF, BC đồng quy tại L.
Ta có M là tâm của đường tròn (BFEC), suy ra MJ  EF, kết hợp với JD  LM . Suy ra P
là trực tâm tam giác JLM. Do đó MP  JL . Gọi S l| giao điểm của JL và MP, ta có tứ giác
LDPS nội tiếp, suy ra JS.JL  JP.JD (3)
Xét tứ giác toàn phần AEHFBC, ta có  A, H , P, D   1 , mà J l| trung điểm AH nên theo
hệ thức Newton suy ra JH 2  JP.JD (4). Từ (3) và (4) suy ra JS.JL  JH 2  JN 2 , mà
NS  JL suy ra LN vuông góc với JN hay LN là tiếp tuyến của (J). Suy ra
LN 2  LK .LA  LB.LC  LN là tiếp tuyến của đường tròn (BNC) (5).

Từ AKM  ADM  900  4 điểm A, K, D, M cùng thuộc một đường tròn, suy ra
LN 2  LK .LA  LD.LM  LN là tiếp tuyến của đường tròn ngoại tiếp tam giác MND (6).
Từ (5) v| (6) suy ra hai đường tròn (BNC) và (MND) tiếp xúc nhau tại N (đpcm).

Bài 3: (4,0 điểm)

153
Page: Tài Liệu Môn Toán
Website: tailieumontoan.com

(a  b)(b  c)(c  a)
Tìm tất cả các bộ ba số nguyên (a, b, c) sao cho số  2 là một lũy thừa
2
của 20162017 (Một lũy thừa của 20162017 là một số có dạng 20162017 n với n là một số
nguyên không âm).
LG: Giả sử a, b, c là các số nguyên và n là một số nguyên dương sao cho

(a  b)(b  c)(c  a)  4  2.20162017 n .


Đặt a  b   x ; b  c   y và ta viết lại phương trình trên như sau

xy( x  y)  4  2.20162017 n (1)

Nếu n  1 thì vế phải của (1) chia hết cho 7, vì thế ta có


xy( x  y)  4  0 (mod 7) .
Gọi u, v 3, 2, 1,0,1, 2,3  thỏa mãn x  u  mod 7  , y  v  mod 7 

Ta có uv(u  v)  4  0 (mod 7) (2). Từ (2) ta được u  0, v  0, u  v  0 . Giả sử u  v . Khi đó


ta xét c{c trường hợp sau:
+) v  3  u 3, 2, 1,1, 2 thử thấy không thỏa mãn (2)

+) v  2  u 2, 1,1,3 thử thấy không thỏa mãn (2)

+) v  1  u 1, 2,3 thử thấy không thỏa mãn (2)

+) v  1  u 1, 2,3 thử thấy không thỏa mãn (2)

+) v  2  u 2,3 thử thấy không thỏa mãn (2)

+) v  3  u  3 thử thấy không thỏa mãn (2)

Chú ý: ( Hướng khác học sinh có thể làm).


Để chứng minh xy( x  y)  4  0 (mod 7) không xảy ra ta có thể chứng minh như sau: từ
xy( x  y)  4  0 (mod7) suy ra 3xy( x  y)  2 (mod7) hay
( x  y)3  x3  y3  2 (mod 7) (3)
Để ý rằng với mọi số nguyên k , ta có k 3  1;0;1 (mod 7) .

Từ (3) suy ra một trong các số ( x  y)3 , x3 và y 3 phải có số chia hết cho 7. Do 7 là số
nguyên tố nên một trong các số x  y, x, y phải có số chia hết cho 7. Suy ra xy( x  y) chia
hết cho 7 . Đ}y l| một điều mâu thuẫn xy( x  y)  4  0 (mod 7) .

Vì vậy, chỉ có thể là n  0 . Khi đó

154
Page: Tài Liệu Môn Toán
Website: tailieumontoan.com

xy ( x  y )  4  2  xy ( x  y )  2
 xy ( x  y )  1.(2)  (2).1  (1).2  2.(1)

Xét c{c trường hợp sau:


 xy  1
  x  y  1 .
 x  y  2
 xy  2 x  2  x  1
  hoặc  .
 x  y  1  y  1 y  2
 xy  1
 (không có nghiệm nguyên)
x  y  2
 xy  2
 (vô nghiệm)
 x  y  1
Vậy bộ ba số thỏa mãn yêu cầu bài toán là (a, b, c)  (k  2, k  1, k ) (với k  ) cùng các
hoán vị.

Bài 4: (4,0 điểm) Câu hỏi đề xuất của trường THPT chuyên ĐHSP Hà Nội.
bc ca ab  1 1 1 
Cho a, b, c là ba số thực dương thỏa mãn    2     . Chứng
a b c  ab bc ca 
minh rằng: a 2  b2  c2  3  2(ab  bc  ca) .

LG: Giả thiết tương đương với


bc ca a b  1 1 1 
1 1 1  2     3
a b c  ab bc ca 
1 1 1 2 
 a  b  c    3
 a b c abc 
 (a  b  c)(ab  bc  ac  2)  3abc

Theo bất đẳng thức AM-GM ta có:


( a  b  c )3
(a  b  c)(ab  bc  ac  2)  3abc 
9
( a  b  c) 2
Do đó: ab  bc  ac  2   18  9(ab  bc  ca)  (a  b  c)2
9
9 (a  b  c)2 7(ab  bc  ca)  (a 2  b 2  c 2 )
 3  (ab  bc  ca)  
6 6 6

155
Page: Tài Liệu Môn Toán
Website: tailieumontoan.com

7(ab  bc  ca)  5(a 2  b2  c 2 )


 a 2  b2  c 2  3 
6
7(ab  bc  ca)  5(a 2  b2  c 2 )
Do a  b  c  ab  bc  ca, a, b, c nên
2 2 2
 2(ab  bc  ca)
6
Vậy a 2  b2  c2  3  2(ab  bc  ca)

Dấu bằng xảy ra khi và chỉ khi a  b  c  1 .

Bài 5: (4,0 điểm)


Cho một bảng ô vuông kích thước 10 10 , trên đó đã điền các số nguyên dương từ 1
đến 100 vào các ô vuông con theo trình tự như hình a. Ở mỗi bước biến đổi, người ta
chọn tùy ý ba ô vuông con liên tiếp theo hàng hoặc theo cột hoặc theo một đường chéo
của hình vuông kích thước 3  3 (xem hình b) rồi thực hiện: Hoặc là giảm số ở ô nằm
giữa đi 2 đơn vị đồng thời tăng số ở hai ô liền kề lên 1 đơn vị, hoặc l| tăng số ở ô nằm
giữa lên 2 đơn vị đồng thời giảm số ở hai ô liền kề đi 1 đơn vị. Giả sử rằng sau hữu hạn
bước, tập hợp tất cả các số ghi trên bảng ô vuông vẫn là tập {1; 2; 3; <; 100}. Chứng
minh rằng khi đó c{c số ghi trên bảng theo đúng vị trí như trước khi biến đổi.

1 2 3 < 9 10

11 12 13 < 19 20

21 22 23 < 29 30

< < < < < <

91 92 93 < 99 100 Hình b- Ba ô vuông con liên


tiếp
Hình a – Bảng ô vuông ban đầu
Lời giải
Ta kí hiệu ani , j  là số ghi ở ô vuông con thuộc hàng i, cột
j ở ngay sau bước biến đổi thứ n, ở đó thứ tự hàng i tính
từ trên xuống dưới, thứ tự cột j tính từ trái sang phải.
+ Ban đầu (coi l| “ngay sau lần biến đối thứ 0”) trên
bảng có các số a0i , j được điền vào ô theo quy luật
a0i , j  10  i  1  j với mọi i, j  *
, 1  i, j  10 .

156
Page: Tài Liệu Môn Toán
Website: tailieumontoan.com

+ Xét đại lượng Tn  


1i , j 10
a0i , j  .ani , j  với n  .

Ban đầu khi chưa biến đổi, có T0  


1i , j 10
a0i , j  .a0i , j 

Xét từ lần biến


đổi thứ n sang
lần biến đổi thứ
n + 1, bằng cách
thử từng khả
năng về chọn bộ
ba ô vuông liền
kề (ô ở giữa điền
ani , j  ):

Trường hợp chọn 3 ô vuông liên tiếp như hình vẽ trên ta có


Tn 1  a01,1 .an1,11  ...  a0i 1, j .ani11, j  a0i, j .ani,j1  a0i 1, j .ani11, j  ...a010,10 .an101,10

    
 a01,1 .an1,1  ...  a0i 1, j . ani 1, j 1  a0i, j . ani, j  2  a0i 1, j . ani 1, j 1  ... 

 a010,10 .an10,10  Tn  2a0i, j  a0i 1, j  a0i 1, j 

C{c trường hợp còn lại,với cách thức biến đổi tương tự,
ta thấy giá trị của Tn chỉ “tăng” hoặc “giảm” đi một
lượng dạng 2a0i , j  a0p,m  a0q ,r  với p, m, q, r là số nguyên
dương thỏa mãn p  q  2i và m  r  2 j .

Mặt khác:
2ai , j  a p,m  a q,r   2 10  i  1  j   10  p  1  m  10  q  1  r   0
0 0 0

, vậy Tn  Tn1 với mọi n, nghĩa l| Tn là một bất biến của


quá trình biến đổi.
+ Giả sử sau N bước, tập hợp các số ghi trên bảng đúng
bằng {1; 2; 3; <; 100} nghĩa l| bộ
 aN
1,1 
, aN1,2 ,..., aN1,10 , aN2,1 ,..., aN10,10 là một hoán vị của bộ
(1; 2; 3; <; 100).
Ta có TN  TN 1  ...  T0 nên

157
Page: Tài Liệu Môn Toán
Website: tailieumontoan.com


1i , j 10
a0i , j  .aNi , j   
1i , j 10
a0i , j  .a0i , j  .

Mặt khác theo bất đẳng thức về dãy sắp xếp ta luôn có
 a0i, j  .aNi, j    a0i, j  .a0i, j  , dấu đẳng thức chỉ xảy ra
1i , j 10 1i , j 10


khi hai dãy aN1,1 , aN1,2 ,..., aN1,10 , aN2,1 ,..., aN10,10 và 
 a0
1,1 
, a01,2 ,..., a01,10 , a02,1 ,..., a010,10 xếp theo cùng trật tự
tăng giảm.
Hai dãy này là hoán vị của nhau nên điều đó chỉ xảy ra
khi và chỉ khi
 a  , a  ,..., a  , a  ,..., a  
N
1,1
N
1,2
N
1,10
N
2,1
N
10,10

  a  , a  ,..., a  , a  ,..., a   .
0
1,1
0
1,2
0
1,10
0
2,1
0
10,10

Vậy bảng số lúc n|y được sắp xếp đúng như trật tự lúc
đầu.

Đề 25

7 x  y  3xy  x  y   12 x  6 x  1 (1)
 3 3 2

Câu 1. Giải hệ phương trình 


2 x  3  9  y  1 (2)

2 2

Điều kiện x{c định: 3  y  3


Phương trình (1)   x  y    2 x  1  y  1  x (3)
3 3

Thế (3) v|o (2) ta được: 2 x2  3  8  2 x  x 2  0


 2 x 2  3  8  2 x  x 2  4  x 2  3  2 x  8  2 x 8  2 x  x 2

 2  x  1  x
2
 2 

8  2 x  x 2  3  0   x  1  2 

x 
0
8  2 x  x2  3 
 x 1  0
  x
2 0
 8  2x  x2  3
Ta có hai trường hợp:
*TH1:Nếu x = 1 thì y = 0.
Thử lại vào hệ phương trình ban đầu thấy thỏa mãn.
x
*TH2: Nếu 2   0 thì ta có phương trình
8  2 x  x2  3

158
Page: Tài Liệu Môn Toán
Website: tailieumontoan.com

 x  6  0

2 8  2 x  x2   x  6   2 (vô nghiệm)

5 x  4 x  4  0
Vậy hệ phương trình đã cho có nghiệm duy nhất là  x; y   1;0 
Câu 2.

- Gọi E, F, M, N lần lượt là tiếp điểm (O1), (O2) với đường tròn (O) v| AB như hình vẽ.
Gọi K l| giao điểm EF với (O).
Ta có c{c điểm E, O1, O thẳng h|ng; c{c điểm M, O2, O thẳng hàng.
- Hơn nữa EKO  OEF  O1FE  O1F || OK  OK  AB
Vậy K l| điểm chính giữa cung AB.
Như vậy EF đi qua điểm chính giữa K của cung AB.
- Chứng minh tương tự ta cũng có MN cũng đi qua K.
- Từ đó MEF  MNB nên tứ giác EFNM là tứ giác nội tiếp, do đó
PK/ (O1 )  KF .KE  KN .KM  PK/ (O2 )
Vậy điểm K nằm trên trục đẳng phương của (O1), (O2)
Suy ra ba điểm C, T, K thẳng hàng.
Từ đó điểm T nằm trên phân giác của ACB (1)
- Ta có các cặp tam gi{c đồng dạng KAF và KEA ; KBN và KMB
Từ đó KA2  KF.KE  KT 2  KA  KT
Ta lại có KA = KB, suy ra KA = KB = KT.
Vì vậy các tam giác KAT và KBT cùng cân tại K.
Do đó CAT  ATK  ACT  TAK  BAK  TAB
Suy ra AT là phân giác của CAB (2)
Từ (1) v| (2) suy ra T l| t}m đường tròn nội tiếp tam gi{c ABC (đpcm)
Câu 3.

159
Page: Tài Liệu Môn Toán
Website: tailieumontoan.com

Đặt n  mq  r  0  r  m khi đó ta viết 2016n  1  2016mq r  1  2016mq.2016r  1


Ta xét c{c trường hợp sau:
*TH 1: Nếu q là số lẻ thì 2016n  1   2016m   1 .2016r  1  2016r
q

 
Kết hợp với (2016m +1) | (2016n +1) thu được
 2016m  1 |  2016n  1  r  0  m | n
*TH2: Nếu q là chẵn thì 2016n  1   2016m   1 .2016r  2016r  1
q

 
Kết hợp với (2016m +1) | (2016n +1) và  2016m  1 |  2016m   1 ta thu được
2

 
 2016m  1 |  2016r  1 (vô lí vì 0  r  m)
Vậy ta có điều phải chứng minh.
Câu 4.
1 1 1
Đặt x  , y  , z ta có x, y, z là các số dương n|y v| xy  yz  zx  1
a b c
x2 y 2 z 2
Ta cần chứng minh 3  3      x  y  z 
2

y z x
x2 y 2 z 2  x  y  z   x  y  z 
2 2 2

Trước hết ta chứng minh    (1)


y z x xy  yz  zx
Thật vậy, ta có:
x2
(1)   xy  yz  zx     x  y  z   x2  y 2  z 2 
xyz y
x3 z
 x3  y 3  z 3  x 2 z  z 2 y  y 2 x    x3  y 3  z 3   x 2 y
xyz y xyz

3 3 3
xz y x z y
    xz 2  zy 2  yx 2 (2)
y z x
Theo bất đẳng thưc AM – GM ta có
x3 z y 3 x y3 x z 3 y x3 z z 3 y
  2 x 2 y;   2 y 2 z;   2z2 x
y z z x y x
Cộng theo vế ba bất đẳng thức trên suy ra bất đẳng thức (2) được chứng minh.
Vậy bất đẳng thức (1) được chứng minh.
x2 y 2 z 2
Từ (1) suy ra 3  3     3  3   x  y  z   x 2  y 2  z 2 
y z x
Vì vậy ta cần chứng minh 3  3   x  y  z   x 2  y 2  z 2    x  y  z 
2

  x  y  z   x2  y 2  z 2   x2  y 2  z 2 3  1

  x 2  y 2  z 2   x  y  z  1  3  1 (3)

160
Page: Tài Liệu Môn Toán
Website: tailieumontoan.com

Do x2  y 2  z 2  xy  yz  zx  1và x  y  z  3  xy  yz  zx   3 nên ta có bất đẳng


thức (3) được chứng minh. Từ đó ta có đpcm.
Câu 5.
Tổng số phần tử trong 64 tập con lớn hơn 64.1008 = 32.2016. Vì vậy tồn tại một phần tử
a của tập X thuộc ít nhất 33 tập con, giả sử là X1, X2,....X33.
Xét 31 tập con còn lại, lí luận tương tự suy ra tồn tại một phần tử b của tập X thuộc ít
nhất 16 tập con, giả sử là X34, X35,....X49.
Xét 15 tập con còn lại, lí luận tương tự suy ra tồn tại một phần tử c của tập X thuộc ít
nhất 8 tập con, giả sử là X50, X51,....X57.
Xét 7 tập con còn lại, lí luận tương tự suy ra tồn tại một phần tử d của tập X thuộc ít
nhất 4 tập con, giả sử là X58, X59, X60, X61.
Xét 3 tập con còn lại, lí luận tương tự suy ra tồn tại một phần tử e của tập X thuộc ít
nhất 2 tập con, giả sử là X62, X63.
Với tập X64 còn lại ta lấy một phần tử f
Như vậy tập con A chứa các phần tử a, b, c, d, e, f thỏa mãn bài toán.
Suy ra điều phải chứng minh.
Đề 26
Câu 1. a) điều kiện: x  1  0  x  1
Phương trình đã cho tương đương với:

4 x 2  12 x x  1  27  x  1  36 1  x   2 x  3 1  x   6 
2 2
1 x
2 x  3 1  x  6 1  x 3 1  x  2 x 1
 
 2 x  3 1  x  6 1  x 9 1  x  2 x  2 

9 1  x   4 x
 
4 x  9 x  9  0
2 2

Ta có (1)     x3
x  0
 x  0

8 1  x   4 x
 4 x 2  81x  81  0 81  9 97
2

Ta có (2)    x
x  0
 x  0
 8
81  9 97
Vậy x = 3; x  là nghiệm của phương trình đã cho.
8
x  2
b) Điều kiện: x  5  3  0  
x  8
9 9
Trường hợp 1: Xét x < 2 ta có (1)   2 x   2 x
5 x 3 2 x
  2  x   9  3  x  2  3  1  x  5 vậy 1  x  5 là nghiệm.
2

9 9
Trường hợp 2: Xét 2 < x < 5 ta có (1)   x2  x2
5 x 3 2 x
161
Page: Tài Liệu Môn Toán
Website: tailieumontoan.com

   x  2   9 vậy (Bất phương trình vô nghiệm)


2

9 9
Trường hợp 3: Xét 5 < x  8 ta có (1)   x2   x  2  0
x 8 x 8
9   x  8  x  2   x 2  10 x  7
 0 0
x 8 x 8
x  5  3 2
  x  8   x 2  10 x  7   0  
8  x  5  3 2
Kết hợp với miền x đang xét ta có 8  x  5  3 2 là nghiệm của bất phương trình.

Vậy tập nghiệm của bất phương trình l|: S   1; 2   8;5  3 2 

Câu 2. Giả sử có số nguyên dương n sao cho n + 26 = x3 và n – 11 = y3 với x,y là hai số
nguyên dương (x>y).
Khi đó ta được x3  y3  37   x  y   x 2  xy  y 2   37

x  y  1
 (1)
Ta thấy o  x  y  x2  xy  y 2 nên ta có  2
 x  xy  y  37

2
(2)
Thay x = y +1 từ (1) v|o (2) ta được y  y  12  0 từ đó có y = 3 v| n = 38.
2

Vậy n = 38 là giá trị cần tìm.


Câu 3. Cách 1: Đặt AB = c, AC = b, BC = a, KAC  
Khi đó: KAB  2, BAC  3
Áp dụng định lí sin cho tam giác ABK và ACK, ta
được:
BK AK CK AK
 ; 
sin 2 sin B sin  sin C
Do BK = 2CK, nên từ c{c đẳng thức trên ta có:
sin B
cos   (*)
sin C
Lại có:
 b2  c 2 a 2  a 2 b2  c 2  a 2
FA  FC  
2 2
    bc.cos A  bc cos 3 (1)
 2 4 4 2
LC 2  LA2  b 2  2b.LA.cos   LA2  b 2  2bc cos 2.cos 
 LA2  LC 2  2bc cos .cos 2  b 2  bc  cos   cos 3   b 2
  bc cos a  b 2   bc cos 3(**)
Thay (*) v|o (**), ta được: LA2  LC 2  bc cos3 (2)
Từ (1) avf (2) suy ra: FA  FC  LA  LC
2 2 2 2

Theo định lí carnot, suy ra CA vuoonng góc với FL.

162
Page: Tài Liệu Môn Toán
Website: tailieumontoan.com

Cách 2: Trường hợp 1: L nằm trên đoạn AK.


FK BF
Ta có:  .
KC BC
Gọi M l| trung điểm của BK. Suy ra:
MF .MC  MB.MK  ML2  MLF  MCL
Mà MLK  MKL
LC KC
 FLK  CLK   2
LF KF
Gọi N l| điểm đối xứng với L qua F.
Suy ra LC = LN, BN = LC (BNCL là hình bình
hành). Suy ra NB = NL.
Vậy ALC  ALN  c.g.c 
BAK
Từ đ}y LAN  LAC   BAN  KAN
2
Vì MNKL là hình hành, KL  BL nên MN l| đường
trung trực của BL. N l| giao điểm của đường trung trực
của đoạn thẳng BL với đường phân giác góc A của
đường tròn ngoại tiếp tam giác ABL.
Vậy N l| điểm chính giữa của cung BL (không chứa
điểm A) của (ABL).
Vậy ANL  ABL  900  BAL  900  NAC.
Hay NL  AC .
Trường hợp 2: L nằm ngo|i đoạn AK.
Lập luận tương tự ta cũng có NL  AC
Cách 3: Gọi D l| điểm đối xứng với B qua AK v| E l| điểm trên tia AK sao cho AE = AB
= AD.
Ta thấy tam giác EAD là tam giác cân và C nằm trên phân
giác của EAD .
Ta chứng minh rằng C l| trung điểm của DE.
Thật vậy, giả sử C không nằm trên DE. Gọi C’l| giao điểm
của DE v| AC, AC’ cắt KE tạo K’.
Suy ra K’ l| trọng tâm cảu tam giác EBD.
 BK '  2K ' C  KK '/ /CC '. Vô lý. Vậy C  C '.
Vì C l| trung điểm của DE, suy ra DE  AC .Hơn nữa F là
trung điểm của BC, L l| trung điểm của BD, suy ra FL / / ED.
Vậy FL  AC .
Câu 4: Ký hiệu X là số phần tử của tập hữu hạn X.

163
Page: Tài Liệu Môn Toán
Website: tailieumontoan.com

Gọi B1, B2,...,Bn là tập con của A thỏa mãn:


Bi  3, Bi  B j  2  i, j  1, 2,3,...n  .
Giả sử tồn tại phần tử a  A mà a thuộc vào 4 tập trong số tập B1, B2,...,Bn (chẳng hạn a
 B1, B2, B3, B4), khi đó: Bi  B j  1 i, j  1, 2,3, 4  . Mà Bi  B j nếu i  j, tức là Bi  B j  3 .
Do đó Bi  B j  1 i, j  1, 2,3, 4  .
Từ đ}y A  1  4.2  9, điều này mâu thuẫn.
Như vậy, mỗi phần tử A chỉ thuộc về nhiều nhất là ba trong số các tập hợp B1, B2,...,Bn.
Khi đó 3n  8.3  n  8
Giả sử A = a1; a2 ;..., a8  xét các tập con của A là:
B1 = a1; a2 ; a3  , B2 = a1; a4 ; a5  ; B3 = a1; a6 ; a7  , B4 = a8 ; a3 ; a4 
B5 = a8 ; a2 ; a6  , B6 = a8 ; a5 ; a7  ; B7 = a3 ; a5 ; a6  , B8 = a2 ; a4 ; a7 
Tám tập hợp trên là các tập con gồm ba phần tử A thỏa mãn Bi  B j  2 . Vì vây số n
cần tìm là n = 8
 x  1 y  1  y  1 z  1  z  1 x  1
2 2 2

Câu 5.    x y z 3
3 3 z 2 x2  1 3 3 x2 y 2  1 33 y2 z2 1
Gọi vế trái của bất đẳng thức S. Do ab  a  b  3 3 a 2b2 , a  0, b  0.
 x  1 y  1   y  1 z  1   z  1 x  1   y  1   z  1   x  1
2 2 2 2 2 2

Nên S 
 z  1 x  1  x  1 y  1  y  1 z  1 z 1 x 1 y 1
 y  1   z  1   x  1 
2

  x  y  z  3 (điều phải chứng minh)


 z  1   x  1   y  1
Dấu bằng xảy ra khi và chỉ khi a = b = c =1

Đề 27
Câu 1.
b 1  9
a) Tập x{c định của hàm số là R. a = 1 > 0 ,  , 
2a 2 4a 4
 1  1 
Hàm số nghịch biến trên  ;  đồng biến trên  ;  
 2 2 

164
Page: Tài Liệu Môn Toán
Website: tailieumontoan.com

b) Phương trình ho|nh độ giao điểm x2  x  2  x  m  x2  2x  m  2  0


Đường thẳng cắt đồ thị tại hai điểm phân biệt   '  0  m  3
m
 : x  y  m  0, d  O,   
2
A  x1 ; x1  m  , B  x2 ; x2  m 

AB   x2  x1    x2  m  x1  m   AB  2  x2  x1 
2 2 2

 2  x2  x1   8 x1 x2  2.22  8  m  2   8m  24
2

m
AB  d  O,     m 2  16m  48  0
2
 m  8  4 7 (thỏa mãn điều kiện)
Câu 2. a)
 x 2  10 x  y 2  10 y   81

HPT  
 x  10 x  y  10 y   18

2 2

Đặt u  x2  10 x, v  y 2  10 y. Ta có u.v  81, u  v  18


Suy ra u, v là 2 nghiệm cảu phương trình x2  18x  81  0
Suy ra u = v = -9.
Hệ đã cho tương đương với hệ phương trình sau:

 x  10 x  9
2

 x  10 x  9  0
2
x  1 x  9
 2  2 
 y  10 y  9
  y  10 y  9  0
  y  1 y  9
165
Page: Tài Liệu Môn Toán
Website: tailieumontoan.com

Hệ đã cho có 4 nghiệm 1;1 , 1;9 ,  9;1 , 9;9


b) Đặt t  x2  5 x  7,  t  0
 x 1 x  4  t 2  3
Phương trình trở thành 2t  3  t 2  3  8
2t  3  t 2  3  8  3t 2  2t  1  0
1
t  1 t 
3
1
t không thỏa mãn điều kiện
3
Với t = 1, ta có x2  5x  7  1  x2  5x  6  0  x  2 hoặc x  3
Vậy phương trình có tập nghiệm là 2;3
c) 4  x  4  x  2 16  x 2  m (điều kiện 4  x  4)
Điều kiện cần. Giả sử hệ có nghiệm duy nhất là x0
Ta có 4  x0  4  x0  2 16  x02  m

4    x0   4    x0   2 16    x0   m
2

  x0 là một nghiệm của phương trình


Vì phương trinh duy nhất nên x0   x0  x0  0  m  12
Điều kiện đủ: Xét m = 12 phương trình đã cho trở thành
 
2
2 16  x 2  2 16  8 4 x  4 x  8  2 16  x 2  12

4  x  4  x  2 16  x 2  16
 4  x  4  x  2 16  x 2  4  8  12
Đẳng thức xảy ra  x  0 . Phương trình có nghiệm duy nhất x = 0, vậy m = 12.
Câu 3. a)
 a  b  a  c    b  c  b  a    c  a  c  b   4 a  b  c
BĐT   
a b c
bc ca ab
 abc abc abc  4a  b  c
a b c
bc ca ab
    a  b  c.
a b c
bc ca ca ab ab bc
Áp dụng BĐT CoSi, ta có:   2c,   2a,   2b,
a b b c c a

166
Page: Tài Liệu Môn Toán
Website: tailieumontoan.com

 bc ca ab 
2     2a  b  c
 a b c 
bc ca ab
    abc
a b c
1 1 1 4(a  b  c
   
ab  ac bc  ba ca  cb  a  b  b  c  c  a 
Đẳng thức xảy ra khi a = b = c
b)
S 2   x  y  z   x2  y 2  z 2  2  x y  y z  z x 
2

S 2  x2  y 2  z 2  x  y  z   y  z  x   z  x  y 
Áp dụng bất đẳng thức chứa dấu giá trị tuyệt đối ta có
y  z  y  z  x  x  x  y  z   z2
Chứng minh tương tự y  z  x   y 2 , z  x  y   z 2
Vì vậy S 2  2  x 2  y 2  z 2 
Thay x2  y 2  z 2  8  S 2  16  S  4
Dấu bằng có thể xảy ra, khi  x, y, z    2; 2;0  hoặc các hoán vị, ta có S = 4
Vậy min S = 4
Câu 4.a)
cos A b 2  c 2  a 2 b 2  c 2  a 2 cos B c 2  a 2  b 2 c 2  a 2  b 2
cot A    , cot B   
sin A 2bc sin A 4S sin B 2ca sin B 4S
c2
 cot A  cot B 
2S
c2 a 2  b2
 
2S 2S
 c  a  b2  tam giác ABC vuông tại C
2 2

b)
Ta chứng minh
3
MD  ME  MF  MO
2
Qua M kẻ đường thẳng song song với BC lần lượt cắt AB, AC
tại A1, A2; kẻ đường thẳng song song với AC lần lượt cắt BC,
AB tại B1, B2; kẻ đường thẳng song song với AB lần lượt cắt BC,
AC tại C1, C2.
Các tam giác MB1C1, MA2C2, MA1B2 đều,

167
Page: Tài Liệu Môn Toán
Website: tailieumontoan.com

MD 
1
2

MB1  MC1 
1
  1

ME  MA2  MC2 , MF  MA1  MB2
2 2

1
    
MD  ME  MF   MB2  MC2  MA1  MC1  MA2  MB1 
2  

1
2
 
MA  MB  MC  MO (1)
3
2
Gọi G là trọng tâm của tam giác DEF. Ta có MD  ME  MF  3MG
3
Từ (1), (2) ta có MO  3MG  MO  2MG
2
 M, O, G thẳng hàng. Vậy OM đi qua trọng tâm của tam giác DEF
Câu 5. a)
y  0
A:   A  2;0 
x  y  2  0
Gọi E l| điểm đối xứng của B qua AD: y = 0, ta có E ∈ AC, E(1;-3)
Phương trình đường thẳng AC
x2 y 0
  3x  y  6  0
1  2 3  0
 c  1 3c  3 
C  c;3c  6  , M  ; 
 2 2 
c  1 3c  3
  2  0  c  0  C  0; 6 
2 2
b) Gọi M l| trung điểm của BC, ta có MD = ME

Gọi M  m; 2m  1 , ta có MD  ME nên


 5m2  8m  5  5m2  10m  5  m  0  M  0;1 ,

168
Page: Tài Liệu Môn Toán
Website: tailieumontoan.com

Ta có B  b; 2b  1 , b  0.MB  b  0    2b  1  1  5b 2


2 2

MB  MD  5  5b2  5, b  0  b  1  B 1; 1

Đề 28
Câu 1. 1. Ta có: a  1  0,  '  m2  3m  1
3  5 3  5 
f ( x)  0 x    '  0  m2  3m  1  0  m   ; 
 2 2 
 '  0

2. Yêu cầu bài toán tương đương với  x1  1 x2  1  0

 x1  1   x2  1  0
 3 5 3 5
 m   m 
 '  0 2 2
  3 5
  x1 x2   x1  x2   1  0  3  m  0  m3
  2m  4  0 2
 x1  x2  2  0 

Câu 2.1. Điều kiện: x  3
Phương trình đã cho tương ứng với
 x 1  3  0  x  10
 x 1  3  
x 3 2  0  
 x  3  2  0

x  7
1
2. Điểu kiện x, y ≥
2
Phương trình thứ nhất của hệ tương đương với  x  y   x 2  2xy  2   0 (*)
1
Dox, y ≥ nên x2 + 2xy + 2 > 0
2
Vì vậy (*)  x  y  0  x  y, thay v|o phương trình thứ hai của hệ:
 1
x  2
2 2 x  1  3x  1  
4  2 x  1   3x  12

5
Giải ra ta được x =1, x =
9
5 5
Vậy nghiệm của phương trình l| (1;1) v|  ; 
9 9
2
Câu 3. 1. Điều kiện x 
3
  
Bất phương trình đã cho tương đương với 3x  2 1  x  3  2  x 3  3 x  4 
169
Page: Tài Liệu Môn Toán
Website: tailieumontoan.com

3  x  1 x 1
    x  1  x 2  3x  4 
3x  2  1 x3 2
 3 1 
  x  1  x 2  3x  4     0 (1)
 3x  2  1 x3 2
3 1 3 3x  2 x  3 1 2
Ta có: x 2  3x  4   = x2  x    0, x 
3x  2  1 x3 2 3x  2  1 x3 2 3
Do đó (1)  x 1  0  x  1
2
Vậy  x  1
3
2. Ta có: A  cos2 x  sin 2 x  2sin 2 x 3 3 1  2sin 2 x cos2 x   2 1  3sin 2 x cos2 x 

A  cos 2 x  sin 2 x  2sin 2 x 3 3  6sin 2 x cos 2 x  2  6sin 2 x cos 2 x


A  cos 2 x  sin 2 x  2sin 2 x  1
Vậy A = 1, suy ra A không phụ thuộc vào biến x
4ma2 4mb2
  AB 2
GA  GB  AB
2 2 2
9 9
Câu 4.1 Ta có GA.GB  
2 2
Tương tự với 2 đẳng thức còn lại, sau đó công 3 đẳng thức lại ta được:
4  ma2  mb2  mc2 
  AB 2  BC 2  CA2 
GA.GB  GB.GC  GC.GA  9
2
Sử dụng công thức trung tuyết suy ra điều phải chứng minh.
2. Ta có: d(A; BC) = 2 2
∆ABC vuông tại A nên
1 1 1 1
  2 
AB 2
AC 2
d  A; BC  8
Kết hợp với điều kiện AB = 2AC ta được AC2 = 10
Mà C  BC nên C(a; – a – 1 )
  a  1   a  3  10
2 2

 2a2  4a  10  10  a  0 hoặc a = - 2
+) Với a = 0 suy ra C(0; - 1)
Phương trình AB đi qua A, vecto ph{p tuyến
AC   1; 3 là x + 3y – 7 = 0
Từ đó tìm được tọa độ điểm B (-5;4)
+) Với a = -2 suy ra C(-2;1)
Phương trình AB đi qua A, vecto ph{p tuyến AC   1; 3 là x + 3y – 5 = 0

170
Page: Tài Liệu Môn Toán
Website: tailieumontoan.com

Từ đó tìm được tọa độ điểm B (3;-4)


3. Đường tròn (C1) có tâm I1 (1;3) bán kính R1 = 3, (C2) có tâm I2 (2;-
2) bán kính R2 = 5
Ta có A là một điểm chung của hai đường tròn.
Gọi n   a; b   0 là vecto pháp tuyến của đường thẳng
   : ax  by  a  0
 3b
d  I1 ;   
 a 2  b2

d I ;   a  2b
  2 
Ta có:  a 2  b2
 2 36a 2
 MA  4  R1  d  I1 ;     a 2  b 2
2 2


 MB 2  4 R 2  d 2 I ;   16a  16ab  4b
2 2

  2  2  a 2  b2
Do MA = 2MB nên MA2 = 4MB2  36a 2  4 16a 2  16ab  4b2 
7a
 a  2n hoặc b  
2
+) với a  2n , ta chọn b  1,a  2  phương trình ∆: 2x – y – 2 = 0
7a
+) với b   , ta chọn a  2, b  7  phương trình ∆: 2x –7y – 2 = 0
2
1 
Câu 5: Ta có   1  c   a3  b2  c    a  b  c   9
2

a 
1 
a  1 c 
a
 
a   1  a  ac
a b c
3 2
9 9
Do đó ta chứng minh được
a b c 3   a  b  c    ab  bc  ac 
 3 2  3 
a b c b c a c a b
3 2 2
9
Mà 3  ab  bc  ac    a  b  c   9   ab  bc  ac   3
2

a b c 333
Vậy  3 2  3   1 (điều phải chứng minh)
a b c b c a c a b
3 2 2
9
Đề 29
Câu 1.

Ta có: A   20112  1  20102010  .20102011   20104021  1  1


2010

 

Hay A  M  N  1 với
171
Page: Tài Liệu Môn Toán
Website: tailieumontoan.com

M   20112  1  20102010  .20102011  2011.E 2011


2010

 

N   20104021  1  2011.F 2011

Vậy thêm 2011k  1 (k nguyên) vào thì A chia hết cho 2011.
Câu 2.
 x 1 
Theo đề b|i, ta có: f  x   f    x  1 (1)
 x 
x 1 1
Đặt t  x
x 1 t
 1  t 2
Khi đó: (1)  f    f t   , t  1 (2)
1 t  t 1
1 x 1 t 1
Đặt  x
1 t x t
 t 1   1  2t  1
Khi đó: 1  f   f   , t  0, t  1 (3)
 t  1 t  t

Cộng vế theo vế phương trình (2) v| (3), ta được:


 1  2t  1 t  2
2f     t  1   , t  0, t  1
1 t  t t 1

 1  1 1 1
 f   2t    , t  0, t  1
1 t  2  1 t t 

1 1 1 
 f  x   x    , t  0, t  1
2 x 1 x 

1 1 1 
Thử lại, ta thấy f  x    x    , thỏa mãn điều kiện đề b|i.
2 x 1 x
 

Câu 3.
Điều kiện: x  1
t 4  4 2 t 8  8t 4  16
Đặt t  4 4 x  4  t  0   x  ,x 
4 16
Phương trình đã cho trở th|nh: 3t 8  8t 4  128t  1152  0
  t  2  3t 7  6t 6  12t 5  24t 4  56t 3  112t 2  224t  576   0

 t  2 (vì t  0 ).

Với t  2 , ta có 4
4x  4  2  x  3 .

172
Page: Tài Liệu Môn Toán
Website: tailieumontoan.com

Câu 4.

a3 a  ab  b2  ab  a  b  ab  a  b 
Ta có: 2 a 2 a 2 a
a  ab  b 2
a  ab  b 2
a  ab  b 2
3ab
a3 ab
 a (1)
a  ab  b
2 2
3
Tương tự, ta có:
b3 bc
b (2)
b  bc  c
2 2
3
c3 ca
c (3)
c  ca  a
2 2
3
Cộng vế theo vế của (1), (2) v| (3) ta được:
a3 b3 c3 abc
 2  2 
a  ab  b b  bc  c
2 2 2
c  ca  a 2
3
 S  3, S  3 khi a  b  c  1 .

Vậy gi{ trị lớn nhất của biểu thức S là 3.


Câu 5.
BC 2
[p dụng định lí sin cho tam gi{c ABC, ta có:  4 R 2  2 BC 2  6 R 2
sin 2 A
Khi đó:

      MO  OC 
2 2 2
MA2  MB 2  MC 2  2BC 2  OM  OA  MO  OB  6R 2

 
2
 3MO  2MO OA  OB  OC  3R 2

 OM  R (do OA  OB  OC  0 )  M   C  .

Đề 30
Câu 1. Cách 1: Ta có:

173
Page: Tài Liệu Môn Toán
Website: tailieumontoan.com

sin B  2sin C
sin A   2sin A cos B  sin A cos C  sin B  2sin C
2 cos B  cos C
1
 sin  A  B   sin  A  B   sin  A  C   sin  A  C    sin B  2sin C
2
1
 sin C  sin  A  B   sin B  sin  A  C    0
2
1
 sin  A  B   sin  A  B   sin  A  C   sin  A  C    0
2
cos A  0 (1)
 2 cos A sin B  cos A sin C  0  
 2 sin B  sin C  0 (2)
Phương trình (2) vô nghiệm
Phương trình (1) cho tam gi{c ABC vuông tại A.
Cách 2: Ta có:
sin B  2sin C b  2c
sin A  a
2 cos B  cos C a  c  b2 a 2  c2  b2
2 2
2. 
2ac 2ac
 2a 2 b  2bc 2  2b3  a 2c  b 2c  c3  2b 2c  4bc 2  2a 2 b  2b 3  a 2c  c3  2bc 2  b 2c  0
 2b  a 2  c 2  b 2   c  a 2  c 2  b 2   0   a 2  c 2  b 2   2b  c   0
a 2  c2  b 2  0
  a 2  b2  c2
 2b  c  0 (VN)
Theo định lý Pytago suy ra tam giác ABC vuông tại A.

 x  xy  2 y  3 y  1 (1)
2 2

Câu 2. Ta có: 
 x x  y  x  y  2 (2)

Điều kiện: x  y  0
Xem phuoeng trình (1) l| phương trình bậc hai theo x nên ta có được hai nghiệm là:
x  y  1; x  1  2y
 Với x  y 1  x  y  1 (loại vì x  y  0 )
1
 Với x  1  2y thay v|o (2) ta được: 3y  3  1  2y  1  3y  0 (y  )
3
Đặt t  1  3y; t  0 thay vào giải ta được: t = 2  1  3y  2  y  1  x  3
Vậy hệ thức có nghiệm là x = 3 và y = -1
a  3c a  3b 2a
Câu 3.    5 (1)
ab ac bc
 a c a b  a b c 
Cách 1: Ta có: VT(1)      2   
a b a c   bc a c a b
ac a b
+)  2
a b a c
174
Page: Tài Liệu Môn Toán
Website: tailieumontoan.com

a b c  1 1 1  9 3
+)    a  b  c      3  a  b  c 3
bc ac a b  bc a c a b 2 a  b  c  2
3
 VT1  2  2.  5 (điều phải chứng minh). Dấu bằng xảy ra khi và chỉ kkhi a = b = c.
2
Cách 2: Đặt x = a + b , y = b + c, z = c + a.
y 2z 2x y x z
Ta được VT1  3        5 (điều phải chứng minh).
x x z z y y
Câu 4. Cách 1: Vì m.n = k2 và k không chia hết cho 3 (m, n, k là số nguyên) nên m, n
không chia hết cho 3. Suy ra m = 3m’ +r1, n = 3n’ + r2 (r1, r2 ∈{1;2}
Do m.n = k2 nên m.n ≡ r1 r2 (mod 3), suy ra r1r2 ≡ 1 (mod 3) suy ra r1≡ r2 ≡1 (mod 3)
Suy ra m ≡n (mod 3)  m  n 3 (điều phải chứng minh).
Cách 2: Vì k không chia hết cho 3 nên k2≡ 1 (mod 3) suy ra m.n≡ 1 (mod 3) (*)
Vì m.n không chia hết cho 3 nên m, n không chia hết cho 3.
Giả sử m và n không cùng số dư khi chia cho 3 thì m.n ≡ 2 (mod 3) mẫu thuẩn (*).
Suy m,n chia hết cho 3 có cùng số dư. Vậy m – n chia hết cho 3.
Câu 5. Gọi G  AB  CD suy ra GC = GD (vì
GA.GB = GC2 = GD2 l| phương tích của điểm G với
hai đường tròn).
Theo định lí Talet ta có:
GD GB GC
  m| GC = GD nên AE = AF (điều
AE AB AF
phải chứng minh)

Đề 31
Câu 1. Ta có:

175
Page: Tài Liệu Môn Toán
Website: tailieumontoan.com

 x3  y 3  3 y 2  9  x 3  y 3  3 y 2  9
 2   2
 x  y  x  4 y  x  x   y  4 y
2 2

 x3  3  x 2  x   y 3  3 y 2  9  3   y 2  4 y 

 x 2  x   y 2  4 y
 x3  3x 2  3x  y 3  6 y 2  12 y  9  x 3  3x 2  3x  1  y 3  6 y 2  12 y  8
 2  2
 x  x   y  4 y  x  x   y  4 y
2 2

 x  13   y  2 2  x  1  y  2  x  y  3
  2  
 x  x   y  4 y  y  3   y  3   y  4 y
2 2 2
 x  x   y  4 y
2 2

 9  33  9  33
 y   y 
 x  y  3  4  4
 2  
2 y  9 y  6  0  3  33  3  33
 x  4 

x
4
2 2 2
x y z
Câu 2: Ta có   1
x 8
3
y 8
3
z3  8
Theo bất đẳng thức Cauchy cho các số thực dương ta có:
 x  2   x2  2 x  4 x2  x  6
x  8   x  2  x  2x  4 
3 2

2 2
2 2
x 2x
  2
x3  8 x  x  6
y2 2 y2 z2 2z2
Tương tự, ta cũng có  ; 
y3  8 y2  y  6 z3  8 z 2  z  6
Từ đó suy ra:
x2 y2 z2 2x2 2 y2 2z 2
   2  2  2 (1)
x3  8 y3  8 z3  8 x  x  6 y  y  6 z  z  6
Mặt khác theo công thức Cauchy – Shwarz:
2 x  y  z
2
2 x2 2 y2 2z2
   2 (2)
x  x  6 y  y  6 z  z  6 x  y  z 2   x  y  z   18
2 2 2 2

Ta chứng minh:
2 x  y  z
2

 1  3
x  y  z 2   x  y  z   18
2 2

Thật vậy ta có:

176
Page: Tài Liệu Môn Toán
Website: tailieumontoan.com

x 2  y 2  z 2   x  y  z   18
  x  y  z    x  y  z   2  xy  yz  zx   18
2

  x  y  z    x  y  z   12  0
2

Nên:
(3)  2  x  y  z   x 2  y 2  z 2   x  y  z   18
2

 x2  y 2  z 2  x  y  z  6
Mặt khác, do x, y, z là các số dương nên ta có:
x 2  y 2  z 2  xy  yz  zx
x  y  z  3  xy  yz  zx 
Mà xy  yz  zx  3 nên bất đẳng thức (3) đúng. Từ (1), (2) v| (3), ta có điều phải chứng
minh.
Đẳng thức xảy ra khi và chỉ khi x = y = z = 1.
Câu 3. Gọi I l| trung điểm cảu BC. Ta có:
1
  1
 
FQ.AI  FA  AQ AB  AC  FA.AB  FA.AC  AQ.AB  AQ.AC
2 2

1
 1
 
 0  AF.AC  AQ.AB  0  AF.AC cos FAC  AQ.AB.cos QAB
2 2

 Do AF  AB, AQ  AC, FAB  QAC  90  A  0

 FQ  AI hay FQ  A1G (1)


Ta cos CGB2P là hình bình hành nên GB2 song song và bằng CP nên GB2 song song và
bằng AQ, suy ra AQB2G là hình bình hành, vậy có QB2
song song và bằng AG. Suy ra QB2 song song và bằng
FC2. Nên FQB2C2 là hình bình hành, hay FQ song song
với B2C2 (2)
Từ (1) và (2) suy ra A1G  B2C2
Tương tự cũng có:
B1G  A2C2 ,C1G  A2 B2 .
Vậy c{c đường thẳng đi qua A1, B1, C1 tương ứng vuông
góc với B2C2, C2A2, A2B2 đồng quy tại G nên theo hệ quả
của định lí Carnot, ta có c{c đường thẳng đi qua A2, B2,
C2 tương ứng vuông góc với B1C1, C1A1, A1B1 cũng đồng
quy.
Câu 4: Giả sử m, n là các số tự nhiên thỏa mãn: 4m3 + m = 12m3 + n. Chứng minh rằng m
– n là lập phương của một số nguyên.
Ta có: 4m3  m  12m3  n  4  m3  n 3    m  n   8n 3

177
Page: Tài Liệu Môn Toán
Website: tailieumontoan.com

  m  n   4m2  4mn  4n 2  1  8n 3 (1)


Giả sử p là một ước nguyên tố chung m – n và 4m2  4mn  4n 2  1
Do 4m2  4mn  4n 2  1 là số lẻ nên p là số lẻ.
Từ (1) suy ra 8n 3 p mà p là số nguyên lẻ  n p  m p
Mặc kh{c p l| ước của 4m2  4mn  4n 2  1  p  1 (vô lí)
Do đó m – n và 4m2  4mn  4n 2  1 không có ước nguyên tố chung, suy ra
 m  n, 4m2  4mn  4n 2 1  1
Do 8n 3   2n  , suy ra m – n là lập phương của một số nguyên.
3

Câu 5: Tập M có 144 điểm được tô bằng 3 màu nên tồn tại 1 m|u tô được tô ở không ít
144
hơn  48 điểm.
3
Ta chọn trong c{c điểm của M đúng 48 điểm được tô cùng m|u. Chia c{c điểm của M
th|nh 12 h|ng (c{c điểm có cùng tung độ) và 12 cột (c{c điểm có cùng ho|nh độ).
Gọi ai (i = 1,..., 12) là số điểm trong 48 điểm được chọn có trong một cột thứ i suy ra:
12

a
i 1
i  48

ai  ai  1
Khi đó, số cặp điểm được chọn trong cột thứ i là:
2
12
a i  a i  1
Số cặp điểm có ho|nh độ trùng nhau là: 
i 1 2
  12 2 
a i  a i  1 1  12 2 12  1   
 a i 

12 12

    ai   ai   i 1
  a i   72
i 1 2 2  i 1 i 1  2  12 i 1

 
 
Ta có:
Vì mỗi cặp được chọn trong cùng một cột tương ứng với một cặp h|ng trong đó c{c
điểm trong một hàng có cùng tung độ.
Số các cặp hàng khác nhau là: C122  66
Vì 72 > 66 nên luôn tìm được hai cặp điểm nằm trên 1 cặp hàng.
Vậy luôn tồn tại một hình chữ nhật có các cạnh song song với trục tọa độ v| có 4 đỉnh tô
cùng một màu

Cn  10.
10n  1 Cn
.  10
10n  1 n 1 n
3 2  10 10n  1 3 2
3

9 2 9
Đề 32
2 7
Câu 1. Điều kiện:  x 
3 3

178
Page: Tài Liệu Môn Toán
Website: tailieumontoan.com

Đặt a  7  3x , b  3x  2  a, b  0 . Suy ra
a 2  b 2  5
 2
 2b  1 .a   2a  1 b  2ab  11
2

 s 2  2 p  5 2 p  s 2  5 2 p  s 2  5
  2 
2sp  s  2 p  11  s  s  5   s  s  5  11  s 3  s 2  4s  6  0
2

 s  a  b, p  ab 
 a  2

2 p  s  5
2
p  2  b  1 x  1
     
 s  3  s  2s  2   0
 a 1
s  3  x2
2
 
 b  2
Thử lại thỏa mãn. Vậy nghiệm phương trình l| x = 1 hoặc x = 2.
Câu 2.
Gọi CD  AB tại D. Khi đó AP, BQ, CD đồng quy nên T, B, D, A l| h|ng điểm điều hòa
TDBA  1 .
Do đó ta có TM.TD = TA.TB.
Xét hai đường tròn ngoại tiếp hai tam giác CDM và ngoại tiếp tứ giác ABPQ, tâm của
hai đường tròn n|y đều nằm trên CM.
Nhưng TM.TD = TA.TB v| HP.HA = HQ.HB nên H, T nằm trên trục đẳng phương của
hai đường tròn nói trên. Vậy HT  CM.
Câu 3. Đặt g  x   x3  x thì f  f  x    g  x  . Suy ra
3
4
 
f  g  x   f f  f  x   g  f  x 
Dễ thấy g  x  l| đơn {nh nên từ f  f  x    g  x  suy ra f(x) cũng l| đơn {nh.
 1 1
Gọi x0 là một điểm cố định của hàm g  x   g  x0   xo  x0  0;  ;  .
 2 2
Ta có f  x0   f  g  x0    g  f  x0   . Suy ra f  x0  cũng l| một điểm cố định của hàm
 1 1
g  x  , f  x  là một song ánh trên tâp D  0;  ;  nên:
 2 2
 1 1 1 1
f     f  0  f      0   0
 2 2 2 2
Từ đó ta có điều phải chứng minh.
Câu 4. Ta có: a 4  b4  c 4  abc  a  b  c   k  ab  bc  ca 
2

2
Vì bất đẳng thức đúng với mọi gi{ trị a,b,c nên đúng với a = b = c = 1  k 
3
179
Page: Tài Liệu Môn Toán
Website: tailieumontoan.com

2
Ta chứng minh k  l| gi{ trị lớn nhất
3
2 2
Xét k  bất đẳng thức trở th|nh a 4  b4  c 4  abc  a  b  c    ab  bc  ca 
2
(1)
3 3
 3  a 4  b4  c4    a 2b2  b2c 2  c 2 a 2   abc  a  b  c 

[p dụng bất đẳng thức AM – GM ta có:

a 4
 b4    b4  c 4    a 4  c 4   2a 2b2  2b2c 2  2c 2a 2

Suy ra  3  a 4  b4  c 4   3  a 2b2  b2c 2  c 2 a 2  (2)

Mặt kh{c

a b 2 2
 b2c 2  c 2 a 2   abc  a  b  c  
1
2
2 1
2
2 1
 ab  bc   bc  ca    ca  ab   0 (3)
2
2

2
Từ (2) v| (3) suy ra (1) được chứng minh. Vậy số k lớn nhất k 
3
Câu 5. Xét n  2k t với k,t là cac số tự hiên và t là số lẻ.
Đặt 2013n  1  a n  1
k

 
k t k


a n  1  a 2 t  1  a 2  1  a 2  1  a 2

 
k t 1
 ...  a 2  1
k


Do t là số lẻ nên a n  1 22014  a2  1 22014
k

 k 1


Ta có a 2  1   a 2  1 a 2  1 a k  1 ... a 2  1 a chia 4 dư 1 nên a 2  1 chia 4 dư 2.
k 1

Do đó a n  1 22014   k  1  3  2014
Từ đó suy ra gi{ trị nhỏ nhất của n cần tìm là n  22012

Đề 33
Câu 1. a) Lập phương trình đường thẳng song song với đường thẳng d sao cho ∆ cắt (P)
tại 2 điểm ph}n biệt A, B v| AB = 1.
Đường thẳng ∆ song song với d có dạng y = x + m (m ≠ 3)
Phương trình ho|nh độ giao điểm 4 x2  x  1  m  0 (1)

Để ∆ cắt (P) tại hai điểm ph}n biệt A, B thì (1) có 2 nghiệm ph}n biệt, điều kiện l|
15
0m
16
Gọi x1 , x2 l| hai nghiệm ph}n biệt của (1). Theo định lý Viet ta có

180
Page: Tài Liệu Môn Toán
Website: tailieumontoan.com

1 1 m
x1  x2  ; x1 x2 
4 4
A  x1 ; x1  m  , B  x2 ; x2  m 

AB  1  2  x2  x1   1  2  x2  x1   4 x1 x2   1
2 2
 
1 1 m  23
 2   4.  1 m 
 16 4  16
23
Kết hợp điều kiện ta được m 
16
b) Goi I l| đỉnh của (P); A, B l| hai điểm ph}n biệt, không trùng với đỉnh v| nằm
trên (P) sao cho IA vuông góc với IB. Tìm quỹ tích điểm N của AB khi A, B thay đổi.
Gọi A  a; 4a 2  1 nằm trên (P), đỉnh I  0;1 .

Đường thẳng IB qua I (0;1), nhận IA  a; 4a 2  l| vecto ph{p tuyên. Phương trình của
đường thẳng IB l| x  4ay  4a  0


 y  4x 1
2
 1 1 
Tọa độ của B l| nghiệm của hệ phương trìn:   B ;  1
 x  4ay  4a  0  16a 64a 
2

a 1 1 
N l| trung điểm của AB, suy ra N   ; 2a 2   1
 2 32a 
2
128a
5 5
Nhận xét yN  8 xN2  . vậy quỹ tích của điểm N l| Parabol y  8 x 2 
4 4

Câu 2. 1. Ta có x  1  x 2  1  x x
Điều kiện x  1
 x x  x  1  0
pt  x 2  1  x x  x  1  
 x 2  1  x3  x  1  2 x x 2  x
 x  1

 x  x  1  2 x  x  1  1  0
 x  x  1  1 1 5
 x
 x  1 2
1 5
Vậy phương trình có nghiệm x 
2

181
Page: Tài Liệu Môn Toán
Website: tailieumontoan.com

 x 2  21  y  1  y 2 (1)

2. Ta có: 

 y  21  x  1  x (2)
2 2

Điều kiện: x  1; y  1
Trừ vế với vế của (1) cho (2) ta có
x 2  21  y 2  21  y  1  x  1  y 2  x 2
x2  y 2 yx
    y  x  y  x 
x  21  y  21
2 2
y 1  x 1
 x y 1 
  x  y   x y  0
 x 2  21  y 2  21 y 1  x 1 
 
x y 1
 x  y vì   x  y  0 x  1; y  1
x 2  21  y 2  21 y 1  x 1
Thay x = y vào (1) ta có
x 2  21  x  1  x 2  x 2  21  5  x  1  1  x 2  4
x2  4 x2
    x  2  x  2 
x  21  5
2
x 1 1
 x2 1 
  x  2   x2 0
 x  21  5
2
x 1 1 
   x 2  21  4  1 
  x  2   x  2   0
  x 2  21  5  x  1  1 
   
  x  21  4 
2
1
 x  2 vì  x  2    0
 x 2  21  5  x  1  1
 
Vậy hệ phương trình có nghiệm (2;2)
Câu 3.

182
Page: Tài Liệu Môn Toán
Website: tailieumontoan.com

1. Gọi Q l| giao điểm của AP v| BC, suy ra P l| trung điểm của AQ, tam giác ACQ
cân tại C.
CQ = CA = b suy ra BQ = BC – CQ = a – b.
a c b a c b 1 a b 
BN  .BA, BM  BC; BP   BA  BC 
2c 2c 2 a 
a c b a c b
b) MN  BN  BM  .BA  .BC
2c 2c
1 c
PM  BM  BP   BA  BC
2 2a
c
PM  .MN suy ra PM cùng phương với MN do ddos P, M, N thẳng
a c b
hàng.
2. Cho tam gi{c ABC có AC = b, BA = a, AB = c l| độ dài ba cạnh của tam giác;
ma , mb , mc l| độ d|i ba đường trung tuyến lần lượt xuất phát từ A, B, C. Gọi R, S
lần lượt l| b{n kính đường tròn ngoại tiếp, diện tích của tam giác ABC. Chứng
1 1 1 3
minh rằng nếu    thì tam gi{c ABC đều.
abmc bcma camb 2RS
1 1 1 3 1 1 1 2 3
      
abmc bcma camb 2 RS abmc bcma camb abc
c a b
   2
3mc 3ma 3mb
a a2 2a 2
 
3ma 2b 2  2c 2  a 2 3a 2b 2  2c 2  a 2
3a
4
3a 2  2b2  2c 2  a 2
Vì 3a 2b2  2c 2  a 2   a 2  b2  c 2
2
2
a 2a
Nên  2
3ma a  b  c
2 2

Chứng minh tương tự ta có


b 2b2 c 2c 2 a b c
 2 ,  Vậy   2
3mb a  b  c 3mc a  b  c
2 2 2 2 2
3ma 3mb 3mc
3a 2  2b 2  2c 2  a 2

Dấu bằng xảy ra khi và chỉ khi 3a 2  2b 2  2c 2  a 2  a  b  c
 2
3a  2b  2c  a
2 2 2

Hay tam gi{c ABC đều


Câu 4.Vì tam giác ABC cân tại A nên hóc CBH bằng góc BCK

183
Page: Tài Liệu Môn Toán
Website: tailieumontoan.com

Suy ra cos(BC, BH) = cos(BC, CK)


Đường thẳng BC có vecto pháp tuyến n BC  1; 2 
Đường thẳng CK có vecto pháp tuyến nCK   4;3
Gọi vecto pháp tuyến của đường thẳng BH là n BH   a; b   a 2  b2  0 
cos(BC, BH) = cos(BC, CK)
n BC .n BH n BC .nCK a  2b 10
   
n BC n BC n BC nCK 5. a  b
2 2
5.5

  a  2b   4  a 2  b 2   3a 2  4ab  0
2

+) Nếu b = 0 thì a = 0 (loại).


+) Nếu b ≠ 0, chọn b = 3 suy ra a = 0 hoặc a = 4
+) Nếu a = 4; b= 3 thì n BH = (4;3) suy ra n BH = nCK (loại)
+) Nếu b = 3, a = 0 suy ra phương trình BH l| y – 6 = 0
 x  2 y  17  0
Tọa độ của C là nghiệm của hệ phương trình   C 1;8
4 x  3 y  28  0
Phương trình của AC: x =1
B l| giao điểm của BH và BC suy ra B(5;6)
Phương trình của BA: 3x – 4y + 9 = 0
A l| giao điểm của AB và AC suy ra A(1;3)
Diện tích tam giác ABC:
BC  20
1  6  17
d  A, BC   2 5
5
1
S ABC  BC.d  A, BC   10
2
1 1 4
Câu 5. Ta có:   .
a  b b  c a  2b  c
1 1 4 1 1 4
Chứng minh tương tự ta có   ,  
b  c a  c a  2c  b a  b a  c b  2a  c
1 1 1  1 1 1 
Suy ra    2   
ab bc ac  b  2a  c a  2b  c b  2c  a 
1 4
Ta chứng minh  2
b  2a  c a  28

184
Page: Tài Liệu Môn Toán
Website: tailieumontoan.com

1 4
 2
b  2a  c a  28
 a 2  28  4b  8a  4c  2a 2  b 2  c 2  16  4b  8a  4c  0

 2  a  2    b  2    c  2   0 (điều n|y luôn đúng)


2 2 2

1 1 1 4 4 4
Vậy    2  2  2
b  2a  c a  2b  c b  2c  a a  28 b  28 c  28
1 1 1 8 8 8
Suy ra    2  2  2
a  b b  c c  a a  28 b  28 c  28
Dấu bằng xảy ra khi và chỉ khi a = b = c = 2.
Đề 34
Câu 1. Ta có: 2 x  x 2  2 x  7  4  4 2 x  1
1
Điều kiện: x  
2
Phương trình   x  2  3  2 x  1  2  x  2   4 2 x  1
2

1
+) Xét x   không thỏa mãn phương trình.
2
2  x  2
2
1  x2 
+) Xét x   ta có phương trình    3  4
2  2x 1  2x 1
x2
Đặt t  ta có t 2  3  2t  4  t 2  3  2t  4  t  1
2x 1
x2
Với t  1 , ta có  1  2x 1  x  2  x  3  6
2x 1
Đối chiếu điều kiện, ta có phương trình có nghiệm duy nhất là x  3  6
Ghi chú: HS có thể giải bằng c{ch đặt a  2 x  1, b  x  2
4 x 2  2 y 2  4 x  4 y  2 xy  x  y (1)

Câu 2. Ta có: 
 2 x  1  2  x  y   3  8 x y  4 y  4  x  y   1 (2)

2 2

Điều kiện x  y;2 x  1  0;2  x  y   3  0


PT 1   4 x 2  2 xy  2 y 2   4  x  y   x  y  0

  x  y  4 x  2 y   4  x  y   x  y  0

 x  y  x  y  4 x  2 y   4 x  y  1  0

x  y

 x  y  4 x  2 y  4   1  0 (3)

185
Page: Tài Liệu Môn Toán
Website: tailieumontoan.com

Phương trình (3) vô nghiệm vì


x  y  4 x  2 y  4   1  x  y   2 x  1   2 x  2 y  3   1  0
Thay x = y v|o phương trình (2), ta có 2 x  1  4 x  3  8 x3  4 x 2  8 x  1
  2x 1  2    
4 x  3  3  8 x3  4 x 2  8 x  6

2x  3 2  2 x  3
    2 x  3  4 x 2  4 x  2 
2x 1  2 4x  3  3
 3
x  2

 1

2
  2 x  1  1 (3)
2
 2 x  1  2 4x  3  3
1 2 1 1
Vế tr{i phương trình (3):    1
2x 1  2 4x  3  3 2 2
1
Dấu “=” xảy ra khi x  
2
1
Vế phải phương trình (3):  2 x  1  1  1 dấu “=” xảy ra khi x  
2

2
1
Do đó, phương trình (3) có nghiệm là x  
2
Đối chiếu điều kiện, ta được nghiệm của hệ phương trình đã cho l|
 3 3 1 1 
 x; y    ;  ;   ;  
 2 2   2 2  
Câu 3. 1. Áp dụng định lý số sin trong tam giác, ta có
sin 2014 A  sin 2014 B  sin 2014 C  a2014  b2014  c2014
Ta có a2014  b2014  c2014  b2014  a  b; a 2014  b2014  c 2014  c 2014  a  c
Do đó, ta chứng minh tam giác ABC nhọn, ta chứng minh góc A nhọn.
Ta có a2014  b2014  c2014  a 2012 .b2  a 2012 .c 2  a 2  b2  c 2
b2  c 2  a 2
Suy ra cos A   0  A nhọn, điều phải chứng minh.
2bc
2. Ta có S ABHK  7S ABC  S ABC  8S CHK
1
CA.CB sin C
S 2 CA.CB
ABC
   8 (1)
S 1 CK .CH
CHK CK .CH sin C
2
CH
AHC vuông tại H, ta có cos C  (2)
CA
CK
BKC vuông tại K, ta có cos C  (3)
CB

186
Page: Tài Liệu Môn Toán
Website: tailieumontoan.com

1
Từ (1), (2), (3) ta có cos 2 C 
8
CH CK HK CH 1
 và góc C chung  HCK ACB    cos C 
CA CB AB AC 2 2
 AB  2 2HK  2 14 (4)
Bán kính đường tròn ngoại tiếp tam giác ABC là:
AB AB 2 14
R   4
2sin C 2 1  cos 2 C 1
2 1
8
2
 AB  S
  8
ABC
Lưu ý: Ở (4), có thể sử dụng tỉ số đồng dạng 
 HK  S CHK

Câu 4. Đường tròn (C) có tâm I(-1;-4), bán kính R  3


Gọi A v| B l| hai giao điểm của (C) v| (C’)
Gọi H l| giao điểm của EI với AB.
Từ giả thiết ta có IA = IB = AB = 3 do đó IAB đều.
3
Do đó đường cao IH  , IE  5IH
2
Xảy ra 2 trường hợp sau:
Trường hợp 1: H nằm giữa E và I
(C’) có bán kính
2
AB 2  3 3
R '  EA  EH 2  HA2   EI  IH     5     13
2

4  2 4
Khi đó PT đường tròn (C’) l|  x  3   y  1  13
2 2

Trường hợp 2: I nằm giữa E v| H. (C’) có b{n kính


2
AB 2  3 3
R '  EA  EH  HA   EI  IH     5     43
2 2 2

4  2 4
Khi đó PT đường tròn (C’) l|  x  3   y  1  43
2 2

1 1 1 3
Câu 5. Ta có:  2  2 
x  2 y z  1 y  2x z  1 z  2x y  1 4
2 2 2 2 2 2 2

Áp dụng bất đẳng thức Cauchy, ta có 3xyz  x  y  z  3 3 xyz  xyz  1


Ta có x2  2 y 2 z 2  1  2 x  2 y 2 z 2  4 xy 2 z 2  4 yz
1 1 1 1 
Suy ra   1   dấu “=” xảy ra khi x  y  z  1
x  2 y z  1 4 yz 8 
2 2 2
yz 
Do đó, ta có

187
Page: Tài Liệu Môn Toán
Website: tailieumontoan.com

1 1 1 1 1 1 1 3
 2  2  3    
x  2 y z  1 y  2x z  1 z  2x y  1 8 
2 2 2 2 2 2 2
xy yz zx  4
1 1 1
(Vì 3xyz  x  y  z     3)
xy yz zx
Dấu “=” của bất đẳng thức xảy ra khi x  y  z  1. Điều phải chứng minh.

Đề 35

 y  3x y  28
3 2

Câu 1. Ta có:  2
 x  6 xy  y  6 x  10 y

2

Nhận xét

y 3  3x 2 y 
 y3  3 y 2 x  3 yx2  x3    y3  3 y 2 y  3 yx2  x3    y  x 3   y  x 3
2 2
 y  x   y  x
3 3
 yx  yx
3 3

y  3x
3 2
y  28   28       7
2  2   2 
yx yx yx yx yx yx
x  ,y  . Đặt a  ,b 
2 2 2 2 2 2
a  b  7
3 3

Biến đổi hệ ban đầu về hệ:  2


a  a  2  b  2b 
2

a 3  b3  7 a 3  b3  7
 3 
 a  3a 2
 3a  1    b 3
 6 b 2
 12 b  8   a  1    b  2 
3 3

b 2  b  2  0  a  1, b  2
 
a  b  1  a  2, b  1
 x  3, y  1
Tìm được 
 x  3, y  1
Câu 2.

188
Page: Tài Liệu Môn Toán
Website: tailieumontoan.com

Gọi O l| t}m đường tròn ngoại tiếp tam gi{c ABC, P l| tiếp điểm đường tròn (O) với
AB, giao điểm của MN với AO l| I.
Do AO l| tia ph}n gi{c góc BAx nên hai điểm P v| N đối xứng nhau qua AO.
Suy ra (PI; PO) = (NI; NO) = (MI;MO)(modπ) (do tam gi{c MIN c}n). Từ đó suy ra 4
điểm P, I, O, M, cùng thuộc một đường tròn (1).

Mặt kh{c (O) tiếp xúc với cạnh AB, BC ở P v| M nên OPB  OBM  900 suy ra tứ gi{c
AMBP nội tiếp đường tròn đường kính BO (2)
Từ (1) v| (2) suy ra 5 điểm B, M, I, O v| P cùng thuộc đường tròn b{n kính BO. Do đó
BIO  BPO  900
dẫn đến I l| hình chiếu của P trên AO.
do góc BAx cố định v| B cố định trên đường thẳng AO cố định v| suy ra điểm I cố
định.
Vậy đường thẳng MN đi qua điểm I cố định.
Câu 3.
 x  zy  0
Không mất tổng quát, giả sử x  y  z   do x, y, z  (0;1)
 y  zx  0
+) Nếu z – xy < 0 khi đó VT ≤0≤ VP, bất đẳn thức đúng.
+) Nếu z – xy ≥ 0, ta chứng minh bất đẳng thức sau: với mọi a, b, c thuộc (0;1) ta có:
bc 1  a    b  ac  c  ab 
Thật vậy: bc 1  a    b  ac  c  ab   bc 1  a    b  ac  c  ab 
2

 bc 1  2a  a 2   bc  ab2  ac 2  a 2bc  a  b  c   0
2

Áp dụng bất đẳng thức trên cho x, y, z thuộc (0;1) ta được

189
Page: Tài Liệu Môn Toán
Website: tailieumontoan.com

yz 1  x    y  xz  z  xy 
zx 1  y    z  xy  x  yz 
xy 1  z    x  yz  y  xz 
Nhân vế với vế của ba bất đẳng thức trên ta thu được:
xyz 1  x 1  y 1  z    x  yz  y  xz  z  xy  (điều phải chứng minh)
Câu 4.Ta có:
m3  n3  4  0  mod p   mn  m  n   4  0  mod p 
 3mn  m  n   12  0  mod p 
Kết hợp m3  n3  4  0  mod p  suy ra  m  n   8  0  mod p 
3

 m  n  2  m2  n2  2mn  2m  2n  4  0  mod p 
Do p là số nguyên tố nên có hai khả năng xảy ra:
Trường hợp 1:
m  2
Nếu m  n  2 m2  n 2  m  n  2  m  m  1  n  n  1  2  
n  2
Thử lại thấy (m; n) = (1; 2),(2; 1),(1; 1) thỏa mãn
Trường hợp 2: m2  n2  2mn  2m  2n  4 m2  n2 , viết lại:
m2  n 2  2mn  2m  2n  4  2  m  1 n  1  1

  m  1   n  1  2   m  1  1   n  1  1  m2  n 2
2 2 2 2
   
Dấu bằng chỉ xảy ra khi m = n = 1.
Vậy trong mọi trường hợp ta tìm các bộ số thỏa mãn là (1; 2),(2; 1),(1; 1)

Câu 5.
a)Vì c{c số thực được điền v|o c{c ô vuông l| những số nguyên dương nên tồn tại
số a nhỏ nhất trong c{c số được điền.
Giả sử tồn tại một ô vuông cơ sở có chứa a m| 4 ô vuông cơ sở có cạnh liền kề có
ít nhất một ô vuông có chứa số b a. gọi c d e l| ba
số ở ba ô vuông cơ sở có cạnh liền kề còn lại.
b  a 1
Khi đó   (b x  d  e)  a trái với giả
c , d , e  a 4
thiết.
Như vậy nếu có một ô vuông có chứa số a thì 4 ô
vuông có cạnh liền kề với nó cũng chứa số a.

190
Page: Tài Liệu Môn Toán
Website: tailieumontoan.com

Do đó tất cả c{c ô vuông đều chứa số a.


b) Nếu c{c số được điền l| c{c số hữu tỉ thì 4 số ở 4 ô vuông có cạnh liền kề với ô
vuông cơ sở không nhất thiết phải bằng nhau.
Ta x}y dựng một hệ trục tọa độ vuông góc có c{c trục tọa độ song song hoặc
trùng với c{c cạnh của lưới ô vuông v| có đơn vị trên mỗi trục bằng độ d|i cạnh
của ô vuông cơ sở. Ở mỗi hình vuông cơ sở ta điền một số bằng trung bình cộng
hai tọa độ t}m của hình vuông đó.
Khi đó do tọa độ của t}m c{c hình vuông cơ sở đều l| số hữu tỉ nên số đặt v|o đó
cũng l| số hữu tỉ.V| số đặt v|o 4 ô vuông cơ sở có cạnh kề với nó không bằng
nhau.
Ta chứng minh số điền v|o c{c ô vuông cơ sở bằng trung bình cộng của 4 số ở 4 ô
vuông có cạnh liền kề với nó như sau:
Không mất tổng qu{t giả sử có hình vẽ bên.
khi có t}m của 4 hình vuông cơ sở A B C D l| 4 đỉnh của một hình vuông nhận
t}m I của hình vuông cơ sở ở giữa l|m t}m, nên dẫn đến tọa độ điểm I l| trung
bình cộng của độ của c{c điểm A B C D do đó được đặt trong hình vuông tâm I là
số hữu tỉ v| l| trung bình cộng của 4 số hữu tỉ được đặt trong c{c hình vuông t}m
ABCD

Đề 36
2
Câu 1. Điều kiện: x 
2
Phương trình tương đương với: 2  3x  1 2 x 2  1  10 x 2  3x  6
 4  2 x 2  1  2  3x  1 2 x 2  1  2 x 2  3x  2  0 (1)

Đặt 2 x 2  1  t  t  0  khi đó phương trình (1) trở thành:


4t 2  2  3x  1 t  2 x 2  3x  2  0 (2) phương trình (2) có nghiệm:
 '   3x  1  4  2 x 2  3x  2    x  3 phương trình (2) có nghiệm:
2 2

 3 x  1   x  3 x  2 x2
t     2x2 1
 4 2 2

 3 x  1   x  3 2 x  1  2 x  1
t    2 x2  1
 4 2 
 2

191
Page: Tài Liệu Môn Toán
Website: tailieumontoan.com

  x  2
 2  2  60
 7 x  4 x  8  0 x 
 
7
1  (thoả điều kiện )
 x  2  60
 
2 x 
 2  7
 4 x  4 x  5  0
2  60 2  60
Vậy phương trình có 2 nghiệm x  ,x
7 7
Câu 2. Điều kiện: x  0; y  0;; y  3x  0.
+ Nhận xét x  0, y  0
 12  12 2  1 3
(1  y  3x ) x  2 1  y  3x  x  x  y 1
  
+ Với x  0, y  0   
(1  12 ) y  6 1  12  6  1  3  12
 y  3x  y  3x y  x y y  3x

1 9 12 y y y y
   y 2  6 xy  27 x 2  0  ( )2  6( )  27  0   3;  9 (loại)
x y y  3x x x x x
y
   
2 2
Với  3  y  3x suy ra x  1  3 ; y  3 1  3
x
Câu 3. Từ giả thiết 9  a 4  b4  c 4   25  a 2  b2  c 2   48  0

 25  a 2  b 2  c 2   48  9  a 4  b 4  c 4   48  3  a 2  b 2  c 2 
2

 3  a 2  b 2  c 2   25  a 2  b 2  c 2   48  0  3  a 2  b 2  c 2 
2 16
3
Biến đổi
a2 b2 c2
P  
b  2c c  2a a  2b
 a 2  b2  c 2 
2
a4 b4 c4
 2   
a  b  2c  b2  c  2a  c 2  a  2b  a 2b  b 2c  c 2 a  2  a 2c  b 2 a  c 2b 

Lại có: a 2b  b2c  c 2 a  a.ab  b.bc  c.ca  a  b


2 2
 c 2  a 2b 2  b 2c 2  c 2a 2 

 a bb cc a  a b c
2 2 2 2 2 2 a 2
 b2 c 2 2

3
a  b2  c 2 
2 2

Tương tự  a c  b a  c b  a  b  c
2 2 2 2 2 2

3
a b c 2 2 2
Từ đó P  1
3
Dấu bằng xảy ra khi a = b = c =1. GTNN của P = 1
192
Page: Tài Liệu Môn Toán
Website: tailieumontoan.com

Câu 4. Cách 1
MA HB NC
Xét tích T  . . (1)
MB HC NA
DB AB
Do AD là phân giác của BAC nên  (2)
DC AC
Do tứ giác AMDN nội tiếp nên ta có
BM .BA  BH .BD, CN.CA  CD.CH
BA BH CD CN
  ,  (3)
BD BM CA CH
Do AD là phân giác của MAN v| AD l| đường kính
nên AM = AN (4)
Thay (2), (3), (4) v|o (1) ta được
MA HB NC BA CD BA CD
T . .  1. .  . 1
NA MB HC BD CA CA BD
Do đó c{c đường thẳng CM, BH, AH đồng quy.
Cách 2
Ta chứng minh bài toán cho cả elip v| đường tròn như sau: “Elip hoặc đường tròn (E)
cắt cạnh BC, CA, AB của ABC ở A1,A2; B1,B2; C1,C2. Chứng minh rằng nếu AA1, BB1,
CC1 đồng quy thì AA2, BB2, CC2 cũng vậy”
Thật vậy, áp dụng định lý carnaot: “Cho đường cong bậc hai:
F  x, y   ax 2  2bxy  cy2  2dx  2ey  f  0  C 
Ai, Bi, Ci (i = 1, 2) lần lượt chia cạnh BC, CA, AB của
∆ABC theo tỉ số i , i ,  j (Ai, Bi, Ci ≠ đỉnh). Vậy thì: Ai, Bi, Ci ∈
(C)  1212 1 2  1 " , ta có:
AA1 , BB1 ,CC1 đồng quy  111 = -1 nên từ
1212 1 2  1  22  2  1  AA2 , BB2 ,CC2 đồng quy.
Quay trở lại bài toán trên, ta thấy đường tròn đường kính AD cắt ba cạnh của tam giác
ABC tại 6 điểm H, D; N,A; A,M m| AD, BA, CA đồng quy tại A nên AH, BN, CM đồng
quy.
Câu 5. Vì (17, 10) = 1 (1) và 17 là số nguyên tố nên theo định lý Fecma nhỏ ta có:
1017 10 17  10 1016 1 17 (2)
Từ (1) và (2) suy ra 1016  1 17  1016  1 mod17 
Do đó, với mọi n nguyên dương thì 1016.n  1 mod17   1016.n  1 17
Mặt khác 1016.n  1  99...9
n.16

Vậy có vô số số hạng của dãy 9; 99; 999;9999;... chia hết cho 17

Đề 37
193
Page: Tài Liệu Môn Toán
Website: tailieumontoan.com

Câu 1. Điều kiện: x  3


2
Bất phương trình tương đương với 7 x 2  7 x  9  
  x  2  x  3   2 2 x 1 

 6 x 2  14 x  7  4  x  2  x  3. 2x 1
 3  2 x 2  5 x  3  4  x  2  x  3. 2x 1   x  2  0

2 x2  5x  3 2 x2  5x  3 18 x 2  46 x  29  0
 3. 4 1  0   2
x2 x2 2 x  6 x  5  0
 23  1051
 x 
 18

   x  23  1051
 18
 3  19 3  19
 x
 2 2
23  1051 3  19
Kết hợp với điều kiện đã x{c định, ta được x
18 2
Câu 2. Vì tứ giác ABCD nội tiếp nên theo định lí Ptoleme ta có
AB.CD + CD.BC = AC.BD (1)
Vì AP, CP tương ứng là phân giác góc A và C nên
AB PB CB
   AB.CD  AD.BC (2)
AD PD CD
Từ (1) và (2) suy ra 2AB.CD = AC.BD
M| Q l| trung điểm BD nên BD = 2BQ
AB BQ
Do đó: Ab.CD = AC.BQ hay  . Mà
AC CD
ABQ  ACD (góc nội tiếp chắn cung AD) nên
ABQ ACD  AQB  ADC

Mà AQB  DQK (đối đỉnh); ADC  DCK (số lẽ trong) (*)

Suy ra DQK  DCK  tứ giác CQDK nội tiếp  BQC  CK (**)

Chứng minh tương tự QBC DAC  BQC  ADC (***)

Từ (*), (**), (***)  DCK  CKD


194
Page: Tài Liệu Môn Toán
Website: tailieumontoan.com

Suy ra tam giác DCK cân tại D.


3
Câu 3. Ta chứng minh giá trị nhỏ nhất của S bằng
2
1 1 1
Đặt  a,  b,  c
x y z
a b c
Ta có a, b, c là các số thự dương a + b + c = 3 v| S   
1  b 1  c 1  a2
2 2

Áp dụng bất đẳng thức giữa trung bình cộng v| trung bình nh}n ta được
a ab2 ab2 ab
a a a
1 b 2
1 b 2
2b 2
ab  ab  ca
Viết 2 kết quả tương tự và cộng lại ta được S  a  b  c 
2
3
Dùng a + b + c =3 và (a+b+c)2 ≥ 2(ab + bc + ca) ta có S 
2
3
Mà khi x = y = z =1 thì S  . Suy ra điều phải chứng minh.
2
Câu 4. Lấy 5 điểm tùy ý sao cho không có 3 điểm nào thẳng hàng trong mặt phẳng. Khi
đó vì chỉ dùng hai m|u để tô c{c điểm nên theo nguyên lí Dirichlet phải tồn tại ba điểm
trong số đó cùng m|u.
Giả sử đó l| 3 điểm A, B, C, m|u đỏ
Gọi G là trọng tâm tam giác ABC. Nếu G có m|u đỏ
thì ta được tam gi{c có 3 đỉnh và trọng t}m m|u đỏ.
Nếu G m|u xanh. Kéo d|i GA, GB, GC c{c đoạn AA’
, BB’, CC’ sao cho
AA’ = 3GA, BB’ = 3GB, CC’ = 3GC. Gọi M, N, P tương
ứng l| trung điểm BC, CA, AB thì AA’ = 3GA = 6GM,
suy ra AA’ = 2AM. Tương tự BB’ = 2BN, CC’ = 2CP.
Do đó tam gi{c A’BC, B’CA, C’AB tương ứng nhận
A, B, C làm trọng tâm.
Mặt kh{c: ta cũng có tam gi{c ABC, A’B’C’ có trọng
tâm G.
Có hai trường hợp có thể xảy ra:
+) Nếu A’, B’ , C’ có cùng mùa xanh, khi đó tam gi{c A’B’C’ v| trọng tâm G có màu
xanh.
195
Page: Tài Liệu Môn Toán
Website: tailieumontoan.com

+) Nếu ít nhất một trong c{c điểm A’, B’ , C’ m|u đỏ. Không giảm tổng quát, giả sử A’
đỏ. Khi đó tam gi{c A’BC v| trọng t}m A có m|u đỏ.

Câu 5. Đặt 7 x 2  9 xy  5 y 2  A ta có 28A = 14 x  9 y   13.17. y 2 , xét số dư khi chia A


2

cho 9, 13, 17 ta thu được


+) A chia cho 9 không có số dư 3; 6
+) A chia cho 13 không có số dư 1; 3; 4; 9; 10; 12
+) A chia cho 17 không có số dư 1; 2; 4; 8; 9; 13; 15; 16
Theo định lí thặng dư Trung Hoa, tồn tại số nguyên dương n thỏa mãn
n  4(mod 9)

n  2(mod13)
n  0(mod17)

+) n + 7, n + 10 không có dạng 7 x 2  9 xy  5 y 2

+) n + 3, n + 5, n + 6, n + 11, n + 12, n + 14 không có dạng 7 x 2  9 xy  5 y 2

+) n + 1, n + 2, n + 4, n + 8, , n + 9, n + 13, , n + 15, , n + 16 không có dạng 7 x 2  9 xy  5 y 2

Từ đó suy ra tại 16 số n + 1, n + 2; ....; n + 16 thỏa mãn bài toán.

Đề 38

Câu 1. Sử dụng các bất đẳng thức cauchy quen biết cho hai số
T  R  T  R
2 2 2

   , dấu “=” xảy ra  T  R


 2  2
 R A R 2  A2
2

   , dấu “=” xảy ra  R  A


 2  2
 N T  N2 T 2
2

   , dấu “=” xảy ra  N  T


 2  2
T  R   R A  N T 
2 2 2

    ...     T  R  ...  N
2 2 2
Ta thu được 
 2   2   2 
Mặt khác, theo giả thiết thì T là hoán vị của D nên
T  R   R A  N T 
2 2 2

    ...     T  R  ...  N
2 2 2

 2   2   2 

196
Page: Tài Liệu Môn Toán
Website: tailieumontoan.com

2014 1007
Suy ra T= R= A= I= H= E = P= N = nên N =
8 4
3
Câu 2. Điều kiện: x  . Biến đổi phương trình về dạng
2
 
 x  r3  2 x  . x  3  2 x 1  0
Từ đó tìm được x = - 3 hoặc x  2 , đều thỏa mãn.
 2
2 x  y 2  8 y  4  0 (2)
 2
Câu 3. Ta có (1)  4 x  y 2  20 x  2 y  24  0 (3)

 y  4; x  5
 2
Nhân hai vế của (3) với 2 cộng với các về tương ứng (2), ta thu đượcc phương trình
10x2 + 3y2 – 40x + 12y + 52 = 0  10(x – 2)2 + 3(y + 2)2 = 0
Suy ra x = 2; y = - 2. Nghiệm (x;y) = (2;-2) thỏa mãn.
Câu 4. Áp dụng định lý Ptoleme cho tứ giác nội tiếp CA2BA ta thu
được
CA2.BA + BA2.AC = BC.AA2
Vì BA2 = CA2 nên suy ra CA2(AB + AC) = BC. AA2 và
BC
AA 2 
AB  AC
A1A 2 AA
Ngoài ra, ta có  1 2
BA2  A2C 2CA2
Mặt khác do CA1 A2 đồng dạng với ACA2 nên suy ra
A1A 2 CA2

2CA2 2A1A 2
A1A 2 BC
Từ đó, ta có  (1)
BA2  A2C 2  AB  AC 
B1 B2 AC
Tương tự, ta có  (2)
CB2  B2 A 2  AB  BC 
C1C2 AB
Và  (3)
AC2  C2 B 2  AC  BC 
Cộng (1), (2), (3) vế theo vế ta thu được
A1A 2 B1B2 C1C2 BC AC AB
    
BA2  A2C CB2  B2 A AC2  C2 B 2  AB  AC  2  AB  BC  2  AC  BC 
Áp dụng bất đẳng thức Nesbit cho ba số dương AB, AC, BC, ta được bất đẳng thứ cần
phải chứng minh
A1 A2 B1B2 C1C2 3
  
BA2  A2C CB2  B2 A AC2  C2 B 4
197
Page: Tài Liệu Môn Toán
Website: tailieumontoan.com

Đẳng thuwscc xảy ra khi và chỉ khi ∆ABC l| tam gi{c đều.
Câu 5. Giả sử ngược P(x) ph}n tích được thành tích của hai đa thức bậc hớn hơn 0 với
hệ số nguyên thì một thừa số l| đa thức bậc nhất.
P(x) = ax3 + bx2 + cx +d = (mx + n)(rx2 + ux + s)
Với m, n, r, u, s 
Nhận xét rằng phương trình P(x) = 0 không thể có nghiệm dương va fnghieemj bằng 0
(do a, b, c, d ∈ {0;1;2...9} và a, d khác 0) nên m, n cùng dấu. Đồng nhất thức hệ số, ta thu
được mr = a, nr = d nên không mất tính tổng quát có thể coi m, n, r, s  * )
Vậy P  x   ax3  bx2  cx  d   mx  n  rx2  ux  s (1)
Thế x = 10 v|o (1) ta thu được: p = (10m +n) 100r  10u  s
Với 0 < 10m < n < 100 < p, trái với giả thuyết nên p là số nguyên tố

Đề 39
m  2
Câu 1. a) Phương trình đã cho có hai nghiệm x1.x2   '  m2  3m  2  0  
m  1
Với điều kiện trên, theo định lí Viets ta có: x1  x2  2m, x1.x2  3m  2
Do đó x12  x22   x1  x2   2 x1 x2  4m2  2  3m  2   4m2  6m  4
2

2
 3 7 7
x  x  4m  6m  4   2m     , m  D   ;1   2;  
2
1
2
2
2

 2 4 4
Đẳng thức xảy ra khi và chỉ khi
3 3
2m   0  m   D
2 2
7 3
Vậy biểu thức x12  x22 đạt giá trị nhỏ nhất bằng khi và chỉ khi m =
4 4
b) Do f  x   0 với mọi x  R nên f  0   0  c  0
a  0
 a  0

Mặt khác f  x   0 với mọi x  R     2
  b  4ac  0
 b  4ac

2

Theo bất đẳng thức Cosi ta có:


4a  c  2 4ac  2 b2  2 b  2b (điều phải chứng minh).

3x  0 x  0
 
Câu 2. a) Cách 1: Điều kiện:  x  2  0   x  2  x  2
2 x  3  0 
 x   3
 2

198
Page: Tài Liệu Môn Toán
Website: tailieumontoan.com

Với điều kiện trên, phương trình đã cho tương đương với:
x  2  2 x  3  3x  1
 x  2  2x  3  2  x  2  2 x  3  3x  1  2 3x

  x  2  2 x  3  3x
  x  2  2 x  3  3 x
 x  1
 2 x 2  x  6  3x  2 x 2  4 x  6  0  
x  3
Kết hợp với điều kiện ta được x = 3. Vậy tập nghiệm của phương trình S = {3}.
Cách 2: Đặt u  x  2; v  2 x  3; t  3x (u, v, t  0  x  2)
Ta có hệ phương trình
u  v  t  1

 2 2 2  u  v   u  v  1  1  u  v  2  2uv  2u  2v.
2 2 2 2 2

u  v  t  1

u  1
Vậy uv  u   v  1  0   u  1 v  1  0  
v  1
Với u  1  x  2  1  x  3
Với v  1  2 x  3  1  x  1 (loại)
Vậy phương trình chỉ có nghiệm duy nhất x = 3.
b) Điều kiện: x  6, y  3
Từ phương trình đầu của hệ ta có:
 x  y   x 2  xy  y 2  3  3  x 2  y 2   2
  x  y   x 2  xy  y 2   3  x  y   3x 2  3 y 2  2
 x3  y 3  3x  3 y  x 2  3 y 2  2
  x  1   y  1  x  1  y  1  y  x  2  x  1
3 3

Thay v|o phương trình thứ hai của hệ ta được:


x  6  x  1   x 2  2 x  8,  x  1
 x  6  3  x  1  2  x2  2x  3  0
x 3 x 3
    x  3 x  1  0
x6 3 x 1  2
 1 1 
  x  3    x  1  0
 x6 3 x 1  2 
 1 1 
 x  3  do   x 1  0
 x6 3 x 1  2 
Khi x  3  y  1
199
Page: Tài Liệu Môn Toán
Website: tailieumontoan.com

So sánh với điều kiện tập x{c định ta được nghiệm của hệ phương trình (x;y) = (3;1).
Câu 3. a) Yêu cầu bài toán
a 2 b2 c2
     a  b  c   3  a 2  b2  c2    a  b  c 
2

b c a
a2 b2 c2
  2a  b   2b  c   2c  a   a  b    b  c    c  a 
2 2 2

b c a
a  b b  c  c  a 
2 2 2

     a  b    b  c    c  a  (*)
2 2 2

b c a
Bất đẳng thức (*) luôn đúng do 0 < a, b, c < 1.
1
Dấu đẳng thữ xảy ra khi và chỉ khi a = b = c = . Vậy bất đẳng thức được chứng minh.
3
b) Tập x{c định x  2 . Đặt t  x  2, t  0 suy ra x  t 2  2, thay vào bất phương trình ta
được: 3 1  t 2  1  t  1  t 2  1  t   t 3  4t 2  4t  0  t t  1t  3  0
3

t  3  x2 3  x  11
  
0  t  1 0  x  2  1  2  x  3
Kết hợp với tập x{c định ta được tập nghiệm là S =  2;3  11;  
Câu 4. a) Ta có
2 AM  AB  AC ; EF  AF – AE
 2 AM .EF   AB  AC  AF – AE 
 AB. AF – AC. AE  do AB. AE  AC. AF  0 
 AB. AF cosBAF – AC. AE cos CAE  0
Do ABE ACF  AB. AF  AC.AE và
BAE  CAE  900  BAC
Vậy AM  EF  AM  EF
Cách 2: Dựng hình bình hành ACDB. Do ABE ACF nên
AB AE
ta có  (1)
AC AF
AB CD
Do ACDB là hình bình hành nên ta có  (2)
AC CA
Do CAB  EAF  1800 ; CAB  ACD  1800 nên ta có
ACD  EAF (3)
Từ (1), (2), (3), ta có DCA EAF
Vậy CAD  AFE
Do CAD  FAH  900
 AFE  FAH  900 nên AM  EF
200
Page: Tài Liệu Môn Toán
Website: tailieumontoan.com

AB AE
Cách 3: Theo giả thiết ABE ACF nên ta có   k (1)
AC AF
Ta có AM 
1
2

AB  AC (2) 
Xét phép quay vecto góc quay +900 , từ (1) ta có AB  k AE; AC  k FA
1
2
 
1

Vậy AM  AB  AC  k FA  AE  k FE (3)
2
1
2

1
Từ (2) và (3), ta có phép quay vecto góc quay +900, AM  k FE
2
1
Kết luận: qua phép quay vecto góc quay +900 AM là k FE
2
Vậy AM  FE
Cách 4: xét hệ trục vuông góc Mxy như hình vẽ.
Ta có M (0;0); C(m;n); B(-m;-n); A(0;a) (m > 0).
Vecto AC có tọa độ AC   m; n  a 
Vecto AB có tọa độ AB   m; n  a 
Phương trình đường thẳng AF nhận AC là vecto pháp
tuyến là m  x  0    n  a  y  a   0
  a  n  yF  a  
Vì F  AF  F   ; yF 
 m 
Phương trình đường thẳng AE nhận AB là vecto pháp tuyến là
m  x  0   n  a  y  a   0
   n  a  yE  a  
Vì E  AE  E   ; yE 
 m 
 AB  kAE
Do có tam gi{c ABE đồng dạng với tam giác ACF nên 
 AC  kAF
Suy ra:

 m2   n  a 2  k   a  n  yE  a     y  a 2
2

 m  k  yE  a 
E
 m 
    yE  yF
   a  n  yF  a  
2

 m  k  y F  a 
 m2   n  a   k     yF  a 
2 2

  m 

Vậy AM  EF
Cách 5: (Dành cho các em học sinh đã học qua số phức)
Gọi a, b, c, e, f, m lầ lượt là những số phước tọa vị là A, B, C, E,
F,M
201
Page: Tài Liệu Môn Toán
Website: tailieumontoan.com

AB AE
Ta có tam gi{c ABE đồng dạng với tam giác ACF nên  k
AC AF
AB biểu diễn số phức b – a.
AC biểu diễn số phức c – a.
AE biểu diễn số phức e – a.
AF biểu diễn số phức f – a.
Vì AC  AF nên c – a = - ki(f – a) (2)
Từ (1) và (2) ta có b + c – 2a = ki(e – f)  2(m – a) = ki(e – f) (k ∈ R)
Vì AM biểu diễn số phức m – a, FE biểu diễn số phức e – f, nên ta có AM  EF
2S
sin A 4S
b) Ta có tan A   2 bc2  2 2
cos A b  c  a 2
b  c  a2
2bc
4S 4S
Tương tự ta tính được tan B  2 , tan C  2 2 2
c  a b2 2
a b c
4S 4S 4S
Theo giả thiết tan A + tan B = 2tanC  2 2  2 2 2
b c a c  a b
2 2 2
a  b2  c 2
2 2

 a 4   b2  c2   b4   c 2  a 2   2 c 4   a 2  b2 
2

 a 4  b 4  c 4  2b 2c 2  b 4  c 4  a 4  2c 2 a 2  2c 4  2a 4  2b 4  4a 2b 2

 2c 4   a 2  b 2   c 2  a 2  b 2 
2

 2c 4   a 2  b 2   2c 4  a  b
2

Hay tam giác ABC cân.


c) Gọi B (a;1 – 2a) ∈ d.
3
Gọi N l| trung điểm AC suy ra BN  BG (1)
2
 11 2
Mà BN   xN  a; yN  2a  1 , BG    a; 2a  
3 3
Theo (1)
 3  11 
 xN  a  2  3  a   11  a
    xN 
  2
 y  2a  1  3  2a  2   y  a
 N    N
2 3
 11  a   3 a 
 N ; a  Ta có: IN   ; a  .BM   4  a; 2a  1 mà
 2   2 
IN / / BM  k  : IN  k BM

202
Page: Tài Liệu Môn Toán
Website: tailieumontoan.com

3  a a  1
  k 4  a 
 2  1  B 1; 1 , N  5;1
a  k  2a  1 k  3
 
Ac đi qua N(5;1) v| có vecto pháp tuyến n  IN  1;1 suy ra AC có phương trình x + y
– 6 = 0. Đường tròn ngoại tiếp tam giác ABC có tâm I(4;0), bán kính R = IB = 10 nên có
phương trình:  x  4   y 2  10
2

Suy ra tọa độ A, C là nghiệm của hệ phương trình:


y  6  x
x  y  6  0
 y  6  x
 
    x  3
 x  4   y  10  x  4   y  10
2 2
 x  7
2 2
 

Vậy A(3;3), B(1;-1), C(7;-1) hoặc A (7;-1), B (1;-1), C(3;3)
Đề 40
Câu 1. 1. Ta có: 2  x  6  3 x  5  x  3 (*)

Điều kiện x  5
Khi đó (*)  2  6  x   x  3  3 x  5  2  6  x   
x  3  3 x  5  48  8x

 6 x 0
 
(1)
 6  x x  3  3 x  5  46  x  
 x  3  3 x  5  4 (2)

Từ (1)  x  6 thỏa mãn điều kiện


Từ (2)
 29
 x
29  5 x  0  5 17  3 5
 3  x  3 x  5   29  5 x   2  x
 x  17 x  61  0  x  17  3 5 2

 2

17  3 5
Nghiệm của phương trình x  6, x 
2
2. C{c số a, b, c thỏa mãn điều kiện a  2b  5c  0 . Chứng minh phương trình
ax2  bx  c  0 (1) có nghiệm

Trường hợp 1: a  0 suy ra 2b  5c  0 phương trình (1) trở th|nh bx  c  0 (2)


+) Nếu b  0  c  0 : Phương trình (2) có nghiệm (vô định).
+) Nếu b  0 phương trình (2) có nghiệm (duy nhất).
a  5c
Trường hợp 2: a  0 . Ta có b   .
2
203
Page: Tài Liệu Môn Toán
Website: tailieumontoan.com

  b2  4ac  4  4b2  16ac   a  5c   16ac  a 2  6ac  25c 2   a  3c   16c 2  0


2 2

Vậy phương trình (1) luôn có nghiệm


 x 2  4 xy  x  2 y  0 (1)
Câu 2. Ta có: 
 x  8 x y  3x  4 y  0 (2)
4 2 2 2


 x  4 xy  x  2 y  0
2  x 2  2 y  4 xy  x

 4  2
 x  2 y   12 x y  3x
2
 x  8 x y  3x  4 y  0 

2 2 2 2 2


 x  2 y  4 xy  x
2

 x  2 y  4 xy  x
2

   2
 4 xy  x   12 x y  3x  x  4 y  1  3x  4 y  1
2 2 2 2 2
 


 x  2 y  4 xy  x
2

 2
 x  4 y  1 4 y  4   0

Trường hợp 1: x  0  y  0   x; y    0;0 l| nghiệm của hệ.

Trường hợp 2: y  1  x2  3x  2  0   x; y   1;1   x; y    2;1


1 1
Trường hợp 3: y   x 2   0 (loại)
4 2
Vậy hệ có 3 nghiệm  0;0 , 1;1 ,  2;1 .

Câu 3. Trong mặt phẳng với hệ tọa độ Oxy, cho điểm A 1;3 , B  5; 3 . X{c định tọa độ
điểm M trên đường thẳng d : x  2 y  1  0 sao cho 2MA  MB nhỏ nhất.

Gọi I  x0 ; y0  l| điểm thỏa mãn 2IA  IB  0

2 1  x0   x0  5
  x0  1
 2 IA  BI   
2  3  y0   y0  3  y0  1

Vậy I  1;1 . Ta có

 
2MA  MB  2 MI  IA  MI  IB

 3MI  2 IA  IB  3 MI  3MI

Như vậy 2MA  MB nhỏ nhất khi v| chỉ khi MI nhỏ nhất. Suy ra M l| hình chiếu của I
trên d.
 x  2t  1
Phương trình tham số của d  . Gọi tọa độ M  2t0  1; t0 
y  t
1
Suy ra IM   2t0 ; t0  1 . Ta có IM .ud  0  2.2t0  t0  1  0  t0  .
5
204
Page: Tài Liệu Môn Toán
Website: tailieumontoan.com

 3 1 
Vậy M  ;  .
 5 5

Câu 4. 1. Ta có:
b2  c 2  a 2 a 2  c2  b2 b2  a 2  c 2
cot A  ;cot B  ;cot C 
4s 4s 4s
1
Khi   . Ta có:
2
1
cot A  cot C  cot B
2
b2  c 2  a 2 a 2  b2  c 2 1 c 2  a 2  b 2
  
4s 4s 2 4s

 5b2  a2  c2

Ta có:
4 2 4  b2  c 2 a 2  4 4  a 2  b2 c 2 
AG 2  AA1     ; CG 2  CC12    
9 9 2 4 9 9 2 4

4 a 2  c 2  4  5b2  4b2  2
Suy ra AG 2  CG 2   b2      b  AA1  CC1 .
9 4  9 4 

Vậy góc giữa AA1 và CC1 bằng 90°.

2.
b2  c 2  a 2 a 2  b2  c 2 c 2  a 2  b2
cot A  cot C  2cot B   2  a 2  c 2  2b2
4s 4s 4s
a 2  c 2  b2 a 2  c 2 2ac 1
Ta có cos B     . Suy ra B  60 .
2ac 4ac 4ac 2
Dấu = xảy ra khi tam gi{c ABC đều.
1 1 1
Câu 5. Cho a, b, c là các số dương thỏa mãn 2
 2  2 1
a b c
Tìm gi{ trị lớn nhất của biểu thức:
1 1 1
P  
5a  2ab  2b
2 2
5b  2bc  2c
2 2
5c  2ca  2a 2
2

Ta có: 5a2  2ab  2b2   2a  b    a  b    2a  b 


2 2 2

1 1 1 2 1
Suy ra      (1)
5a  2ab  2b
2 2 2a  b 9  a b 

205
Page: Tài Liệu Môn Toán
Website: tailieumontoan.com

2 1 1 1 1 9 9
(vì       )
a b a a b a  a  b 2a  b
1 1 2 1
Tương tự     (2)
5b2  2bc  2c 2 9  b c 
1 1 1 2 1 
     (3)
5c  2ca  2a
2 2 2c  a 9  c a 

1 1 1 1
Cộng theo vế của (1), (2) v| (3) suy ra P     
3 a b c 
2 2
1 1 1 1 1 1 1 1 1 1 1 1 1 1
Mặt kh{c  2  2           1    3
a 2
b c 3 a b c  3 a b c  a b c

3
Suy ra P  . Dấu = xảy ra khi a  b  c  3 .
3
Đề 41
Câu 1. Ta có:
a5 a 2  a 3  2b3   2a 2b3 a 2b3
  a 2
 2
a 3  2b3 a 3  2b3 a 3  2b3
a 3  2b3  a 3  b3  b3  3 3 a 3 .b3 .c 3  a 3  2b3  3ab 2
a 2b3 a 2b3 a 2b3 ab
 2   2 
a  2b 3
3ab 2
a  2b 3
3
a 2b3 2 a5 2
 a2  2  a 2
 ab   a 2  ab
a  2b
2 3
3 a  2b
3 3
3
Chứng minh tương tự
b5 2 c5 2
 b 2
 bc,  c 2  ca
b  2c
3 3
3 c  2a
3 3
3
2 2 2
Từ đ}y ta có S  a 2  b2  c 2  ab  bc  ca
3 3 3
2 2 2 1 1
a 2  b 2  c 2  ab  bc  ca   a  b    b  c    c  a     ab  bc  ca 
2 2 2

3 3 3 2   3
1
 S   ab  bc  ca 
3
Áp dụng bất đẳng thức  x  y  x   3  xy  yz  zx  , ta có
2

 ab  bc  ca   3  ab  bc  ca  3
2

3 1
S Đẳng thức xảy ra khi và chỉ khi a = b = c 4
3 3

206
Page: Tài Liệu Môn Toán
Website: tailieumontoan.com

3  1 1 1 
Vậy min S = tại  a; b;c    4 ; 4 ; 4 
3  3 3 3
Câu 2. Giả sử rằng  a1 !  1 .....  an !  1  32  x 2 . Khi đó nếu ai = 2 thì không ảnh hưởng
đến tích nên có thể nên có thể xét ai ≥ 3 với k giá trị i. Nếu ai ≥ 4 thì ai !  1 là số hạng 4k –
1 nên phải có ước số nguyên tố p có dạng 4k – 1. Ta có ngay p | x2 + 32 nên p = 3. Tuy
nhiên aj ≥ 3 nên a j !  1 không thể chia hết cho 3 với mọi j, mâu thuẩn. Vậy ai = 3 với mọi
i. Thành thử ta đưa đến phương trình 5k = x2 + 9. Ta chỉ ra k là số chẵn. Nếu k lẻ thì 5k ≡ -
1 (mod 3) do đó x2 ≡ -1 (mod 3). Từ đó đưa về 5k + x2 = 9 hay  5k /2  x  5k /2  x   9 suy ra
5k /2  x  1,5k /2  x  9. Do đó 5k /2  5 . Vậy k = 2.
Thành thử ta có 2 trong số n số bằng 3 còn lại bằng 2.
Câu 3. Lời giải
Gọi G = BN ∩ CM; K = EM ∩ FN (h.1)
Dễ thấy G thuộc OH (đường thẳng Euler); K thuộc GH (định lí Pappus).
Do đó O, H, K thẳng hàng (1)
Dễ thấy B, C, E, F ( BEC  900  BFC ) và M,N, E, F cùng thuộc một đường tròn (đường
tròn Euler)
Do đó PH / (O1 )  HB.HE  HC.HF  PH / (O2 ) ; PK/ (O1 )  KB.KE  KN .KF  PK/ (O2 )
Điều đó có nghĩa l| HK  O1O2 (2)
Từ (1) và (2) suy ra OH  O1O2

Câu 4.Ta chứng minh k bén nhất là 11. Xét tập S0 gồm 10 số tự nhiên là {0,6,12,18,24}
∪{1,7,13,19,25,31}

207
Page: Tài Liệu Môn Toán
Website: tailieumontoan.com

Mỗi số ở nhóm 2 đều chia 6 dư 1, mỗi số ở nhóm 1 đều chia hết cho 6. Do đó không thể
chọn được 6 số từ S0 mà tổng chia hết cho 6. Ta chỉ ra một tập con S bất kì của tập các số
nguyên mà S = 11 thì đều chọn ra được 6 số có tổng chia hết cho 6. Thực vậy, đầu tiên
ta chứng minh trong 5 số bất kì bao giờ cũng chọn được 3 số có tổng chia hết cho 3.
Thực vậy, nếu 5 số này có 3 số chia hết cho 3 được 3 số dư kh{c nhau thì tổng của ba số
đó chia hết cho 3. Nếu 5 số này chia cho 3 có tối đa hai số dư khác nhau, thì theo
5
nguyên lý Dirichel, tồn tại ít nhất    3 số có cùng số dư khi chia cho 3.
2
Áp dụng kết quả trên, chnj 5 số bất kì, khi đó có ba số có tổng chia hết cho 3. Kí hiệu
nhóm 3 số đó l| S1 và loại bỏ ra khỏi tập S đang xét. Trong 11 – 3 = 8 số còn lại, lấy tiếp 5
số v| do đó chọn được 3 số có tổng chia hết cho 3. Nhóm 3 số này là S2. Loại tiếp 3 số
này ta còn lại 8 – 3 = 5 số. Áp dụng kết quả trên một lần nữa ta chọn được S3 gồm 3 số có
tổng chia hết cho 3.
Tổng số trong S1, S2, S3 là 3 số chia hết cho 3. Trong 3 số ấy có hai số cùng tích chẵn lẻ,
do đó tổng hai số đó phải chia hết cho 2 x 3 = 6. Thành thử ta có 6 số có tổng chia hết
cho 6.
Nhận xét: Đ}y l| trường hợp đặc biệt cảu định lý EGZ đã kh{ kinh điển: Cho số nguyên
dương n. khi đó trong một tập hợp S gồm 2n -1 số nguyên tùy ý, luôn chọn ra được n số
có tổng chia hết cho n. Lược đồ chứng minh định lý n|y như sau: Đầu tiên chứng minh
b|i to{n đúng cho số nguyên tố p. Sau đó chứng minh đúng cho pk v| cuối cùng chứng
minh thêm: nếu b|i to{n đúng cho n = a v| n = b với (a,b) = 1 thì b|i to{n đúng cho n =
ab. Hay không?

Ngày thi thứ hai


Câu 1. Điều kiện x, y  0
Cách 1.
x  y  2 xy  4, t  xy  x  y  4  2t
x  15  y  15  8  x  y  30  2 xy  15  x  y   225  64

 4  2t  30  2 t 2  15  4  2t   225  64  2 t 2  30t  285  2t  30

 t 2  30t  285  t  15  30t  285  30t  225  t  1  x  y  1


Cách 2:
Xét u   
x ; 15 , v   
y ; 15 ,

uv    
x  y ; 2 15  2; 2 15 
 
2
u  x  15, v  x  15, u  v  22  2 15 8

208
Page: Tài Liệu Môn Toán
Website: tailieumontoan.com

Câu 2. Gọi ba nghiệm là x1 , x2 , x3 khi đó xi  0 nên yi   xi  0 . Ta có


P( x)   x  y1  x  y2  x  y3 
Theo giả thiết a + b + 1 = 7c ta suy ra 1+ a + b+ c = 8c do đó P(1) = 8P(0). Theo định lý
Viet, 8 y1 y2 y3   x  y1  x  y2  x  y3   8 y1 y2 y3 . Suy ra y1 y2 y3  1 hay x1 x2 x3  1 .
Do đó c  1 hay c  1
Phương trình P(Q(x)) = 0 tương đương với Q(x) – xi = 0 với i = 1, 2, 3 n|o đó. Ta có
i  1  d  xi . Theo trên y1 y2 y3  1 nên ít nhất một số yi  1 . Khi đó xi  1 . Suy ra
i  d  0 . Vậy phương trình
Q(x) – xi = 0 vô nghiệm. Thành thử trong 3 phương trình Q  x   xi  0 với i = 1, 2, 3 có ít
nhất một phương trình vô nghiệm. Thành thử phương trình tối đa 4 nghiệm
Câu 3. Lời giải
Gọi G = AB ∩ CD (h.25)
1) Vì (GPAB) = -1; MA = MB và (GQCD) = -1; NC = ND nên
GM .GP  GAGB
.  GC.GD  GN .GQ
Do đó M, N. P, Q cùng thuộc một đường tròn, tâm T.
2) Gọi L l| trung điểm của È; S = MQ ∩ NP ; {U;V} = EF ∩ (O)
Dễ thấy TS  LG (định lí Brocard) (3)
Dễ thấy (GPAB) = -1.
Kết hợp với MA = MB, theo hệ thức Maclaurin, ta có
PG/  O  GA.GB  GM.GP  PG/ T 
Dễ thây (EFUV) = - 1 = (EFPQ).
Theo hệ thức Newton cho hai h|ng điểm điều hòa (EFUV) và (EFPQ), ta có
LE 2  LF 2  LP.LQ  LU .LV
Mà LP.LQ = PL /(T) ; LU.LV = PL /(O) Vậy PL /(O) = PL /(T)
Do đó OT  LG (4)
Từ (3) và (4) suy ra O, T, S thẳng h|ng (đpcm)

209
Page: Tài Liệu Môn Toán
Website: tailieumontoan.com

Câu 4.
a) Giả sử rằng d là một ước nguyên tố của a 2 +1. Suy ra ordd (a)  2t , vowsi
n

0  t  n  1 . Neeus t  n thì a 2 - 1⋮ a 2 -1⋮d , suy ra 2⋮ d, mâu thuẩn vì d lẻ. Vậy t =


n t

n + 1, suy ra d ≡ 1 (mod 2n1 ).


b) Đặt m  2n q  r với 0  r  2n . Nếu q là số chẵn thì từ a m  1  a r  a mr  1   a r  1
, ta suy ra a r  1 4 , mâu thuẩn do A > a r  1 . Thành thử q lẻ. Từ
a m  1  a r  a mr  1  1  a r chia hết cho A.
m
Vậy r = 0 nên  q là số lẻ.
2n

Đề 42
Câu 1. Hàm số f(x) x{c định khi và chỉ khi
1  x  3
 x 2  2 x  3  2  x  3
 2   x  2 
 x  2 x  x  0  1  x  0
 
Vậy tập x{c định cảu hàm số f(x) là S =  1;0    2;3
Câu 2. a) Với mọi x1.x2   1;   , x1  x2 ta có:

210
Page: Tài Liệu Môn Toán
Website: tailieumontoan.com

x1 x
 2
f  x1   f  x2  x1  1 x2  1
K 
x1  x2 x1  x2
x1  x2  1  x2  x1  1 x1  x2 1
   0
 x1  x2  x1  1 x2  1  x1  x2  x1  1 x2  1  x1  1 x2  1
(Do x1.x2   1;   )
Do đó K > 0  f(x) đồng biến trên khoảng  1;  
b) Tập x{c định hàm số alf D  2015; 2015 Với mọi x  D , ta có  x  D ,
f   x   2015  x  2015  x    
2015  x  2015  x   f  x 
Suy ra f(x) là hàm số lẻ
 x 2  x  6  0

Câu 3. Điều kiện: 2  x  0  3  x  2
3  x  0

Bất phương trình đã cho tương đương với
19  3x  4  2  x 3  x   6  2 x  2 3 x 
Đặt t  2  x  2 3  x , t  0 ta có:
t 2  2  x  4 3  x   4  2  x 3  x   14  3x  4  2  x 3  x 
t  1
Thay vào phương trình trên ta được: 5  t 2  6t  t 2  6t  5  0  
t  5
+) t  1  2  x  2 3  x  1  2  x  4  3  x   4  2  x 3  x   1
 3x  13  4  x2  x  6  0 vô nghiệm do 3  x  2
+) t  5  2  x  2 3  x  5  2  x  4  3  x   4  2  x 3  x   25
16   x  x  6   11  3x 
 2 2

 4  x  x  6  11  3x  
2

11  3x  0

25 x 2  50 x  25  0

  11  x  1 thỏa mãn điều kiện
x 
 3
Vậy tập nghiệm của phương trình đã cho l| S = {1}

 x  2 y  3xy  y  1  0
2 2
(1)
Câu 4. Ta có:  2 (I)

 x  y 2
 y  3  0 (2)
 x  y 1
Ta có (1)   x  y  1 x  2 y  1  0  
x  2 y 1

211
Page: Tài Liệu Môn Toán
Website: tailieumontoan.com

y  2
Với x  y  1 thay v|o (2) ta được 2 y  3 y  2  0  
2
y   1
 2
+) y  2  x  1
1 3
+) y    x  
2 2
 y  1
Với x  2 y  1thay v|o (2) ta được 5 y  3 y  2  0  
2
y  2
 5
+) y  1  x  1
2 9
+) y   x 
5 5
 3 1 9 2
Vậy (I) có nghiệm (x;y) là: 1; 2  ,  1; 1 ,   ;   ,  ; 
 2 2 5 5
Câu 5. Bất phương trình đã cho vô nghiệm khi và chỉ khi
 m 1 x2  2  m  2 x  2m  2  0 x 
2
Trường hợp 1: Nếu m = 1 thì 6x + 4 < 0, x   x   , x  vô lí.
3
Trường hợp 2: Nếu m ≠ 1 thì  m  1 x  2  m  2  x  2m  2  0 x 
2

m  1

 m  1  0 
 m  1 

  2    m  2  10  m  2  10
 '   m  2    m  1 2m  2   0   m  4m  6  0 
2
 
  m  2  10


Vậy tập hợp các giá trị cua rm là S = ; 2  10 
Câu 6. Cách 1:
Kết quả cơ bản: cho tam giác ABC trọng tâm G.
Khi đó với mọi điểm O ta có OA  OB  OC  3.OG
Do M, N, P lần lượt là trọng tâm các tam giác OBC, OCA, OAB nên:
OB  OC  3.OM , OC  OA  3.ON , OA  OB  3.OP
  
Cộng từng vế 3 hệ thức trên ta được: 2 OA  OB  OC  3 OM  ON  OP 
 2.3.OG  3.3.OG '  2.OG  3.OG '  O, G,G' thẳng hàng
Cách 2: Vì M, N, P lần lượt là trọng tâm của tam giác OBC, OCA, OAB nên ta có
OM ON OP 2
  
OA ' OB' OC' 3
( A ', B ', C ' lần kuowtj l| trung điểm của BC, CA, AB)

212
Page: Tài Liệu Môn Toán
Website: tailieumontoan.com

3
Xét vị tự V : MNP  A ' B ' C ' . Vậy trọng t}m G’ của tam
O
2

giác MNP biến thành trọng tâm G của tam giác A ', B ', C ' .
Mà trong tâm G của tam giác A ', B ', C ' chính là trọng tâm G
của tam giác ABC
Vậy O, G, G' thẳng hàng
Câu 7. Theo định lí hàm số sin và côsin ta có:
a
sin A abc
tan A   2 2R 
cos A b  c  a R b  c2  a2 
2 2 2

2bc
abc abc
Tương tự ta có tan B  , tan C 
R c  a  b 
2 2 2
R  a  b2  c 2 
2

abc abc abc


 tan A  tan C  2.tan B    2.
R b  c  a  R  a  b  c 
2 2 2 2 2 2
R  a  c2  b2 
2

1 1 1
  2  2. 2 2 2
b c a
2 2 2
a b c
2 2
a c b
  c 2  a 2  b 2  a 2  b 2  c 2    b 2  c 2  a 2  a 2  c 2  b 2 
2 2

 2  b 2  c 2  a 2  a 2  b 2  c 2   a 4   b 2  c 2   c 4   a 2  b 2   2 b 4   a 2  c 2 
2

 a 2  a 2  b 2  2c 2    c 2  b 2  c 2  2b 2   0  b  c (do a 2  b 2  2 c 2 )
Kết hợp với a 2  b2  2c2  a  b  c
Vậy tam gi{c ABC đều
Câu 8.Nhận xét.Các tứ giác BHCM, AHCN là các hình bình hành suy ra nếu gọi E, F lần
lượt l| trung điểm của BC, CA thì E, F cũng tương tựng l| trung điểm của HM, HN.
7 5 3 5
Do đó E  ;  , F  ; 
2 2 2 2
7 5
Đường thẳng BC đi qua điểm P(4;2), E  ;  nên:
2 2
x4 y2
BC :   x y6  0
7 5
4 2
2 2
AH vuông góc với BC suy ra AH co vecto pháp
tuyến n AH  1; 1 kết hợp với AH đi qua điểm H
(2;2) suy ra: AH: 1(x – 2) – 1(y – 2) = 0  x – y = 0
A  AH  A  a; a  ,C  BC  C  b;6  b  ,
Do F l| trung điểm AC nên:

213
Page: Tài Liệu Môn Toán
Website: tailieumontoan.com

 xA  xC
 xF  2 a  b  3 a  1
    A 1;1 , C  2; 4 
 y  y A  yC a  6  b  5 b  2
 F 2
Do E l| trung điểm BC nên:
 xB  xC
 xE  2  xB  2 xE  xC  xB  5
    B  5;1
y  B y  yC  B
y  2 y E  yC  B
y  1
 E 2
Vậy A 1;1 , B  5;1 , C  2; 4
Câu 9. Thay 2015 = a + b+ c thì bất đẳng thức cần chứng minh có dạng:
a b  c  b  c  a  c  a  b   bc ca ab 
   6  2 2    
bc ca ab  a b c 
a b  c  b  c  a  c  a  b  a a b b c c
Ta có   6      6
bc ca ab b c a c a b
bc ca ab bc ca ab
 2 2 22 .2  2 .2  2 .2
a b c a b c
 bc ca ab 
 2 2    
 a b c 
2015
Dấu bằng xảy ra khi và chỉ khi a = b = c =
3
Đề 43
Câu 1 (4,0 điểm) Giải phương trình sau trên tập số thực:

1  x2  x2  x  1  3 1  x  1 .
LG:

1  5
Điều kiện x{c định:  x  1 (1).
2
u, v, t  0

Đặt u  1  x , v  x  x  1, t  1  x ta được u  v  t  1
2 2 3
(2).
u 2  v 2  t 3  1

Từ (2) suy ra 0  u, v, t  1  1  u 2  v 2  t 3  u  v  t  1 . Do đó

214
Page: Tài Liệu Môn Toán
Website: tailieumontoan.com

 u  1
u, v, t  0 

u  v  t  1  v  t  0
  v  1
 2
(2)  u  u   .
v 2  v  u  t  0
 
t  t  t  1
 u  v  0
3

Thay lại biến x ta được tập nghiệm của phương trình l| S  {1}.
Câu 2 (4,0 điểm) Cho tam giác ABC . Gọi (O1 ) l| đường tròn đi qua B và tiếp xúc với
AC tại A ; (O2 ) l| đường tròn đi qua C và tiếp xúc với AB tại A . P l| giao điểm thứ
hai của (O1 ) và (O2 ) ; K , L theo thứ tự l| giao điểm thứ hai của (O1 ), (O2 ) với đoạn
thẳng BC . Gọi ( S ) l| đường tròn ngoại tiếp tam giác PKL .
a) Chứng minh rằng: AK , AL tiếp xúc với ( S ) .
b) Gọi Q l| giao điểm thứ hai của ( S ) và AP ; E l| giao điểm của QK và AB ; F
l| giao điểm của QL và AC . Chứng minh rằng c{c điểm A, K , L, S , E, F cùng thuộc
một đường tròn.
LG:
a) Tứ giác ABKP là tứ giác nội tiếp nên ABP  AKP .
AC là tiếp tuyến của (O1 ) nên ABP  PAC . Suy ra AKP  PAC (1).

Tứ giác APLC là tứ giác nội tiếp nên PAC  PLK (2).


Từ (1) và (2), suy ra AK là tiếp tuyến của đường tròn ( S ) .
Tương tự, ta chứng minh AL l| cũng l| tiếp tuyến của đường tròn ( S ) .

215
Page: Tài Liệu Môn Toán
Website: tailieumontoan.com

O2
O1 F

P
E

B K L C
S

b) Cách 1. Dễ thấy AKSL là tứ giác nội tiếp. Ta chứng minh tứ giác AEKL là tứ giác nội
tiếp. Thật vậy, Ta có BEQ  EAQ  EQA (3).
Tứ giác KPLQ là tứ giác nội tiếp nên KQP  PLK (4).
AB là tiếp tuyến với (O2 ) nên EAQ  PLA (5).

Từ (3), (4) và (5) nên BEQ  ALK (đpcm).


Cách 2. Ta có KLQ  KPQ và KPQ  ABK nên ABK  KLQ , suy ra QL AB .
Do đó BEK  KQL . Mà KQL  ALK (do AL là tiếp tuyến với (S)) nên

BEK  ALK .
Câu 3 (4,0 điểm) Cho đa thức f ( x)  x4  x3  mx2  nx  p , trong đó m, n, p là các số
nguyên đôi một phân biệt, khác không, sao cho f (m)  m4  m3 và f (n)  n4  n3 . Tìm
m, n, p .

LG: Xét đa thức g ( x)  f ( x)  x 4  x3  mx 2  nx  p . Theo giả thiết g (m)  g (n)  0 . Do


g ( x) l| đa thức bậc 2 nên g ( x)  a( x  m)( x  n) .

Từ đó ta có: mx2  nx  p  a( x  m)( x  n).


Đồng nhất các hệ số cho ta p  amn , n  a(m  n) và m  a .

Từ đó ta được n  m(m  n) hay (m  1)n  m2 . Từ đ}y ta được m  1∣ 1 hay m  1  1 .


suy ra m  2 . Từ đó n  4 và p  16 .
Vậy m  2, n  4, p  16 .

216
Page: Tài Liệu Môn Toán
Website: tailieumontoan.com

Chú ý. Học sinh có thể thay trực tiếp m, n rồi giải hệ phương trình nghiệm nguyên để
tìm m, n, p.
Câu 4 (4 điểm) Tìm tất cả các cặp số nguyên dương (a, b) thỏa mãn đồng thời hai điều kiện
sau:
i) a  b2 l| lũy thừa của một số nguyên tố;
ii) a 2  b chia hết cho a  b2 .
a2  b b4  b
LG: Đặt a  b  p , p nguyên tố và m nguyên dương. Ta viết
2 m
 a b 
2
,
a  b2 a  b2
suy ra p m ∣ (b4  b)  b(b3  1).

Từ (b, b3  1)  1, và b  1  b  a  b2  p m nên ta suy ra p m ∣ b3  1 .

Ta có b3  1  (b  1)(b2  b  1) và (b  1, b2  b  1)∣ 3.

+ Nếu (b  1, b2  b  1)  1 thì p m ∣ b  1 hoặc p m ∣ b2  b  1. Từ p m  b2  a  b2  b  1 nên ta


chỉ có p m | b  1 và suy ta p m  a  b2  b  1 . Do đó a  b  1.

+ Nếu (b  1, b2  b  1)  3 suy ra p  3.
Xét m  1, không có (a, b) .
Xét m  2, (a, b)  (5, 2).

Xét m  3, khi đó 3∣ b  1 hoặc 3∣ b2  b  1 và 3m1 l| ước của phần tử còn lại.

Từ b  1  b2  a  1  3m1 , vì vậy 3m1 ∣ b2  b  1. Do đó b2  b  1  0 (mod 9), mâu thuẫn.


Vậy (a, b) {(1,1);(5, 2)}.
Câu 5 (4 điểm) Cho tập S  {1, 2,3,..., 2025} . Tìm số nguyên dương nhỏ nhất n sao cho:
Với mọi tập con T của S gồm n phần tử, tồn tại hai phần tử phân biệt u, v  T sao cho
u  v  20.

LG: Giả sử n là số nguyên dương nhỏ nhất thỏa mãn đề. Xét tập
T  {1, 2,...,10} {20,21,..., 2025} .
Ta thấy, với mọi u, v T phân biệt thì:
Nếu u, v {20, 21,..., 2025} thì u  v  41  20. Vậy không có u, v thỏa mãn u  v  20.
Nếu u, v {1, 2,3,...,10} thì u  v  19  20. Vậy không có u, v thỏa mãn u  v  20.
Nếu u {1, 2,3,...,10}, v {20, 21,..., 2025} thì u  v  21  20. Vậy không có u, v thỏa mãn
u  v  20. Vì | T | 2016 nên n  2017.

217
Page: Tài Liệu Môn Toán
Website: tailieumontoan.com

Mặt khác, với mọi tập T  S ,| T | 2017 , xét 9 cặp số sau (1;19),(2;18),...,(9;11) .
Nếu một trong các cặp trên thuộc T thì đó l| cặp (u; v) thỏa mãn u  v  20.
Nếu không có cặp nào thuộc T thì | T | 2025  9  2016 , vô lí.
Vậy với mọi tập T  S ,| T | 2017 luôn tồn tại u, v T thỏa mãn u  v  20 .

Kết luận: Giá trị nhỏ nhất của n là 2017.


Đề 44
Câu 1: (1,5 điểm)
Yêu cầu bài toán  PT sau có hai nghiệm phân biệt
x2  3x  2   x  m hay x2  2 x  2  m  0 (*)có  '  0  m>1
xA  xB
Gọi x A ; x B là 2 nghiệm của (*), I l| trung điểm AB ta có x I   1;
2
yI  x I  m  m  1

Yêu cầu bài toán  yI  x I

 m  1  1  m  2;m  0

Kết hợp ĐK, kết luận m  2


Câu 2 (2,0 điểm)

a) Giải phương trình sau trên : 3x  1  x  1  9  x .


9
b) Giải bất phương trình sau:  x2 .
x 5 3

LG: Điều kiện: 1  x  9 . Ta có 3x  1  x  1  9  x


 3x  1  x  1  9  x
 3x  1  8  2 x  1 9  x
 7
x 
 3
9 x  42 x  49  4( x  1)(9  x)
2

218
Page: Tài Liệu Môn Toán
Website: tailieumontoan.com

 7
x 
 3
13x  82 x  85  0
2

 x5
x  2
b) Điều kiện: x  5  3  0   .
 x  8
9 9
TH1 : Xét x  2 ta có : 1  2 x 2 x
5 x3 2 x
  2  x   9  3  x  2  3
2

 1  x  5 Vậy 1  x  2 là nghiệm.
9 9
TH2 : Xét 2  x  5 ta có : 1   x2  x2
5 x3 2 x
   x  2   9 ( Bpt vô nghiệm)
2

9 9
TH3 : Xét 5  x  8 ta có : 1   x2   x  2  0
x 8 x 8
9   x  8 x  2   x 2  10 x  7
 0 0
x 8 x 8

  x  8  x 2  10 x  7   0

x  5  3 2

8  x  5  3 2

Kết hợp với miền x đang xét ta có 8  x  5  3 2 là nghiệm của Bpt.


Vậy tập nghiệm của Bpt là : S   1;2   8;5  3 2 

2 x  y  3xy  3x  2 y  1  0
 2 2

Câu 3 (1,0 điểm) Giải hệ phương trình 


4 x  y  x  4  2 x  y  x  4 y

2 2

LG:

219
Page: Tài Liệu Môn Toán
Website: tailieumontoan.com

2 x  y  3xy  3x  2 y  1  0(1)
 2 2

 2
4 x  y  x  4  2 x  y  x  4 y (2)

2

Điều kiện: 2x+y  0, x+4y  0. Từ (1) ta được y=x+1 hoặc y=2x+1


*) Với y=x+1, thay v|o (2) ta được 3x2  x  3  3x  1  5x  4

 3( x 2  x)  ( x  1  3x  1)  (x  2  5 x  4)  0
1 1
 ( x 2  x)(3   )0
x  1  3x  1 x  2  5x  4
x  0
 x 2  x  0  
x  1
 ( x; y )  {(0;1);(1; 2)}

*) Với y=2x+1, thay v|o (2) ta được

3  3x  4 x  1  9 x  4
 3x  ( 4 x  1  1)  ( 9 x  4  2)  0
4 9
 x(3   )0
4x 1 1 9x  4  2
 x  0
Khi đó ta được nghiệm (x;y) là (0;1)
Đối chiếu điều kiện bài toán ta được nghiệm (x;y) của hệ đã cho l| (0;1) v| (1;2)
Câu 4 (2,5 điểm)
3 y 1
a) D(B;  )= ; C(0:y0) ; D(C;  )= 0 , theo bài ra ta có
5 5
y0  1 9
  y0  10; y0  8
5 5
C khác phía B đối với  suy ra C(0;-8)
Gọi B’(a;b) l| điểm đối xứng với B qua  thì B’nằm trên AC.
Do BB'  u   (1; 2) nên ta có: a  2b  3  0 ;

Trung điểm I của BB’ phải thuộc  nên có: 2a  b  2  0


Từ đó ta có: a= -7/5; b=4/5
3
Theo định lý Ta - Let suy ra CA  CB'
2

220
Page: Tài Liệu Môn Toán
Website: tailieumontoan.com

 7 44 
A(x; y);CA   x; y  8 ;CB'    ; 
 5 5 
21 26
Từ đó suy ra A( ; ) ;C(0;-8)
10 5
b) A

Kẻ đường cao AH, ta có b  a.CH;c  a.BH nên


2 2

b2 .BH  c2 .CH . Do đó:


B H C

b2 .BH  c2 .CH  0

Suy ra b2 .IB  c2 .IC  b2 .IH  c2 .IH  a 2 .IH


Kết hợp giả thiết suy ra 2a 2 .IA  a 2 .IH hay 2.IA  IH
Do đó điểm I thỏa mãn gt là I thỏa mãn A l| trung điểm IH

Với x, y, z tùy ý thỏa mãn: x.IA  y.IB  z.IC  0 (*) bình phương vô hướng 2 vế (*), chú ý
rằng 2IA.IB  IA2  IB2  AB2 ta có:
(x.IA2  y.IB2  z.IC2 )(x  y  z)  xyc2  xzb2  yza 2

Từ đó có (2a 2 .IA2  b2 .IB2  c2 .IC2 )  3b2c2

Mặt khác xMA2  x(IA  IM)2  x(IM2  IA2  2IA.IM)

Tương tự cho yMB2; zMC2 rồi cộng c{c đẳng thức đó lại ta có
xMA2  yMB2  zMC2  (x  y  z)IM2  xIA2  yIB2  zIC2
Thay số có:
2a 2 MA2  b2 MB2  c2 MC2  a 2 IM2  3b2c2  3b2c2
Dấu bằng xảy ra khi M trùng I

Câu 5 (2,0 điểm)


a) Chứng minh rằng c{c biểu thức sau không phụ thuộc v|o a.

E  sin 4 a  4cos2 a  cos4 a  4sin 2 a

221
Page: Tài Liệu Môn Toán
Website: tailieumontoan.com

Ta có E  sin 4 a  4(1  sin 2 a)  cos4 a  4(1  cos2a)

 E  (sin 2 a  2)2  (cos 2 a  2)2


 E  (2  sin 2 a)  (2  cos 2 a)  3
b)
A

C
K
F
B

Đặt AB=c, AC=b, BC=a, KAC   . Khi đó: KAB  2 ; BAC  3 .

Áp dụng định lí sin cho tam giác ABK v| ACK, ta được:


BK AK CK AK
 ; 
sin 2 sin B sin  sin C
sin B
Do BK=2CK, nên từ c{c đẳng thức trên ta có: cos  (*)
sin C
Lại có:
 b2  c2 a 2  a 2 b2  c 2  a 2
FA  FC  
2 2
    bc.cos A  bc cos 3 (1)
 2 4  4 2
LC 2  LA2  b 2  2b.LA.cos  LA2  b 2  2bc cos 2 .cos
 LA2  LC 2  2bc cos  .cos 2  b 2  bc  cos  cos3   b 2
  bc cos   b 2   bc cos 3 (**)

Thay (*) v|o (**), ta được: LA2  LC 2  bc cos3 (2)

Từ (1) và (2) suy ra: FA2  FC 2  LA2  LC 2


 2FL.CA  0  FL  CA.
Câu 6 (1,0 điểm)
Cho x, y, z là các số thực dương thỏa mãn x  y  z  xyz . Chứng minh rằng:
1  1  x2 1  1  y 2 1  1  z 2
   xyz (I)
x y z

222
Page: Tài Liệu Môn Toán
Website: tailieumontoan.com

1 1 1
Giả thiết suy ra:    1 . Ta Có:
xy yz zx
1 x2 1 1 1 1  1 1  1 1  1  2 1 1 
 2              ;"  "  y  z
x x xy yz zx  x y  x z  2  x y z 
Viết hai BĐT tương tự rồi cộng lại ta được:

1  1  x2 1  1  y 2 1  1  z 2  1 1 1
   3     ;"  "  x  y  z
x y z x y z
 1 1 1
Ta sẽ CM: 3      xyz  3  xy  yz  zx    xyz    x  y  z 
2 2

x y z
  x  y    y  z    z  x   0 Điều n|y luông đúng
2 2 2

Dấu bằng có khi và chỉ khi x=y=z


Vậy (I) được CM, dấu bằng có khi và chỉ khi x=y=z= 3
Lưu ý: Học sinh làm theo cách khác đúng vẫn cho điểm tối đa.

Đề 45

 y  2 xy  7 y   x  7 x  8
4 2 2 2
(1)
Câu 1. Xét hệ phương trình 
 3  x  y  1  x  x  4 y  3 (2)

2 3 2 2

Điều kiện x{c định: x ≤ 3


 y2  x 1
Ta có phương trình (1)   y  x   7  y  x   8  0   2
2 2 2

 y  x  8
Vì x ≤ 3 nên x – 8 < 0, do đó không thể xảy ra trường hợp y 2  x  8 .
Vậy y 2  x  1
Thay vào (2) ta có: 3  x  x  2  x3  x 2  4 x  1 (điều kiện x ≥ - 2).

223
Page: Tài Liệu Môn Toán
Website: tailieumontoan.com

 x3  x 2  4 x  4  2  x  2  1  3  x  0
x2 x2
  x _ 2  x  2  x  1   0
x  2  2 1 3  x
 1 1 
  x  2   x  2  x  1   0
 x  2  2 1 3  x 
 1 1 1 1
  x  2   x  2  x  1     0
 3 x  2  2 1 3  x 3
 
x 1 x 1
  x  2   x  2  x  1   0


3  x2 2  x  2 1 3 3  x 1  
3 x  2 

 
1 1
  x  2  x  1  x  2    0

 
3 x2 2  x  2 1 3 3  x 1  
3 x  2 

  x  2  x  1  0 (Điều kiện x  2)
Từ đó ta thu được nghiệm của hệ đã cho l|  1;0  2; 3 2;  3   
Câu 2.a) Vì ACQP và PDQB là các tứ giác nội tiếp nên ta có:
XAQ  CAQ  CPQ  DPQ  DBQ  XBQ nên AXQB nội tiếp (1)
Vì AXQB và BPDQ là các tứ giác nội tiếp nên ta có:
QXC  ABQ  PBQ  CDQ nên tứ giác XDQC nội tiếp (2)
Từu 1 và (2) suy ra QX là trục thẳng phương của hai đường tròng (ABQ) avf (CDQ) do
đó IJ  XQ

b) Ta sẽ chứn gminh rằng đường thẳng YZ đi qua điểm Q cố định v| đường thẳng này
cũng đi qua điểm X.
Vì XDQC nội tiếp nên DQX  DCX  PCA (3)
Từ PZ || AC nên PCA  CPZ  DPZ (4)
Từ (3) và (4) suy ra DQX  DPZ
Mặt khác PDQZ nội tieeso nên DPZ  DQZ  1800 , do đó DQX  DPZ  1800 hay Z, Q, X
thẳng hàng. Chứng minh tương tự ta được Y, Q, X thẳng hàng.
Từ đó suy ra điều phải chứng minh
Câu 3. Từ giả thiết ta có y3  x2  0 (modulo p)
Suy ra  y  x   y 2  yx  x 2   0 (modulo p) (1)
Ta có y – xx là số nguyên dương bé hơn p v| p l| số nguyên tố nên y – x và p là nguyên
tố cùng nhau. Do đó (1) ta được x2  y 2  z 2  0 (modulo p) (2)
Chứng minh tương tự ta cũng có: y 2  yz  z 2  0 (modulo p) (3)

224
Page: Tài Liệu Môn Toán
Website: tailieumontoan.com

Và z 2  zx  x 2  0 (modulo p) (4)
Từ (2) và (3) ta có: z 2  x2  yx  xy  0 (modulo p)
Suy ra  z  x  x  y  z   0 (modulo p)
Do đó x + y + z chia hết cho p, mà 0 < x + y + x< 3p
 x + y + z bằng p hoặc 2p (5)
Sử dụng (2) ta có (x + y)  xy (modulo p) , kết hợp với x + y   z (modulo p) ta được
2

z 2  xy (modulo p) , thay trở lại (2) ta có x2  y 2  z 2  0(modulo p) (6)


Nếu x + y + z = p thì (6) có ngay x 2  y 2  z 2 chia hết cho x + y + z
Nếu x + y + z = 2p thì (6) có ngay x 2  y 2  z 2 chia hết cho x + y + z
Câu 4.Theo bất đẳng thức giữa trung bình cộng và trung bình nhân ta có

 x y y z   1 1 1
z  x  8 xyz  P  8 xyz   
x y z
Cũng theo bất đẳng thức giữa trung bình cộng v| trung bình nh}n ta được
4
 1   1   1 
4 4
1 1 1 1
8 xyz     1313 8 xyz .          1313 21
 4x   4 y   4z  2 xyz .  xyz 
3
x y z

 x yz
3
1
Và xyz     6 . Suy ra P ≥ 13
 3  2
1
Mà khí x = y = z = thì P = 13, suy ra giá trị của P là 13
4
Câu 5. Ta sẽ giải thích bài toán tổng quát:
Bài toán. Cho m là số gnuyeen dương lớn hơn 1. Có 2m học sinh tham gia một buổi giao
lưu. Biết rằng cứ 3 học sinh bất kỳ, đều có ít nhất một cặp đôi gồm hai học sinh có trao
đổi kinh nghiệm học tập với nhau. Kí hiệu k là số cặp đôi như thế.
Tìm giá trị nhỏ nhất của k.
Lời giải Với mỗi số nguyên dương m > 1, rõ r|ng tồn tại giá trị nhỏ nhất của k, ta kí hiệu
giá trị này bởi k(m)
Ta thấy k(2) = 2
Bây giờ giả sử m > 2
Xét buổi giao lưu gồm 2m học sinh sao cho cớ 3 học sinh bất kỳ, đều có ít nhất một cặp
đôi gồm hai học sinh có trao đổi học tập với nhau bằng k(m).
Tồn tại ít nhất hai học sinh (kí hiệu l| A v| B) không trao đổi học tập với nhau, loại A và
B ra khỏi buổi giao lưu n|y ta có một buổi giao lưu gồm 2(m – 1) học sinh mà cứ 3 học
sinh bất kỳ, đều có ít nhất một cặp đôi gồm hai học sinh có trao đổi kinh nghiệm học
tập với nhau. Số cặp dôi gồm hai học sinh có trao đổi kinh nghiệm học tập với nhau
trong buổi liên hoan mới sẽ không ít hơn k(m – 1), mà mỗi học sinh trong buổi liên hoan
mới sẽ trao đổi kinh nghiệm học tập với A hoặc B (vì A v| không trao đổi học tập với
nhau)
225
Page: Tài Liệu Môn Toán
Website: tailieumontoan.com

Suy ra k(m)≥k(m – 1) + 2(m – 1)


Do đó k(m) ≥ m(m – 1) với mỗi số nguyên dương m > 1 (1)
Với mỗi số nguyên dương m > , ta xét mội buổi giao lưu gồm 2m học sinh như sau:
Các học sinh trong buổi giao lưu thuộc một trong hai nhốm (gọi là X và Y). Nhóm X
gồm m học sinh có trao đổi học tập từng đôi một, nhóm Y gồm m học sinh có trao đổi
học tập từng đôi một. Mỗi học sinh của nhóm n|y đều không có trao đổi học tập với bất
kỳ một học sinh nào của nhóm kia.
Rõ ràng trong buổi giao lưu n|y, cứ 3 học sinh bất kỳ, đều có ít nhất một cặp đôi gồm
hai học sinh có trao đổi kinh nghiệm học tập với nhau số cặp đôi trao đổi học tập với
nhau bằng m(m – 1).
Suy ra k(m) ≤ m(m – 1) với mỗi số nguyên dương m > 1 (2)
Từ (1) và (2) suy ra k(m) = m(m – 1) với mỗi số nguyên dương m > 1.
Trở lại bài toán ban đầu.
Theo trên ta có giá trị k bé nhất là k(21) = 420

Đề 46

Câu 1. x 3  x 2  3x  1  x3  6 x  2  5 (1)
 ( x3  x 2  1  2)  ( x3  6 x  2  3)  0
x3  x 2  3x  5 x3  6 x  7
 0
x3  x 2  3x  1  2 x3  6 x  2  3
 x  1  x 2  2 x  5  x  1  x 2  x  7 
 0
x3  x 2  3x  1  2 x3  6 x  2  3

  x  1 
 x 2  2 x  5  x2  x  7  
  0  x 1
 x3  x 2  3 x  1  2 x3  6 x  2  3 
Thử lại x = 1 thỏa mãn (1). Vậy phương trình có nghiệm x = 1.
Câu 2. a) Dễ thấy: BHC  1800  A
Từ đó suy ra MKC  1800  BKC  1800  BHC  A
Do đó NKM  1800  A suy ra tứ giác ANKM nội tiếp

226
Page: Tài Liệu Môn Toán
Website: tailieumontoan.com

b) Ta có BTC  1800  BHC  BAC nên T đối xứng với A qua BC.
Do đó PKC  TBC  ABC  B , suy ra tứ giác PKB nội tiếp.
Tương tự ta có tứ giác PKMC nội tiếp.
Do đó PMC  PKC  B  PBN ; MKC  NKB  NPB  PBN PMC
Vì X, Y l| hai trung điểm tương tự của BN, CM nên XPB  MPY , từ đó suy ra
XPB  BPM  1800  MPY  1800  MKC  1800  A không đổi.
Câu 3. Ta có x4  1  2 x2  x4  1  2 xy  2 x  x  y  . Do đó
x 1

x  1  2 xy 2  x  y 
4

y 1 z 1
Tương tự  ; 4 
y  1  2 yz 2  y  z  z  1  2 zx 2  z  x 
4

x y z 1 1 1 1 
Do đó  4  4      (*)
x  1  2 xy y  1  2 yz z  1  2 zx 2  x  y y  z z  x 
4

1 1 4
Áp dụng bất đẳng thức quen thuộc  
x y x y
1 1 1 11   1 1 1 3
           (**)
x y y z z x 4 x y y z z x 2
Từ (*) v| (**) suy ra điều phải chứng minh.
Câu 4.Với tam thức f(x) = ax2  bx  c , kí hiệu biệt thức của f(x) alf  F  b2  4ac. Với
phép biến đổi (i). ax2  bx  c biến đổi thành cx2  bx  a , suy ra chúng có cùng biệt thứ
  b2  4ac.
Với phép biến đổi (ii), gọi x1, x2 là nghiệm của f1(x) = ax2  bx  c suy ra x1 + t; x2 + t là
nghiệm của
f2(x) = a(x + t)2 + b(x + t) + c.
b c
Vì x1 + x2 =  ; x1x2 = nên
a a

227
Page: Tài Liệu Môn Toán
Website: tailieumontoan.com

 b  2 c
 f1  b2  4ac  a 2    4   a 2   x1  t    x2  t     f2
2

 a  a 
Tức là phép biến đổi n|y không l|m thay đổi biệt thức của tam thức. Do đó, c{c phép
biến đổi trên không l|m thay đổi biệt thứ ∆ của tam thứ (*)
Mặt khác, các tam thức x2  8x  2015 , 2016 x2  8x 1 có biệt thức ∆ l| 8124; 8128.
Do đó, từ (*) suy ra yêu cầu bài toán là không thể thực hiện được.
Câu 5. +) Với n = 0, ta có n7  n5  2n4  n3  n2  1  1 (không thỏa mãn)
+) Với n = 1, ta có n7  n5  2n4  n3  n2  1  3 thỏa mãn)
Xét n ≥ 2 ta có n4  n  1  n  n3  n  1   n2  1  n3  n  1  1suy ra tồn tại các số nguyên

n  n  1  p
4 s

dương s, t sao cho s > t v|  3


n  n  1  p

t

Ta có n2  1   n4  n  1  n  n3  n  1  p s  npt suy ra n2  1  pt
 n2  1  n3  n  1  n  n2  n  1  0 . Vô lý.
Vậy tất cả các giá trị cần tìm cua rn là n = 1.

Đề 47

 5 x  2 xy  2 y  2 x  2 xy  5 y  3  x  y 
2 2 2 2
(1)
Câu 1. Ta có: 
 2 x  y  1  2 7 x  12 y  8  2 xy  y  5
 (2)
3

Từ phương trình (1)  x  y  0 và


5 x 2  2 xy  2 y 2  2 x 2  2 xy  5 y 2   2x  y    x  y    x  2y   x  y
2 2 2 2

  2x  y    x  2y  2x  y  x  2 y  3 x  y 
2 2

Dấu “=” xảy ra  x  y  0


Thế y  x v|o (2), ta được: 3x  1  2. 3 19 x  8  2 x2  x  5
Từ phương trình (3)  3x  1   x  1  2  3 19 x  8   x  2   2 x 2  2 x

 x2  x 2  x3  6 x 2  7 x 
   2 x2  2x
3x  1  x  1 19 x  8   x  2  19 x  8   x  2 
3 3 2

 x2  x 2  x 2  x   x  7 
   2  x2  x   0
3x  1  x  1 19 x  8   x  2  19 x  8   x  2 
3 3 2

 x2  x  0

 1 2  x  7
 3 x  1  x  1 3 19 x  8  x  2 3 19 x  8  x  2 2  2  0 (*)

    
228
Page: Tài Liệu Môn Toán
Website: tailieumontoan.com

Vì x  0 nên (*) vô nghiệm. Do đó (3)  x  0 hay


x 1
Vậy hệ phương trình có nghiệm  x; y   0;0  , 1;1
Câu 2. Xét hệ trục Oxy sao cho A(0;1), B(0;-1)
Ta có: (O): x2 + y2 = 1; C ∈ (O) nên C(cost;sint).
Vì C không trùng A v| B nên cost ≠0.
CP là tiếp tuyên của (O) tại C
 CP: cost.x + sint.y – 1 = 0
1  sin t
N(xN;-1) ∈ CP  N ( ; 1)
cos t
 1  sin t  1
Đường thẳng AN có vecto chỉ phương l| AN   ; 2   1  sin t; 2cos t 
 cos t  cos t
 AN : 2x cos t  1  sin t  y  1  0  AN  2x cos t  1  sin t  y  1  sin t
BD : 1  sin t  x  2cos t  y  1  0  BD : 1  sin t  x  2y cos t  2cos t
Ta có D  AN  BD nên tọa độ D thỏa hệ:
2 x cos t  1  sin t  y  1  sin t
 4cos t 5sin t  3
 x ;y
1  sin t  x  2 y cos t  2cos t
 5  3sin t 5  3sin t
4cos t 5sin t  3
DP:  .x  . y  1  0 (Do DP là tiếp tuyến cảu (O) tại D)
5  3sin t 5  3sin t
 4cos t.x   5sin t  3 . y  5  3sin t
Vì P  DP  CP nên tọa độ P thỏa hệ:
4cos t.x   5sin t  3 . y  3sin t  5 
 4 x cos t   5 y  3 sin t  3 y  5
 
cos t.x  sin t. y  1
 4 x cos t  4 y sin t  4

3 1  y 
2
3y 1
 sin t  và  cos t  . Vì cost ≠ 0 nên y ≠ 1 v| y ≠ -1
y 3 x  y  3
Ta có sin 2 t  cos2t  1  x 2  3 y  1  9 1  y 2   x 2  y  3
2 2 2

 9 x 2 y 2  6 x2 y  x 2  9 1  y 2   x 2 y 2  6 x 2 y  9 x 2  0
2

 8x 2  y 2  1  9 1  y 2   0  8x 2  9  y 2  1  0 (vì 1  y 2  0)
2

x2 x2
  y  1 . vậy P thuộc elip (E):
2
 y2  1
9 9
8 8

Câu 3. Áp dụng bất đẳng thức BCS ta có: (7a2 + b2 + c2) (7 + 1 + 1) ≥ (7a + b + c)2

229
Page: Tài Liệu Môn Toán
Website: tailieumontoan.com

1 9 1 3
   
7a  b  c  7a  b  c  7a 2  b 2  c 2 7a  b  c
2 2 2 2

a 3a
 
7a 2  b 2  c 2 7a  b  c

1 1 1 2 1 3a 1 2 a
  (  )  (  )
7a  b  c 3a  3a  a  b  c 9 3a a  b  c 7a  b  c 3 3 a  b  c
a 1 2 a
Do đó:  (  )
7a  b  c
2 2 2 3 3 abc
b 1 2 b
Tương tự ta có:  (  )
a 2  7b2  c 2 3 3 a  b  c
c 1 2 c
 (  )
a  b  7c
2 2 2 3 3 abc
a b c
Cộng vế theo vế ta được:   1
7a 2  b 2  c 2 a 2  7b2  c 2 a 2  b 2  7c 2
Câu 4. Không mất tính tổng quát, giả sử x ≥ y. Xét gi{ trị k nguyên dương sao cho
phương trình đã cho có nghiệm nguyên dương. Trong c{c nghiệm ấy ta gọi (x0;y0) là
nghiệm sao cho x0≥y0≥1 v| x0+ y0 nhỏ nhất.
Ta có x02   ky0  1 x0  y02  y0  0 nên x0 là nghiệm của phương trình.
f  x   x02   ky0  1 x0  y02  y0  0
Vì f(x) là bậc 2 nên f(x) còn có thêm nghiệm là x1 + y0 ≥ x0+ y0  x0≥y0≥1
Khi đó y0 nằm ngoài khoảng hai nghiệm của tam thức bậc hai f(x) có hệ số bậc 2 là số
dương. Từ đó f(y0)≥0.
2
Do f(y0) = 2y02  y0  ky02 nên ta có k  2   4 (vì y0 ≥ 1). Suy ra k ∈ {1;2;3;4}
y0
y 3
+) Với k = 1 thì (1)  x2 + y2 + x + y = xy  (x  )2  y 2  x  y  0 (vô lí)
2 2
+) Với k = 2 thì (1)  x2 + y2 + x + y = 2xy   x  y   x  y  0 (vô lí)
2

+) Với k = 3 thì (1)  x2 + y2 + x + y = 3xy có nghiệm (x;y) = (2;2).


+) Với k = 4 thì (1)  x2 + y2 + x + y = 4xy có nghiệm (x;y) = (1;1).
Câu 5. Ta chứng minh bằng quy nạp theo n.
+) Với n = 2;
Giả sử bốn vận động viên tham gia l| A, B, C, D v| có 5 v{n đấu diễn ra.
Nếu hai trong ba người BCD đều đã đấu với nhau một v{n thì ta có điều phải chứng
minh.

230
Page: Tài Liệu Môn Toán
Website: tailieumontoan.com

Nếu có hai trong ba người BCD chưa đấu với nhau. giả sử B v| C chưa đối với nhau thì
do số trận tối đa l| C42  1  5 m| đã có 5 v{n diễn ra nên chỉ có B v| C l| chưa đó với
nhau. khi đó ba người A B C D v| E C D thỏa mãn yêu cầu b|i to{n.
+) Giả sử b|i To{n đúng với n = k  k  *
, k  2

+) Ta Chứng minh b|i to{n đúng với n = k +1


Giả sử E v| F l| 2 vận động viên đã đấu với nhau
nếu tổng số v{n đấu của 2k vận động viên còn lại lớn hơn hoặc bằng k2 + 1Thì theo giả
thiết quy nạp ta có điều phải chứng minh.
Nếu tổng số v{n đấu giữa 2k vận động viên còn lại nhỏ hơn hoặc bằng k2 m| tại thời
điểm n|y có
(k + 1)2 + 1 = k2 + 2k + 2 v{n đấu diễn ra nên tổng số v{n m| E v| F đã đấu lớn hơn hoặc
bằng 2k + 2 ( kể cả v{n đấu giữa E v| F). Suy ra số v{n đấu giữa E, F với nhóm 2k vận
động viên lớn hơn hoặc bằng 2k + 1 (*).
Nhận xét: Nếu không có người n|o trong nhóm 2k vận động viên đã thi đấu với cả E v|
F thì số phấn đấu tối đa l| 2k (m}u thuẩn với (*))
do đó trong số 2k vận động viên còn lại, phải có ít nhất một người đã thi đấu với E v| F
( Giả sử l| G). khi đó ta có 3 vận động viên E F G thỏa yêu cầu b|i to{n.
Vậy b|i to{n được chứng minh.
Câu 6: Ta có f(x) + f(x+1) = f(x+2).f(x+3) – 168
f(k+1) + f(x+2) = f(x+3).f(x+4) – 168
Do đó k  *
thì f(x+2) – f(x) = f(x+3).  f  k  4   f  k  2 

Suy ra rằng:
f(3) – f(1) = f(4).f(6) ... f(2k).  f  2k  1  f  2k  1 (1)

f(4) – f(2) = f(5).f(7) ... f(2k+1).  f  2k  2   f  2k  (2)

Do đó: f  3  f 1  f  4  . f  6  ... f  2k  . f  2 x  1  f  2k  1 với k ∈ *


,k  2

Nếu f(3) ≠ f(1) thì f(2k+1) ≠ f(2k-1)


Vì f  2x  1  f  2k  1 l| số nguyên dương nên

f  2x  1  f  2k  1  1 v| f(n) ≥ 2, x  *

Do đó f  3  f 1  2k 1 , với k ∈ *
,k  2
231
Page: Tài Liệu Môn Toán
Website: tailieumontoan.com

Điều n|y không thể xảy ra. Vậy f(3) = f(1) suy ra f(2x+1) = f(2x-1) = a
Tương tự f(2k+2) = f(2k) = b với a, b ∈ *
,a, b  2

Giả thuyết : a + b = ab – 168  ab – a – b + 1 = 169 = 132  (a – 1)(b – 1)=132


b  14
a  1  169 a  1  1 
 a – 1 = b – 1 = 13 hoặc  hoặc   b  2
b  1  1 b  1  169 b  170

Vậy f(2014) = 2 hoặc f(2014) = 14 hoăc f(2014) = 170

232
Page: Tài Liệu Môn Toán
Website: tailieumontoan.com

233
Page: Tài Liệu Môn Toán

You might also like